КулЛиб - Классная библиотека! Скачать книги бесплатно 

Геометрия. Профильный уровень. Методическое пособие для 10 класса [Валерий Александрович Гусев] (pdf) читать онлайн

Книга в формате pdf! Изображения и текст могут не отображаться!


 [Настройки текста]  [Cбросить фильтры]
ГЕОМЕТРИЯ
ПРOФИЛЬНЫЙ УРОВЕНЬ

Методическое пособие
для 10 класса
ЭЛЕКТРОННОЕ
ИЗДАНИЕ

Москва
БИНОМ. Лаборатория знаний

УДК 372.016:514(072)
ББК 22.151.0я721.6
Г36

А в т о р ы:
В. А. Гусев, Л. Н. Ерганжиева, А. Б. Пятерикова,
В. И. Хорхордин, И. Г. Шведова
Геометрия. Профильный уровень [Электронный реГ36 сурс] : методическое пособие для 10 класса / В. А. Гусев [и др.]. — Эл. изд. — Электрон. текстовые дан.
(1 файл pdf : 223 с.). — М. : БИНОМ. Лаборатория знаний, 2014. — Систем. требования: Adobe Reader XI ;
экран 10".
ISBN 978-5-94774-931-1
Методическое пособие к учебнику геометрии для 10 класса
является частью учебно-методического комплекта для старших классов школ с изучением математики на профильном
уровне. Приведены варианты контрольных работ к каждой
теме соответствующего учебника, методические рекомендации для учителей, дидактические материалы и поурочное
планирование, подробно разобраны сложные задачи из учебника.
Для учителей, работающих в классах физико-математического и естественно-научных профилей.
УДК 372.016:514(072)
ББК 22.151.0я721.6

В соответствии со ст. 1299 и 1301 ГК РФ при устранении
ограничений, установленных техническими средствами защиты
авторских
прав,
правообладатель
вправе
требовать
от
нарушителя возмещения убытков или выплаты компенсации
ISBN 978-5-94774-931-1

c БИНОМ. Лаборатория знаний, 2014


ПРЕДИСЛОВИЕ

Предлагаемое вниманию учителя геометрии пособие
содержит методические рекомендации и дидактические материалы к учебнику «Геометрия. Профильный
уровень: учебник для 10 класса», авторами которого являются В. А. Гусев, Е. Д. Куланин, А. Г. Мякишев, С. Н. Федин. Авторы предоставляют учителю
методическую поддержку на сайте авторской мастерской http://metodist.lbz.ru/authors/matematika/3/. Там же
можно скачать методические рекомендации по преподаванию предмета и по работе с цифровыми образовательными
ресурсами.
Содержание учебника соответствует возрастным и психологическим особенностям школьников старших классов,
имеющих определенный интерес к математике. Учебник
содержит систематическое изложение содержания геометрии на профильном уровне.
Структура методического пособия в основном соответствует содержанию учебника; оно состоит из семи глав.
Глава 1. Планиметрия
Раньше планиметрия не входила в курс геометрии для
10–11 классов даже при углубленном ее изучении. Первая
глава методического пособия посвящена планиметрии,
работу над этой темой рекомендуется равномерно распределить по учебному плану.
Глава 2. Параллельные прямые и плоскости
В этой главе достаточно полно изучен вопрос об аксиоматическом построении курса стереометрии. Анализируется
вся система аксиом пространства и доказательства первых
теорем стереометрии. Показано, как изучение геометрии
окружающего пространства учитель может использовать
для вовлечения всех учащихся в процесс размышлений,
для активации их творческой деятельности.

4

Предисловие

Глава 3. Векторы и координаты в пространстве
В методическом пособии показана вся структура использования векторного аппарата при изучении фигур в пространстве. Очень важно, что это сделано параллельно
с введением метода координат; таким образом, выделено
особое место координатно-векторного метода при изучении
курса стереометрии.
Глава 4. Перпендикулярность прямых и плоскостей
в пространстве
Материал этой главы достаточно традиционен. Вместе с тем
обращается особое внимание на изложение вопросов, связанных с признаком перпендикулярности прямой и плоскости. Во многих действующих учебниках эти вопросы
изложены поверхностно. В этом методическом пособии
четко выделены частности и общий подход к рассмотрению вопросов, связанных с перпендикулярностью прямой
и плоскости, подробно объяснена сущность определения
перпендикуляра к плоскости.
Глава 5. Многогранные углы
Весь материал о свойствах трехгранных и многогранных
углов собран вместе. Это очень удобно, и это дает
возможность в дальнейшем «плавно» перейти к изучению
свойств многогранников.
Глава 6. Контрольные работы
В этой главе даются варианты контрольных работ по
курсу стереометрии. Решения задач, аналогичных предлагающимся в контрольных работах, подробно разобраны
в основном тексте пособия, и к ним даются только
ответы. Однако для удобства учителя к некоторым задачам
приводятся и решения.
Глава 7. Планирование учебного материала
В этой главе приводится поурочное планирование из расчета 105 часов в год.
Представляется, что этот материал поможет учителям
успешно работать по учебнику геометрии для 10 класса.
Доктор педагогических наук, профессор

В. А. Гусев

Глава 1

ПЛАНИМЕТРИЯ

Последовательность рассмотрения вопросов планиметрии
(12 часов).
1. Геометрия
треугольника — основные
свойства
и вычисление его элементов (2 часа).
2. Подобие треугольников. Метод подобия.
3. Теорема Чевы и теорема Менелая.
4. Формулы площади треугольника. Метод площадей.
5. Углы, связанные с окружностью.
6. Пропорциональные отрезки, связанные с окружностью.
7. Вписанные и описанные четырехугольники.
8. Геометрические места точек.
9. Эллипс, гипербола и парабола как ГМТ.
10. Решение задач с помощью геометрических преобразований.
11. Задачи на построение. Неразрешимость классических задач на построение с помощью циркуля
и линейки.
Традицией российской школы является последовательное изучение систематических курсов планиметрии
и стереометрии в рамках единого учебного предмета —
геометрии. В то же время геометрическая содержательнометодическая линия проходит через всю школьную программу с 1 по 11 класс, что предполагает непременное
включение в курс математики отдельных геометрических
вопросов с ознакомительными целями. Тем самым реализуется дидактический принцип концентризма в изучении
материала, что позволяет наилучшим образом учитывать
индивидуальные и возрастные особенности учащихся без
нарушения принципа научности обучения.

6

Глава 1. Планиметрия

Изучение систематического курса планиметрии завершается в 9 классе, однако очевидно, что богатейший
и обширнейший теоретический материал, красота планиметрических задач, возможность задействовать в работе
над ними и образное, и логическое мышление, тем самым
развивая творческие способности учащихся, с необходимостью требуют включения элементов планиметрии в курс
геометрии 10–11 классов.
В большинстве программ по геометрии для 10 класса
рассмотрение отдельных вопросов планиметрии выделяется в небольшой модуль «Повторение», который предваряет изучение первых разделов стереометрии. Это оправдано с той точки зрения, что помогает осуществлению
плавного перехода от изучения двумерного пространства
к трехмерному пространству. Но существенным недостатком такого выстраивания геометрической линии является ощущение искусственности происходящего: крайне
трудно перекидывать мосты в неведомое, коим является
стереометрия. Поэтому мы предлагаем распределить уроки,
посвященные вопросам планиметрии, более или менее
равномерно по всему учебному плану. При этом 12 часов,
отведенные на повторение, распределятся по одному уроку
раз в две недели. Кроме того, и в плановые уроки стереометрии можно понемногу вводить повторение основных
фактов первой главы учебника — главы «Планиметрия».
Учителя математики, работающие в профильных математических классах, достаточно квалифицированны, и нет
необходимости регламентировать организационные формы
и методы их работы. Поэтому основной своей целью мы
считаем наполнение уроков задачным материалом.
По каждой теме мы предлагаем несколько задач различного уровня сложности — от тренировочных до конкурсных. Объединенные общей идеей, они могут быть
выстроены в цепочку «от простого к сложному» или же,
напротив, разбор сложной задачи потребует отвлечения
на более мелкие и очевидные. Данные в задачах мы
представляем в виде параметров, что поможет учителю
организовать самостоятельную работу учащихся, задавая
конкретные числовые значения. Безусловно, предлагаемые
задачи могут быть заменены учителем по его желанию,

§ 1. Геометрия треугольника — основные свойства

7

так как являются лишь образцами задач по той или иной
теме.
§ 1.

ГЕОМЕТРИЯ ТРЕУГОЛЬНИКА — ОСНОВНЫЕ СВОЙСТВА
И ВЫЧИСЛЕНИЕ ЕГО ЭЛЕМЕНТОВ

Основная цель первых двух уроков — актуализация знаний учащихся, полученных при изучении планиметрии.
Поэтому эти два урока следует посвятить основным
понятиям и теоремам, связанным с треугольником, а также
вспомнить основные формулы. Теоретический материал
можно рассматривать в том порядке, в каком он изложен
в первом параграфе «Метрические соотношения в треугольнике. Решение треугольников» учебника:
1. Теорема о сумме углов треугольника.
2. Теорема синусов.
3. Теорема косинусов.
4. Основное свойство биссектрисы треугольника.
5. Формула Стюарта.
6. Формула длины медианы.
7. Тождество параллелограмма.
8. Первая формула биссектрисы.
9. Вторая формула биссектрисы.
10. Теорема об отрезках касательных для вписанной
окружности.
11. Теорема об отрезках касательных для вневписанной
окружности.
12. Формулы площади треугольника.
В связи с нехваткой времени лишь некоторые теоремы
можно доказать на уроке, причем остановиться на основных моментах или идеях доказательств.
Основные теоремы планиметрии учащиеся могут предварительно повторить самостоятельно, чтобы сэкономить
время на уроке.
Полезно вначале повторить и стандартные условные
обозначения основных элементов треугольника, чтобы
в дальнейшем не возникало разночтений в формулах.
Проверку можно организовать в виде зачета, включив
в него основные теоремы и формулы, приведенные в учебнике.

8

Глава 1. Планиметрия

§ 2.

ПОДОБИЕ ТРЕУГОЛЬНИКОВ. МЕТОД ПОДОБИЯ

Среди задач, решаемых методом подобия, можно выделить
две группы наиболее часто встречающихся задач: задачи
на отношение длин пересекающихся отрезков и задачи,
связанные со вспомогательной окружностью, в которых
подобие треугольников доказывается путем обнаружения
равных углов, связанных с окружностью.
Первый тип задач необходим в решении стереометрических задач на отношение длин отрезков, связанных
с параллелепипедом и призмой, поэтому может считаться
базовым в геометрии.
Задача 1. На сторонах BC и CD параллелограмма
ABCD взяты соответственно точки M и N так, что
BM : MC = m : n, CN : ND = p : q. Отрезки MN и AC пересекаются в точке O. Найдите отношения, в которых точка O
делит каждый из них.
Указание. Продлим отрезок MN до пересечения с продолжениями сторон AD и AB и рассмотрим пары подобных
треугольников, коэффициенты подобия которых определяются данными в условии отношениями и тем, что
противоположные стороны параллелограмма равны.
Заметим, что аналогичные задачи можно составить,
заменив параллелограмм трапецией с известным отношением оснований.
Решение задач второго типа требует от учащихся
умения рассматривать рисунок и вычленять на нем углы
(вписанные в окружность, углы между касательными
и хордами и проч.), опирающиеся на одну дугу или
равные дуги. Две пары равных углов, в свою очередь,
позволят обнаружить подобные треугольники. Хорошо
иллюстрирует это следующая задача.
Задача 2. Из точки M, лежащей вне окружности, проведены к этой окружности две касательные. Расстояния от
точки C, лежащей на окружности, до касательных равны
a и b. Найдите расстояние от точки C до прямой AB, где
A и B — точки касания.
Решение. Пусть P, Q, N — основания перпендикуляров,
опущенных из точки C на прямые MA, MB, AB

§ 2. Подобие треугольников. Метод подобия

9

соответственно. Тогда четырехугольники APCN и BQCN
вписанные, так как в них суммы противоположных углов
равны 180◦ . Поэтому углы CPN, CAN и CAB равны; а из
теоремы об угле между касательной и хордой следует, что
равны углы CAB, CBQ и CNQ, значит, угол CPN равен
углу CNQ. Аналогично, равны углы CNP и CQN.
Значит, треугольники PCN и NCQ подобны по двум
2
углам. Тогда CN : CQ =
√CP : CN, поэтому CN = CP · CQ = ab.
Следовательно, CN = ab.
В поиске равных углов при доказательстве подобия треугольников может помочь прием «перебрасывания углов»,
который показывает лемма Фуса.
Лемма Фуса. Даны две пересекающиеся в точках A
и B окружности. Если на одной из них выбраны точки
P1 и P2 , а прямые P1 A и P2 B пересекают вторую
окружность в точках Q1 и Q2 соответственно, то
прямые P1 P2 и Q1 Q2 параллельны.
Доказательство состоит в перебрасывании углов с одной
окружности на другую. Четырехугольник P2 P1 AB — вписанный, следовательно, сумма углов P1 и B равна 180◦ ,
следовательно, углы P2 P1 A и ABQ2 равны. Аналогично,
равны и углы ABQ2 и ABM. Таким образом, равны углы
P2 P1 A и P1 Q1 M, следовательно, прямые P1 P2 и Q1 Q2
параллельны.
На этом же уроке можно доказать теорему о том, что
отрезок, соединяющий основания двух высот треугольника, отсекает от него треугольник, подобный данному
с коэффициентом подобия, равным модулю косинуса
общего (равного в случае тупоугольного треугольника)
угла, и решить задачу с применением этого свойства.
Задача 3. В остроугольном треугольнике ABC из вершин
A и C опущены высоты AA1 и CC1 . Известно, что площадь
треугольника ABC равна S, площадь треугольника A1 BC1
равна s, длина отрезка A1 C1 равна m. Найдите радиус
описанной около треугольника ABC окружности.
Указание. Треугольники A1 BC1 и ABC подобны по двум
углам, коэффициент подобия определяется из отношения

10

Глава 1. Планиметрия

площадей и равен cos B. Найдя sin B и длину AB, по
теореме синусов найдем радиус описанной окружности.
§ 3.

ТЕОРЕМА ЧЕВЫ И ТЕОРЕМА МЕНЕЛАЯ

В курсе планиметрии общеобразовательной школы теоремы
Чевы и Менелая предлагается изучать как дополнительный материал, поэтому не все десятиклассники знакомы
с ними. В связи с этим необходимо разобрать их доказательства в классе и обязательно остановиться на тригонометрической форме этих теорем. Знание этих теорем
и их применение к решению задач во многих случаях
упрощают и значительно сокращают время, затрачиваемое
на решение, что крайне важно в условиях экзамена.
Так, задачи на вычисление отношений отрезков в треугольниках, весьма популярные в практике вступительных
экзаменов, могут быть решены как минимум шестью
различными способами, известными учащимся (по теореме
Фалеса, методом подобия, методом площадей, векторным
методом, координатным методом, по теореме Менелая), но
именно с использованием теоремы Менелая их решение
становится устным. Кроме того, первые аффинные задачи
курса стереометрии на вычисление отношения, в котором
сечение делит ребро пирамиды, требуют от учащихся
свободного владения теоремой Менелая. Теорема Чевы
позволяет эффективно доказывать многие факты геометрии
треугольника. Полезно будет применить эту теорему для
доказательства существования следующих точек: центров
вневписанных для треугольника окружностей, существования точки Жергонна J (точки пересечения прямых,
проходящих через вершины треугольника и точки касания
вписанной окружности с противолежащими вершинам
сторонами) и точки Нагеля N (точки пересечения прямых,
проходящих через вершины треугольника и точки касания
вневписанной окружности с противолежащими вершинам
сторонами).
В качестве тренировочных упражнений на применение
теоремы Менелая можно предложить базовые задачи на
вычисление отношений пересекающихся отрезков в треугольнике.

§ 4. Площадь треугольника

11

Задача 1. На стороне AC треугольника ABC взята точка
B1 так, что AB1 : B1 C = m : n, а на стороне BC — точка A1
такая, что BA1 : A1 C = k : l. Отрезки BB1 и AA1 пересекаются в точке O. Найдите отношения AO : OA1 и BO : OB1.
Задача 2. На сторонах AC и BC треугольника ABC взяты
точки B1 и A1 соответственно. Отрезки BB1 и AA1
пересекаются в точке O. Известно, что BO : OB1 = m : n
и AO : OA1 = k : l. Найдите отношения, в которых точки A1
и B1 делят соответствующие стороны.
§ 4.

ПЛОЩАДЬ ТРЕУГОЛЬНИКА

Повторив в начале урока основные формулы площади
треугольника (полезно при этом остановиться и на выводах
этих формул), следует перейти к следствиям. А именно
рассмотреть отношение площадей треугольников с равными высотами — в этом случае площади треугольников
относятся как длины сторон, к которым проведены высоты,
и отношение площадей треугольников с равными углами
(или углами, дополняющими друг друга до 180◦ ) — в этом
случае площади треугольников относятся как произведения
длин сторон этих углов. Эти следствия крайне полезны, так
как они будут использованы при выводе свойств объемов
тетраэдров с равными высотами или равными (смежными)
трехгранными углами. Кроме того, эти свойства площадей
треугольников часто эксплуатируются в практике вступительных экзаменов.
Задача 1. На сторонах AB, BC и CA треугольника
ABC взяты точки K, M и P соответственно так, что
AK : KB = a : b, BM : MC = c : d, CP : PA = e : f. Найдите площадь треугольника KMP, если площадь треугольника ABC
равна 1.
Указание. Площадь искомого треугольника равна разности площади треугольника ABC и площадей треугольников
AKM, BKM и CPM, найденных с использованием свойства
площадей треугольников с общим углом.
Задача 2. На сторонах AB, BC и CA треугольника ABC
взяты точки K, M и P. Известно, что AK : KB = a : 1,

12

Глава 1. Планиметрия

BM : MC = b : 1 и что треугольники AKP и CMP равновелики. Найдите CP : PA.
Указание. Пусть CP : PA = k : 1. Выразите площади треугольников AKP и CMP через площадь данного треугольника и приравняйте их.
О т в е т:

m(n + 1)
.
m+1

На рассматриваемых свойствах основан метод площадей, когда, например, требуется выразить отношение длин
отрезков. Примером может служить теорема Менелая.
Задача 3. Докажите необходимое условие теоремы Менелая
методом площадей.
Доказательство. Пусть в треугольнике ABC проведена
секущая, пересекающая стороны AB, BC и продолжение
стороны AC в точках C1 , A1 и B1 соответственно.
Обозначим длины получившихся отрезков следующим
образом: AC1 = a, C1 B = b, BA1 = c, A1 C = d, CB1 = e,
CA = f. Проведем прямые BB1 и AA1 и выразим отношение площадей треугольников через введенные длины
отрезков: SBA B : SA CB = c : d, SA CB : SAA B = e : (e + f),
1 1
1
1
1
1
1 1
SAA B : SBA B = a : b. Перемножив левые части этих
1 1
1 1
равенств, получим 1, следовательно, и произведение правых частей равенств равно 1, что и требовалось доказать.
Задача 4. Точка D делит сторону BC треугольника ABC
в отношении 1 : m, считая от B. Точка E делит сторону
AC в отношении 1 : n, считая от C. Какую часть площади
треугольника ABC составляет площадь треугольника BKD,
где K — точка пересечения BE и AD?
Указание. Воспользовавшись теоремой Менелая, можно
найти отношение BK : KE, после чего, определив отношение площадей треугольников BKE и ABC, найдем, какую
часть площади треугольника BEC составляет площадь
треугольника BKD и, окончательно, какую часть площади
треугольника ABC составляет площадь треугольника BKD.
О т в е т:

1
.
(m + 1)(mn + n + 1)

§ 5. Углы, связанные с окружностью

13

Задача 5. Прямые AD и CE разбивают треугольник ABC
на три треугольника и четырехугольник. Найдите площадь
треугольника, прилежащего к стороне AC, если площадь
каждой из трех оставшихся фигур равна S.
Указание. Пусть AD и CE пересекаются в точке M. Тогда
AM : MD = CM : ME = X : S, где X — площадь треугольника
AMC, и треугольники EMD и AMC подобны с коэффициентом k = S : X. Значит, отрезки ED и AC параллельны
и относятся как k. Площадь треугольника EMD равна
S2 : X, площадь треугольника EBD равна S − (S2 : X).
Треугольники EBD и ABC подобны с коэффициентом k,
2
SX − S2
S
=
, откуда X = 3S.
следовательно,
X(X + 3S)

X

О т в е т: 3S.
Задача 6. На сторонах AB, BC и CA треугольника ABC
взяты точки D, E и F соответственно так, что AD = DB,
BE : EC = 1 : m, CF : FA = 1 : n. При пересечении прямых
CD, AE и BF образуется треугольник. Найдите отношение
его площади к площади треугольника ABC.
Указание. Можно соединить вершины образовавшегося
треугольника с вершинами треугольника ABC и найти
отношение площадей образовавшихся 9 треугольников
к площади данного треугольника. Применить теорему
Менелая.
О т в е т:
§ 5.

1
n
n
+

.
4m + 2
2n + 4
mn + n + 1

УГЛЫ, СВЯЗАННЫЕ С ОКРУЖНОСТЬЮ

В этой теме мы предлагаем решить задачи на прямой
подсчет углов, связанных с окружностью, задачи на доказательство равенства углов, а также две известные и сложные
задачи на подсчет углов треугольника с данными углами,
выраженными круглыми числами. При решении двух
последних задач применим тот факт, что некоторые
диагонали правильного 18-угольника пересекаются в одной

14

Глава 1. Планиметрия

точке (который доказывается с помощью теоремы Чевы
в тригонометрической форме).
Задача 1. Окружность разделена точками A, B, C, D так,
что градусные меры дуг AB, BC, CD и DA относятся как
3 : 2 : 13 : 7. Хорды AD и BC продолжены до пересечения
в точке M. Найдите угол AMB.
О т в е т: 72◦ .
Задача 2. На окружности последовательно отмечены точки
A, B, C, D. Точки M, N и K — середины хорд AB, BC
и CD соответственно. Докажите, что углы BMN и NKC
равны.
Указание. Рассмотрите MN и NK как средние линии
треугольников ABC и BCD соответственно.
Задача 3. В равнобедренном треугольнике угол при вершине равен 2α. На боковой стороне как на хорде построена
окружность, которая касается основания в его конце. Две
вершины и точка пересечения окружности с другой боковой
стороной делят окружность на три части. Найдите их
угловые величины.
О т в е т: 4α, 180◦ − 6α, 180◦ + 2α.

Задача 4. В треугольнике ABC AB = BC, угол B равен 20◦ .
Точки D и E лежат на сторонах BC и BA соответственно,
причем угол DAC равен 60◦ , угол ECA равен 50◦ . Найти
угол ADE.
О т в е т: 30◦ .
Задача 5. В треугольнике ABC AB = AC. Внутри треугольника взята точка M так, что угол ABM равен 20◦ , угол
CBM равен 30◦ , угол ACM равен 40◦ . Найти угол AMC.
О т в е т: 70◦ .
Указание. Как уже говорилось, задачи 4 и 5 связаны
с правильным 18-угольником. Каждая его сторона стягивает дугу описанной окружности величиной в 20◦ . Угол
A14 A1 A11 — искомый в задаче 4 угол, если диагонали
A1 A14 , A11 A17 и A7 A16 пересекаются в одной точке M.

§ 5. Углы, связанные с окружностью

15

Для доказательства этого факта достаточно воспользоваться теоремой Чевы в тригонометрической форме для
треугольника A1 A11 A16 . Углы между чевианами этого
треугольника и сторонами равны 40◦ , 60◦ , 20◦ , 30◦ , 20◦ ,
10◦ (рис. 1.1).
A1
A17


20◦ 30
A16

60◦
40◦ M

10◦
A14
20◦

50◦
A7
60◦

A11

Рис. 1.1.

По теореме Чевы, диагонали A1 A14 , A11 A17 , A7 A16 , на
которых лежат чевианы треугольника A1 A11 A16 , пересекаются в одной точке, если
sin 40◦ · sin 20◦ · sin 20◦ = 1.
sin 60◦ sin 30◦ sin 10◦

Преобразуем левую часть этого равенства:





2 sin 40◦ sin
√ 20 sin 80 = 2 · 2 · 2 sin 20 √sin 10 sin 80 =
3 1
3
2 2
4 sin 80◦ (cos 20◦ − cos 60◦ ) 4 sin 80◦ (cos 20◦ − 0,5)


=
=
=
3
3



2(cos 30◦ + cos 10◦ − cos 10◦ )

= 4 cos 10 cos√20 − 2 cos 10 =
= 1.
3
3

16

Глава 1. Планиметрия

Таким образом, диагонали пересекаются в точке M, и в
задаче 4 искомый угол равен 30◦ (угол MA1 A11 равен углу
A14 A1 A11 ).
Для решения задачи 5 рассмотрим такие диагонали
правильного 18-угольника: A1 A13 , A3 A14 , A6 A15 (рис. 1.2).
На этих диагоналях лежат чевианы треугольника A1 A6 A14 .
Если диагонали пересекаются в одной точке M, то угол
A1 MA6 — искомый и равен 70◦ .
A1
A3

A15
M
A14

A6

A13

Рис. 1.2.

Вычислим:

sin 40◦ · 1
sin 10◦ · sin 40◦ · sin 30◦ =
2
=
sin 70◦ sin 10◦ sin 20◦
cos 20◦ sin 20◦

=

2 · 1 sin 20◦ cos 20◦
2
= 1.
sin 20◦ cos 20◦

Таким образом, по теореме Чевы, диагонали пересекаются в точке M, и искомый угол равен 70◦ .
§ 6.

ПРОПОРЦИОНАЛЬНЫЕ ОТРЕЗКИ,
СВЯЗАННЫЕ С ОКРУЖНОСТЬЮ

Задача 1. На стороне AB треугольника ABC как на
диаметре построена окружность, пересекающая сторону

§ 6. Пропорциональные отрезки

17

BC в точке D. Найдите AC, если известно, что CD = a,
AB = BC = b.
Указание. Пусть окружность пересекает сторону AC
в точке M, тогда BM — высота к основанию равнобедренного треугольника, значит, и медиана. По свойству
секущих, проведенных к окружности
из одной точки,

получаем 0,5AC2 = ab, откуда AC = 2ab.

О т в е т:
2ab.
Задача 2. Вершины треугольника ABC лежат на окружности, AB : BC = a : b. Точка D делит дугу AC пополам,
отрезок BD пересекает сторону AC в точке E. Хорда
KM проходит через точку E, причем KE = m, EM = n.
Найдите AC.
Указание. Так как точка D — середина дуги AC, то DE —
биссектриса треугольника ABC, и по свойству биссектрисы
треугольника AE : EC = AB : BC.
О т в е т:


(a + b) mn
.
ab

Задача 3. В треугольнике ABC точка D — середина стороны AC, точка E — середина BC. Окружность, описанная
около треугольника DCE, проходит через точку M пересечения медиан треугольника ABC. Найдите длину медианы
CK треугольника ABC, если AB = c.
Указание. MC и DE — пересекающиеся хорды одной
1

1

окружности, причем MN = CK, CN = CK, DN = NE,
6
2
где N — точка пересечения CK и DE.

О т в е т: 0,5 c 3.
Задача 4. (Физический факультет МГУ, 2000 г., март.)
Из точки A проведены к окружности две касательные
(M и N — точки касания) и секущая, пересекающая эту
окружность в точках B и C, а хорду MN — в точке P,
причем AB : BC = 2 : 3. Найдите AP : PC.
Решение.1) Проведем AO (O — центр окружности), радиус
OM + AM и OH + AC (рис. 1.3). В точке K пересекаются
1) Медведев Г. Н. Задачи вступительных экзаменов по математике на
физическом факультете МГУ. 1971 — 2006 гг. — М.: КомКнига, 2007.

18

Глава 1. Планиметрия

M
C
H
B

P

A

K

O

N

Рис. 1.3.

отрезки AO и MN. Заметим, что треугольник AMK
подобен треугольнику AOM, треугольник APK подобен треугольнику AOH, откуда AK : AM = AM : AO (1),
AK : AP = AH : AO (2).
Далее: из AM2 = AB · AC, а также из (1) и (2)
следует AM2 = AB · AC = AK · AO = AP · AH, следовательно,
AP = AB · AC/AH (3).
Пусть AB = 2a, AC = 5a, тогда AH = 7a/2. Из (3)
тогда находим: AP = 20a/7, PC = AC − AP = 15a/7. Значит,
AP : PC = 4 : 3.
Вариант решения. Повторите чертеж по условию задачи
и проведите только AO. В окружности PM · PN = BP · PC,
следовательно, (MK − PK)(KN + PK) = BP · PC (1).
Пусть AB = 2a, BC = 3a, AP = x, угол PAK = α. Тогда
2 AK = x cos α, PK = x sin α (2), KN =
AM2 = AB
√ · AC = 10a , √
2
2
= MK = AM − AK = 10a2 − x2 cos2 α (3), BP = x − 2a (4),
PC = AC − AP = 5a − x (5).
Подставляя (2), (3), (4), (5) в (1), находим x = AP = 20a/7,
PC = 15a/7. Значит, AP : PC = 4 : 3.
О т в е т: 4 : 3.
§ 7.

ВПИСАННЫЕ И ОПИСАННЫЕ ЧЕТЫРЕХУГОЛЬНИКИ

Задача 1. В треугольнике ABC проведены высоты AA1
и BB1. Найти угол между прямыми A1 B1 и CO, где O —
центр описанной окружности.

§ 7. Вписанные и описанные четырехугольники

19

Эта же задача может быть сформулирована в следующем
виде:
Доказать, что радиус описанной окружности перпендикулярен прямой, проходящей через основания высот
на сторонах треугольника, проведенных из той же
вершины, что и радиус. При этом ответ задачи становится
известным, что может помочь в ее решении.
Указание. При решении этой задачи ключевым моментом
является тот факт, что точки A, B, A1 и B1 лежат на одной
окружности (прямые углы опираются на один отрезок AB),
из чего следует равенство углов BAA1 и BB1A1 , а значит,
и углов ABB1 и A1 B1 C. Угол ABC равен половине угла
AOC как вписанный и соответствующий ему центральный, откуда вытекает равенство углов BAA1 и OCA.
Далее, зная, что сумма углов треугольника равна 180◦ ,
приходим к выводу о перпендикулярности прямых A1 B1
и OC.
Задача 1 довольно сложная, при ее решении от учащихся требуется не только знание теории, но и умение
анализировать рисунок, включая одни и те же объекты
в различные отношения. Именно в этом состоит отличие
простой задачи от сложной. На наш взгляд она будет
полезной для демонстрации этого умения.
Из более простых, базовых, задач в данной теме
можно взять задачи на описанную трапецию. Они часто
встречаются в практике конкурсных экзаменов, в то время
как в 8 классе на них, возможно, не останавливались.
Задача 2. Докажите, что боковая сторона трапеции видна
из центра вписанной окружности под прямым углом.
Указание. Доказательство вытекает из свойства односторонних углов при параллельных прямых и секущей.
Задача 3. Найдите радиус окружности, вписанной в равнобедренную трапецию с основаниями a и b.
Указание. Боковая сторона трапеции точкой касания
вписанной окружности делится на отрезки 0,5a и 0,5b.

20

Глава 1. Планиметрия

Используя результат задачи 2, радиус окружности находим по свойству высоты прямоугольного треугольника,
проведенной к гипотенузе.

О т в е т: 0,5 ab.
Задача 4. Найдите радиус окружности, описанной около
трапеции, если даны все стороны трапеции.
Указание. Задача является базовой вычислительной задачей курса планиметрии, решение которой, однако, часто
вызывает затруднения. Эти трудности могут быть преодолены, если довести до сознания учащихся, что находить радиус описанной около многоугольника окружности
можно, заменив данный многоугольник на треугольник,
вершинами которого являются три любые вершины данного многоугольника. В таком случае достаточно найти все
стороны этого треугольника, а затем и радиус описанной
окружности. В трапеции находится высота, проводится
диагональ, вычисляется синус угла трапеции, противолежащего этой диагонали, и применяется теорема синусов.
Задача 5. Расстояния от центра вписанной в прямоугольную трапецию окружности до концов большей боковой
стороны равны a и b. Найдите площадь трапеции.
Указание. Сумма оснований описанной трапеции равна
сумме боковых сторон, одна из которых равна диаметру
вписанной окружности и равна высоте трапеции, а другая находится по теореме Пифагора (задача 2). Радиус
вписанной окружности находим, как в задаче 3.
О т в е т:

(a + b)2 ab
a2 + b2

.

Задача 6. Средняя линия равнобедренной трапеции
равна m. Известно, что в трапецию можно вписать
окружность. Средняя линия трапеции делит ее на части,
площади которых относятся как a : b. Найдите высоту
трапеции.
Указание. По свойству описанной равнобедренной трапеции ее боковая сторона равна средней линии, а высота —

§ 7. Вписанные и описанные четырехугольники

21

диаметру вписанной окружности. Используя отношение
площадей частей трапеции, найдем одно из оснований,
а затем и высоту трапеции.
О т в е т:


m (3a − b)(3b − a)
.
a+b

Задача 7. (Физический факультет МГУ, 2006 г., июль.)
В трапеции PQRS (QR k PS) PQ = RS. Две прямые,
параллельные основаниям QR и PS, делят трапецию на
3 части, в каждую из которых можно вписать окружность.
Радиус наименьшей из этих окружностей в 2 раза меньше
радиуса средней окружности. Найти отношение радиуса
наибольшей из этих окружностей к радиусу наименьшей.
Решение.1) Центры всех окружностей, касающихся сторон
PQ и RS, лежат на одной прямой O1 O3 — биссектрисе угла,
образованного продолжениями этих сторон. Заметим, что
при условии PQ 6= RS окружности, конечно, не касаются
друг друга.
Пусть r1 , r2 и r3 — радиусы наименьшей, средней
и наибольшей окружностей соответственно (рис. 1.4).
Биссектрисы смежных углов при параллельных прямых и боковой стороне PQ образуют с линией центров
окружностей два прямоугольных треугольника с катетами
a, b и c, d, причем a k c, b k d, следовательно, острые углы
в этих треугольниках равны.
Радиусы окружностей, перпендикулярные стороне PQ,
образуют с отрезками a, b, c, d еще две пары прямоугольных треугольников с катетами r1 , r2 , r3 и гипотенузами
a, b, c, d.
Треугольники с катетами a, b и c, d подобны, следовательно, есть еще две пары подобных треугольников со
сторонами r1 , a, r2 , c и r2 , b, r3 , d соответственно, значит,
2
r
r1
r2
r2
a
b
2
= = = , r2 = r1 r3 , следовательно,
= 3.
r2

c

d

r3

r1

r1

Задавая в последнем равенстве одно отношение, находим
r
r
другое. По условию 2 = 2, следовательно, 3 = 4.
r1

r1

1) Медведев Г. Н. Задачи вступительных экзаменов по математике на
физическом факультете МГУ. 1971–2006 гг. — М.: КомКнига, 2007.

22

Глава 1. Планиметрия
Q

Q

R
r1 O1

r1 O1

u

a

R

L

b
r2

v

O2

r2

O2

c
N
d
r3

w

O3

S

P

r3

O3

S

P

Рис. 1.4.

Рис. 1.5.

Вариант решения (рис. 1.5). Рассмотрим две тройки
параллельных прямых O1 Q k O2L k O3 N и O1 L k O2 N k O3 P,
следовательно, треугольники O1 QL, O2 LN, O3 NP подобны.
Пусть QL = u, LN = v, NS = w. По теореме Фалеса, для
указанных троек параллельных прямых
O1 O2
v
=u
v=w .
O2 O3

(1)

Из подобия указанных треугольников (r1 , r2 и r3 в них —
высоты)
r1
u
r2 = v ,

Из (1) и (2) следует
О т в е т: 4.

r2
v
r3 = w .

r1
r
= 2 , т. е. r22 = r1 r3 и т. д.
r2
r3

(2)

§ 8. Геометрические места точек

§ 8.

23

ГЕОМЕТРИЧЕСКИЕ МЕСТА ТОЧЕК

Задачи на геометрические места точек можно разделить
на две группы. К первой относятся задачи на нахождение
ГМТ по описанию. Решение этих задачи вызывает большие
трудности у учащихся. Поэтому на уроке можно подробно
разобрать решение одной из приведенных в учебнике,
а затем продемонстрировать поиск ГМТ с помощью координатного метода. Например, можно найти ГМТ, для
которых задано отношение расстояний до двух данных
точек (окружность Аполлония).
Задача. Пусть даны точки A и B и число λ > 0. Найти
множество всех таких точек M плоскости, что AM : MB = λ.
Решение. Так как λ > 0, то AM2 = λ2 · MB2. По формуле
расстояния между двумя точками, (x − xА )2 + (y − yA )2 =
= λ2 ((x − xB)2 + (y − yB)2 ). Раскрывая скобки, получим
(λ2 − 1)x2 + (λ2 − 1)y2 + ax + by + c = 0.
Из этого равенства видно, что если λ = 1, то уравнение
принимает вид ax + by + c = 0. Это уравнение серединного
перпендикуляра к отрезку AB. В противном случае это
уравнение окружности.
Окружность Аполлония часто применяется в задачах на
построение, поэтому, на наш взгляд, будет полезно на этом
же уроке построить ее с помощью циркуля и линейки для
заданных точек A и B и конкретного числа λ, например
λ = 2/3.
План построения.
1. С использованием теоремы Фалеса строим точку P1 ,
делящую отрезок AB внутренним образом в отношении 2/3.
2. Строим точку P2 , делящую отрезок AB внешним
образом в отношении 2/3.
3. Находим середину O отрезка P1 P2 .
4. Строим окружность с центром O и радиусом, равным
OP1 — окружность Аполлония.
Получение уравнения окружности координатным методом подготовит учащихся к изучению следующей темы —
«Эллипс, гипербола и парабола как ГМТ».

24

Глава 1. Планиметрия

§ 9.

ЭЛЛИПС, ГИПЕРБОЛА И ПАРАБОЛА КАК ГМТ

Рассмотрение кривых второго порядка на уроке геометрии
тесно связано с курсом алгебры и начал анализа 10 класса,
где исследование и построение графиков функций занимают центральное место. Взгляд на алгебраические уравнения с геометрической точки зрения способствует пониманию математики как единого целого, более глубокому
пониманию идей и методов математической науки.
Материал данного параграфа учебника весьма объемен,
но было бы жаль упустить хотя бы часть из него.
Поэтому можно рекомендовать групповую форму организации его изучения. Класс разбивается на небольшие группы,
и между группами распределяются вопросы данной темы:
1.
2.
3.
4.
5.
6.

Парабола. Определение и уравнение параболы.
Уравнение касательной к параболе.
Оптическое свойство параболы.
Эллипс. Определение и уравнение эллипса.
Гипербола. Определение и уравнение гиперболы.
Уравнение касательной к гиперболе.

Изучив соответствующий пункт учебника, учащиеся
готовят короткие доклады и наглядный материал (в соответствии с технической оснащенностью школы), а затем
выступают и отвечают на вопросы. Уточняющие вопросы,
помогающие восприятию слушателями излагаемого материала, должен продумать учитель.
§ 10. РЕШЕНИЕ ЗАДАЧ С ПОМОЩЬЮ
ГЕОМЕТРИЧЕСКИХ ПРЕОБРАЗОВАНИЙ

В курсе планиметрии геометрические преобразования изучаются в конце девятого класса. На их изучение отводится
так мало часов, что учащиеся в основной своей массе
могут получить лишь общие представления и начальные
умения их выполнения. В связи с этим не представляется
возможным за один (!) урок рассмотреть весь материал,
изложенный в восьмом параграфе учебника. Учителю придется соотносить учебный и задачный материал с уровнем
подготовленности учащихся.

§ 11. Задачи на построение

25

Но какой бы минимальной ни была осведомленность
учеников по этому вопросу, на наш взгляд, нужно продемонстрировать решение задач на построение треугольника
с помощью подобия. Суть метода подобия при решении
задач на построение состоит в том, что строится треугольник, подобный искомому, а затем «восстанавливается»
с использованием коэффициента подобия.
Среди простейших задач — построение треугольника по
двум углам (что позволяет построить подобный треугольник) и одному линейному элементу, а также построение
треугольника и по углу, одному линейному элементу
и отношению двух других линейных элементов.
Например, задача на построение треугольника по двум
углам и медиане, проведенной из вершины одного из
них, решается следующим образом. Строится треугольник
по двум данным углам, подобный искомому. В нем
проводится сходственная данной медиана и продлевается за
ее основание. Затем от вершины угла на луче, содержащем
медиану, откладывается отрезок, равный данному, и через
его конец проводится прямая, параллельная стороне,
к которой проведена медиана. На пересечении этой прямой
с лучами, содержащими стороны подобного треугольника,
получим искомые вершины треугольника.
При решении задачи на построение треугольника ABC
по углу B, отношению противолежащей и прилежащей
сторон b : c, а также разности a − ha между третьей стороной
и проведенной к ней высоте достаточно построить треугольник A1 B1 C1 , подобный искомому, и найти отрезок длины
a1 − h1. Определив коэффициент подобия треугольников
k = (a − ha) : (a1 − h1 ), найдем стороны искомого треугольника и построим его.
§ 11. ЗАДАЧИ НА ПОСТРОЕНИЕ.
НЕРАЗРЕШИМОСТЬ КЛАССИЧЕСКИХ ЗАДАЧ
НА ПОСТРОЕНИЕ С ПОМОЩЬЮ ЦИРКУЛЯ И ЛИНЕЙКИ

Решение классической задачи на построение с помощью
циркуля и линейки состоит из четырех этапов: 1) анализа,
в ходе которого на основе свойств геометрических фигур
намечается план построения, 2) построения, на котором

26

Глава 1. Планиметрия

непосредственно выполняется построение и описывается
пошагово с опорой на основные построения, 3) доказательства, где доказывается, что построенная фигура удовлетворяет условию, и 4) исследования условий разрешимости
задачи и количества ее решений.
Задачи на построение обычно вызывают трудности
у учащихся, и начать их решение следует с овладения
основными задачами на построение, которые являются
шагами в решении любой сложной задачи. На наш взгляд,
именно разбору этих задач следует посвятить урок. Вследствие большой трудоемкости этих задач можно заранее,
в качестве домашнего задания, предложить учащимся список основных построений (еще называемых процедурами),
а на уроке обсудить их решение, обращая внимание также
и на оптимальность выбранного построения (речь может
идти о количестве шагов в построении).
Список основных построений
1. Прямая l, перпендикулярная прямой a, проходящая
через данную точку вне прямой a.
2. Прямая l, перпендикулярная прямой a, проходящая
через данную точку на прямой a.
3. Серединный перпендикуляр к отрезку.
4. Прямая l, параллельная прямой a, проходящая
через данную точку.
5. Прямая l, параллельная прямой a, проходящая на
данном расстоянии от прямой a.
6. Отрезок, равный данному.
7. Середина отрезка.
8. Деление отрезка в данном отношении.
9. Угол, равный данному.
10. Биссектриса угла.
11. Деление дуги пополам.
12. Дуга, вмещающая данный угол, опирающаяся на
данный отрезок.
13. Окружность данного радиуса, касающаяся данной
прямой в данной точке.
14. Окружность данного радиуса, вписанная в данный
угол.

§ 11. Задачи на построение

27

15. Касательная к окружности через точку на окружности.
16. Касательная к окружности через точку вне окружности.
17. Общая внешняя касательная.
18. Общая внутренняя касательная.
19. Геометрическое место середин хорд данной окружности, выходящих из данной точки на окружности.
20. Геометрическое место точек, расстояния от которых
до концов данного отрезка находятся в отношении
m : n (окружность Аполлония).
21. Отрезок x = a + b.
22. Отрезок x = a
√ − b.
23. Отрезок x = √ab.
24. Отрезок x = √a2 + b2 .
25. Отрезок x = a2 − b2 .
26. Отрезок x = ab/c.
27. Прямоугольный треугольник по двум катетам.
28. Прямоугольный треугольник по катету и гипотенузе.
29. Прямоугольный треугольник по катету и острому
углу.
30. Прямоугольный
треугольник
по
гипотенузе
и острому углу.
В заключение учитель может кратко остановиться на
вопросе о неразрешимости классических задач на построение с помощью циркуля и линейки, которые алгебраически
не могут быть сведены к решению алгебраических уравнений степени не выше второй.
Этот вопрос достаточно подробно рассмотрен в учебном
пособии С. Л. Атанасяна и В. Г. Покровского 1) , которое
может оказаться интересным и полезным для учителя
при подготовке к уроку.

1) Атанасян С. Л., Покровский В. Г. Геометрия 1. Учебное пособие
для вузов. — М.: БИНОМ, Лаборатория знаний, 2014.

Глава 2

ПАРАЛЛЕЛЬНЫЕ ПРЯМЫЕ И ПЛОСКОСТИ

До настоящего времени учащиеся изучали планиметрию,
т. е. геометрию на плоскости. Теперь они начинают изучать
геометрию в пространстве — стереометрию — геометрию
настоящего, окружающего нас пространства. Данное обстоятельство учитель может использовать для вовлечения всех
учащихся в процесс размышлений, для активизации их
творческой деятельности. Это тем более важно потому, что
первый месяц ученики будут решать преимущественно
задачи на доказательство. Нужно приучить школьников
к вопросам «Почему?», «Как доказать?», «А что будет,
если. . . ?» в идеале так, чтобы они сами задавались такими
вопросами, привыкли рассматривать с разных сторон
и досконально анализировать встретившуюся ситуацию.
Следует акцентировать внимание учащихся на двух
типах доказываемых утверждений: «существование — НЕ
существование» и «единственность — НЕ единственность».
Существование и не единственность (чего-либо: «Можно
провести. . . », «Можно провести более одной. . . »)
в курсе доказываются конструктивно, т. е. описанием
опирающегося на аксиомы и на доказанные теоремы
или уже решенные задачи воображаемого построения
в пространстве. Единственность или не существование
доказываются чаще всего рассуждением от противного.
§ 12. ВВЕДЕНИЕ В СТЕРЕОМЕТРИЮ.
ОСНОВНЫЕ ТЕОРЕМЫ ИАКСИОМЫ

Стереометрия — это раздел геометрии, в котором изучаются фигуры в пространстве. В стереометрии, так же как
и в планиметрии, свойства геометрических фигур устанавливаются путем доказательства соответствующих теорем.
При этом отправными являются свойства основных гео-

§ 12. Введение в стереометрию

29

метрических фигур, выражаемые аксиомами. Основными
фигурами в пространстве являются точка, прямая и плоскость. Введение нового геометрического образа (плоскости)
заставляет расширить систему аксиом. Поэтому мы вводим
группу аксиом, которая выражает основные свойства
плоскостей в пространстве.
На вводном уроке, повторив с помощью рисунков формулировки аксиом, для контроля усвоения их содержания
можно устно решить (разобрать) следующие три задачи.
Задача 1. Точки A, B, C лежат в каждой из двух различных
плоскостей. Докажите, что эти точки лежат на одной
прямой.
Доказательство. Утверждение является непосредственным следствием аксиомы 2.
Задача 2. Точки A, B, C и D не лежат в одной плоскости.
Докажите, что прямые AB и CD не пересекаются.
Указание. Рассуждать от противного. Если бы прямые
AB и CD пересекались, то по аксиоме 1 через них можно
было бы провести плоскость и в ней лежали бы все четыре
данные точки, что противоречит условию задачи.
Формулировки аксиом полезно сопроводить рисунками.
Нужно объяснить, что аксиомы выражают интуитивно
ясные свойства плоскостей, их связи с двумя другими
основными фигурами стереометрии — с прямыми и точками. Рассмотренные аксиомы относятся только к плоскостям, и к ним необходимо добавить аксиомы о прямых,
аналогичные соответствующим планиметрическим аксиомам.
Задача 3. Докажите, что если четыре данные точки не
лежат в одной плоскости, то никакие три из них не лежат
на одной прямой.
Решение. Допустим, что какие-нибудь три точки лежат
на одной прямой. Проведем через эту прямую и четвертую
точку плоскость (следствие 1). В этой плоскости лежат все
четыре точки. А это противоречит условию задачи. Значит,
никакие три точки не могут лежать на одной прямой.

30

Глава 2. Параллельные прямые и плоскости

Вопросы для закрепления
1. Что изучает стереометрия?
2. Каковы основные (простейшие) фигуры в пространстве?
3. Как устанавливаются свойства фигур (теоремы) в геометрии? (Ответ: путем доказательства, т. е. логического
рассуждения, опирающегося на аксиомы и уже доказанные
теоремы).
4. Сформулируйте стереометрические аксиомы.
5. Сформулируйте следствие 1 о прямой и точке.
6. Что можно утверждать в ситуации этой теоремы, если
точка лежит на прямой? (Иначе говоря, продолжите:
«Через прямую и лежащую на ней точку. . . ».)
7. Какие аксиомы используются при доказательстве следствия 1?
Задачи для закрепления
1. Можно ли через точку пересечения двух данных прямых
провести третью прямую, не лежащую с ними в одной
плоскости? Объясните ответ.
О т в е т: можно. Действительно, пусть прямые a и b
пересекаются в точке C и лежат в плоскости γ (следствие 2).
Возьмем точку D вне плоскости γ (аксиома 1) и рассмотрим
прямую CD. Эта прямая и удовлетворяет условию задачи,
поскольку лежащая на ней точка D не принадлежит плоскости γ, а плоскость, содержащая прямые a и b, единственна
(следствие 2) — это γ (последнее замечание — ссылка на
единственность γ — существенно, однако учащиеся могут
пропустить эту тонкость).
2. Даны три различные попарно пересекающиеся плоскости. Докажите, что если две из прямых пересечения этих
плоскостей пересекаются, то третья прямая проходит через
точку их пересечения.
Доказательство. Если плоскости α и β пересекаются по
прямой a, плоскости β и γ — по прямой b, причем прямые a
и b пересекаются в точке C, то эта точка принадлежит
всем трем плоскостям α, β, γ, а значит, и третьей прямой c

§ 12. Введение в стереометрию

31

пересечения плоскостей α и γ. Вопрос при разборе этой
задачи: какие аксиомы используются при ее решении?
О т в е т: аксиома 2.
3. Докажите, что все прямые, пересекающие данную
прямую и проходящие через данную точку вне прямой,
лежат в одной плоскости.
Доказательство. Данные прямая a и точка A определяют
плоскость α. Если прямая b проходит через точку A
и пересекает прямую a в точке B, то прямая b имеет
с плоскостью α две различные общие точки (A и B)
и поэтому лежит в указанной плоскости α (аксиома 2).
4. Можно ли через три точки, лежащие на одной прямой,
провести две различные плоскости?
О т в е т: можно.
5. Даны четыре точки. Известно, что прямая, проходящая
через любые две из этих точек, не пересекается с прямой,
проходящей через другие две точки. Докажите, что данные
четыре точки не лежат в одной плоскости.
Доказательство. Если бы такие четыре точки A, B,
C, D лежали в одной плоскости, то прямые AB и CD,
AC и BD были бы параллельными, так что точки A, B,
C, D являлись бы вершинами параллелограмма ABDC.
Однако диагонали AD и BC этого параллелограмма должны
пересекаться, что противоречит условию задачи.
6. Даны четыре точки, не лежащие в одной плоскости.
Сколько можно провести различных плоскостей, проходящих через три из этих четырех точек? Объясните ответ.
О т в е т: в точности четыре различные плоскости. Действительно, если точки A, B, C, D не лежат в одной
плоскости, то все они и никакие три из них не лежат на
одной прямой. Поэтому каждая из четырех возможных
троек точек (A, B, C), (A, B, D), (A, C, D) и (B,
C, D) определяет единственную плоскость (аксиома 1).
Эти четыре плоскости все различные, что доказывается
рассуждением от противного с использованием условия
о том, что A, B, C, D не лежат в одной плоскости.

32

Глава 2. Параллельные прямые и плоскости

Упражнения 1)
1(12.1) Приведите примеры пространственных фигур,
плоских фигур, неплоских фигур. Какое минимальное
число точек может содержать неплоская фигура?
О т в е т: неплоская фигура содержит более трех точек.
2(12.3) Какое минимальное количество спичек следует
взять, чтобы сложить из них шесть квадратов со стороной,
равной длине спички?
О т в е т: 12 спичек, если мы соберем куб из этих спичек.
3(12.5) Предложите практический способ непосредственного (без вычислений) измерения диагонали кирпича.
Кирпич ломать не разрешается!
Решение. Как известно, кирпичи чаще всего встречаются
на стройках в больших количествах. Чтобы измерить
диагональ, нам хватит всего лишь трех (более или менее
одинаковых) — см. рис. 2.1.

d

Рис. 2.1.

4(12.9) (шутка). Три мухи одновременно взлетели с одной
лысины. В каком случае они окажутся в одной плоскости?
О т в е т: всегда.
5(12.10) Верно ли, что можно провести плоскость через
любые: а) две точки, б) три точки, в) четыре точки?
О т в е т: а) да; б) да; в) нет.
6(12.13) Верно ли, что через три попарно пересекающиеся
прямые проходит единственная плоскость?
О т в е т: да.
1) В

скобках указаны номера соответствующих задач в учебнике.

§ 13. Взаимное расположение двух прямых в пространстве

33

7(12.16) Докажите, что если четыре данные точки не лежат
в одной плоскости, то никакие три из них не лежат на
одной прямой.
Доказательство. Пусть три точки лежат на прямой l,
тогда через эту прямую и четвертую точку можно провести
плоскость, что противоречит условию.
8(12.17) Пусть прямая l является линией пересечения
плоскостей α и β, прямая a лежит в плоскости α. Докажите,
что прямые a и l пересекаются в том и только том случае,
когда прямая a пересекает плоскость β.
Указание. Любая точка прямой l лежит в плоскости β.
9(12.21) Совпадает ли плоскость, в которой лежит трапеция, с плоскостью, проходящей через: а) середины
ее боковых сторон; б) середины всех сторон трапеции;
в) диагонали трапеции; г) вписанную окружность, когда
эта окружность существует?
О т в е т: а) нет; б) да; в) да; г) да.
10(12.22) Могут ли три различные плоскости пересекаться
только: а) в одной точке; б) в двух точках; в) по двум
пересекающимся прямым?
О т в е т: а) да; б) нет; в) да; г) нет.
11(12.23) Докажите, что существуют две прямые в пространстве, не лежащие в одной плоскости.
О т в е т: рассмотрим точки A, B, C, D, не лежащие в одной
плоскости (такие точки существуют в силу аксиомы 1).
Тогда прямые AB и CD не лежат в одной плоскости, так
как в противном случае точки A, B, C, D лежали бы
в одной плоскости.
§ 13. ВЗАИМНОЕ РАСПОЛОЖЕНИЕ ДВУХ ПРЯМЫХ
В ПРОСТРАНСТВЕ

Тема взаимного расположения прямых и плоскостей в пространстве — узловая по своему значению для развития
пространственных представлений учащихся. Ее изложение
выстраивается вокруг основного, «стержневого» вопроса:
каким может быть взаимное расположение прямых

34

Глава 2. Параллельные прямые и плоскости

и плоскостей в пространстве? Сначала перечисляются
все логические возможности. Например, две различные
прямые либо лежат в одной плоскости, либо не лежат —
тогда они называются скрещивающимися. Если же две
различные прямые лежат в одной плоскости, то возможны
следующие два случая: прямые либо пересекаются (имеют
общие точки), либо не пересекаются; во втором из этих
случаев (когда прямые лежат в одной плоскости и не
пересекаются, т. е. не имеют общих точек) прямые называются параллельными, а в первом, согласно аксиоме, они
имеют только одну общую точку, и тогда они называются
пересекающимися — в геометрическом смысле.
Затем анализируется вопрос о том, осуществляются
ли эти возможности. Основное отношение в этой теме —
параллельность, так что вопрос сводится к существованию
параллельных прямых и плоскостей (a k b, a k β, α k β).
Наконец, обсуждается и так или иначе решается вопрос
о единственности параллельных (в трех указанных
ситуациях).
Во время изучения данной темы рекомендуется продолжить планомерное повторение самого необходимого
планиметрического материала. Хотя в теме по-прежнему
превалируют задачи на доказательство (напомним, что во
многих случаях они решаются конструктивно!), велика
роль данной темы и в обучении решению задач. В теме
впервые появляются такие фундаментальные для решения стереометрических задач идеи, как сведение задачи
к планиметрическим рассмотрениям, привлечение (при
возможности) планиметрического аналога стереометрической задачи.
Конечно, этот фрагмент урока нужно сопровождать
показом параллельности, пересечения, скрещивания прямых хотя бы на паре указок или на модели параллелепипеда. Для закрепления определения скрещивающихся
прямых можно устно ответить на следующий вопрос: могут
ли прямые AC и BD быть скрещивающимися, если прямые
AB и CD пересекаются?
О т в е т: не могут, так как пересекающиеся прямые AB
и CD, а вместе с ними и точки A, B, C, D, а вместе
с ними и прямые AC и BD лежат в одной плоскости.

§ 13. Взаимное расположение двух прямых в пространстве

35

Следующие две задачи можно предложить учащимся
решить самостоятельно с последующим разбором или же
совместно обсудить.
Задача 1. Докажите, что все прямые, пересекающие две
данные параллельные прямые, лежат в одной плоскости.
Учащиеся без труда справляются с решением, требующим применения только определения параллельности.
Доказательство. Так как данные прямые a и b параллельны, через них можно провести плоскость. Обозначим
ее α. Прямая c, пересекающая данные параллельные
прямые, имеет с плоскостью α две общие точки — точки
пересечения с данными прямыми. По аксиоме 2, эта
прямая лежит в плоскости α. Итак, все прямые, пересекающие две данные параллельные прямые, лежат в одной
плоскости — плоскости α.
Задача 2. Прямые a и b пересекаются. Докажите, что
все прямые, параллельные прямой b и пересекающие
прямую a, лежат в одной плоскости.
Нужно предложить сначала как-то определить эту плоскость. К настоящему времени нам уже известны четыре
способа задания плоскости: двумя пересекающимися прямыми, прямой и точкой, тремя точками и парой параллельных прямых (определение параллельности). В этой
задаче учащиеся могут предложить два способа: нужная
плоскость проходит через прямые a и b или же через
прямую b и одну из параллельных b прямых c. Первый
способ оказывается экономнее (и к тому же подсказывается
условием задачи).
Доказательство. Пусть c — прямая, параллельная прямой b и пересекающая прямую a (рис. 2.2). Проведем
через прямые a и b плоскость α. Проведем через точку C
пересечения прямых a и c в плоскости α прямую c′ , параллельную b. По теореме 15, через точку C можно провести
только одну прямую, параллельную b. Отсюда следует,
что прямая c совпадает с прямой c′ , а значит, лежит
в плоскости α. Итак, любая прямая c, параллельная b

36

Глава 2. Параллельные прямые и плоскости

и пересекающая прямую a, лежит в плоскости
и требовалось доказать.

α, что

a

α

A

b

β c

B

Рис. 2.2.

На задачу 2 следует обратить особое внимание учащихся, ибо ее утверждение будет многократно использоваться в дальнейшем (уже в следующих задачах и,
что весьма существенно, при изучении параллельного
проектирования).
Вопросы для закрепления
1. Дайте определение параллельных прямых в пространстве.
2. Верно ли, что две прямые параллельны, если они не
пересекаются?
3. Какие прямые называют скрещивающимися? Приведите
примеры.
4. Сформулируйте и докажите признак скрещивающихся
прямых.
5. Могут ли скрещиваться две прямые, параллельные
третьей прямой?
6. Каким может быть взаимное расположение двух различных прямых в пространстве? (Описать, обосновать
и показать на моделях.)
7. В каком случае прямые в пространстве называются
параллельными? скрещивающимися?
8. Сформулируйте теорему о параллельных прямых в пространстве. Какие аксиомы и теоремы используются при
ее доказательстве?

§ 13. Взаимное расположение двух прямых в пространстве

37

9. Сформулируйте аксиому параллельных и теорему
о параллельных на плоскости. В чем разница между ними?
10. Как через данную точку в пространстве построить
прямую, параллельную данной прямой? (Опишите воображаемое пространственное построение.)
11. Как можно доказывать параллельность прямых в пространстве? (Ответ: исходя из определения или пользуясь
признаком — теоремой 17.)
12. Сформулируйте признак параллельности прямых
в пространстве.
13. Что такое пространственный четырехугольник? Что
можно сказать о серединах его сторон?
Задачи для закрепления
1. Докажите, что если плоскость пересекает одну из двух
параллельных прямых, то она пересекает и другую.
Доказательство. Пусть прямые a и b параллельны
и плоскость α пересекает прямую a в точке A. Прямые a и b
лежат в некоторой плоскости β, отличной от плоскости α
(прямая a лежит в плоскости β, но не лежит в α), но
имеющей с плоскостью α общую точку A. По аксиоме 2,
плоскости α и β пересекаются по некоторой прямой c
(рис. 2.3). Задача свелась к планиметрической: в плоскости
β прямые a и b параллельны, прямая c пересекает прямую
a. Но тогда прямая c пересекает и прямую b, иначе через
точку A проходили бы две параллельные к прямой b. Итак,
прямая c пересекает прямую b, поэтому и содержащая

α
A
W
a
β

C

b

Рис. 2.3.

38

Глава 2. Параллельные прямые и плоскости

прямую c плоскость α пересекает прямую b. Требуемое
доказано.
2. Через конец A отрезка AB проведена плоскость. Через
конец B и точку C этого отрезка проведены параллельные прямые, пересекающие плоскость в точках B1
и C1 . Найдите длину отрезка BB1, если: 1) CC1 = 15 см,
AC : BC = 2 : 3; 2) AC = a, BC = b, CC1 = c.
Решение.
Отыскиваем
коэффициент
подобия
треугольников ACC1 и ABB1: AC : BC = 2 : 3, поэтому
k = AB : AC = 5 : 2 = 2,5 (AC составляет 2 части, BC — 3
части, поэтому AB — 5 таких частей). Далее вычисляем:
BB1 = CC1 · k = 15 · 2,5 = 37,5 (см). Записываем пропорцию
BB1 : AB = CC1 : AC, т. е. BB1 : (a + b) = c : a, откуда

b
BB1 = c 1 + .
a

В планиметрии существуют три признака параллельности прямых на плоскости — по равенству между собой
внутренних накрест лежащих углов между прямыми
и секущей, по равенству суммы внутренних односторонних
углов. Третий признак представляет собой теорему: «Две
прямые, параллельные третьей, параллельны друг другу»,
которая (в планиметрии) легко доказывается рассуждением
от противного, апеллирующим к аксиоме параллельных.
Первые два признака параллельности не имеют аналогов
для прямых в пространстве. Даже если как-либо определить внутренние накрест лежащие или односторонние
углы между двумя прямыми и секущей в пространстве (из
соответствующих условий (равенств) отнюдь не следует,
что прямые будут лежать в одной плоскости — это можно
показать на моделях). Последний признак оказывается
справедливым и в стереометрии.
Для закрепления теоремы 17 — признака параллельности прямых в пространстве — можно решить следующие
четыре задачи.
3. Прямые a и b не лежат в одной плоскости. Можно ли
провести прямую c, параллельную прямым a и b?

§ 13. Взаимное расположение двух прямых в пространстве

39

О т в е т: нельзя, так как в противном случае a и b оказались
бы параллельными (по теореме 17) и, следовательно,
лежали бы в одной плоскости, что противоречит условию.
4. Точки A, B, C, D не лежат в одной плоскости. Докажите,
что прямая, проходящая через середины отрезков AB и BC,
параллельна прямой, проходящей через середины отрезков
AD и CD.
Решение сразу видно из чертежа (рис. 2.4), если на
нем провести прямую AC : достаточно воспользоваться
свойством средней линии треугольника и теоремой 17
(прямые k и l параллельны AC, поэтому параллельны
между собой). Однако возникает вопрос: почему можно
пользоваться этой теоремой? Вдруг наши прямые k и l
совпадают? Конечно, нет, иначе совпадают и плоскости
ABC и ADC, т. е. точки A, B, C, D будут лежать в одной
плоскости. Другой вопрос: справедливо ли утверждение
задачи, если точки A, B, C, D лежат в одной плоскости?
Ответ: «почти справедливо», за исключением того случая,
когда прямые AC и BD параллельны (тогда прямые k и l
совпадут на самом деле).
D

k

B

C

l
A

Рис. 2.4.

Фигура, образованная отрезками AB, BC, CB и DA
с концами A, B, C, D, не лежащими в одной плоскости,
называется пространственным четырехугольником. Он
фигурирует в первой из следующих двух задач, которые
целесообразно дать для самостоятельного решения.
5. Докажите, что середины сторон пространственного четырехугольника являются вершинами параллелограмма.
Доказательство. Пусть ABCD — данный пространственный четырехугольник (вершины четырехугольника не

40

Глава 2. Параллельные прямые и плоскости

лежат в одной плоскости) (рис. 2.5). Пусть A1 , B1 , C1 ,
D1 — середины его сторон. Тогда A1 B1 — средняя линия
треугольника ABC, параллельная стороне AC, C1 D1 —
средняя линия треугольника ACD, тоже параллельная
стороне AC. По теореме 17, прямые A1 B1 и C1 D1 параллельны, а значит, лежат в одной плоскости. Точно так же
доказывается параллельность прямых A1 D1 и B1 C1 . Итак,
четырехугольник A1 B1 C1 D1 лежит в одной плоскости и его
противолежащие стороны параллельны. Следовательно,
он — параллелограмм.
D
C1

D1

C

A
A1

B1
B

Рис. 2.5.

Это решение можно чуть сократить, воспользовавшись
теоремой о средней линии треугольника в полном объеме
(отрезки A1 B1 и C1 D1 параллельны и равны половине
отрезка AC) и вторым из указанных выше признаков
параллелограмма.
6. Параллелограммы ABCD и ABC1D1 лежат в разных
плоскостях. Докажите, что четырехугольник CDD1C1 —
тоже параллелограмм.
В обеих задачах речь идет о параллелограммах, поэтому
стоит предварительно вспомнить с учащимися, что такое
параллелограмм. Как можно доказать, что четырехугольник — параллелограмм? При этом повторении необходимо
отделить определение параллелограмма от его признаков,
которые нужно напомнить. Целесообразно предложить
учащимся обдумать планиметрические аналоги задач 3 и 4,
задав вопрос: верны ли утверждения задач в «плоском
случае» — когда четырехугольник из задачи 3 плоский?
когда параллелограммы ABCD и ABC1 D1 из задачи 5
лежат в одной плоскости? Рассматривая «плоский случай»,
учащиеся без труда найдут решение и в пространственном
случае. (Рассмотрение плоских аналогов пространственных

§ 13. Взаимное расположение двух прямых в пространстве

41

задач, теорем, конструкций, если такие аналоги есть, —
важная методологическая идея стереометрии.)
Доказательство. Противолежащие стороны параллелограммов параллельны и равны, поэтому CD = AB = C1D1 ,
а прямые CD и C1 D1 параллельны прямой AB и, следовательно, параллельны между собой (теорема 17 применима,
так как в пространственном случае прямые CD и C1 D1
не могут совпасть). Таким образом, CDD1 C1 — плоский
четырехугольник, к которому осталось применить второй
признак параллелограмма.
7. Даны параллелограмм ABCD и не пересекающая его
плоскость. Через вершины параллелограмма проведены
параллельные прямые, пересекающие данную плоскость
в точках A1 , B1 , C1 , D1 . Найдите длину отрезка DD1 ,
если: 1) AA1 = 2 м, BB1 = 3 = м, CC1 = 8 м; 2) AA1 = 4 м,
BB1 = 3 м, CC1 = 1 м; 3) AA1 = a, BB1 = b, CC1 = c.
Решение. 3) Пусть M — точка пересечения диагоналей
параллелограмма ABCD. Проведем через M прямую,
параллельную уже проведенным — AA1 и т. д. Эта прямая
тоже пересекает данную плоскость в какой-то точке M1 .
Обозначив искомую длину DD1 через x, длину MM1 через
m, применим к середине M отрезков AC и BD формулу
m = (a + c)/2 = (b + x)/2, откуда находим x = a + c − b.
8. Даны четыре точки A, B, C, D, не лежащие в одной
плоскости. Докажите, что прямые, соединяющие середины
отрезков AB и CD, AC и BD, AD и BC, пересекаются
в одной точке.
Доказательство. Первый и третий из отрезков с концами
в серединах указанных в условии пар отрезков суть диагонали параллелограмма с вершинами в серединах сторон
пространственного четырехугольника ABCD. Поэтому первый и третий отрезки пересекаются и точкой пересечения O
делятся пополам. Аналогично первый и второй отрезки
являются диагоналями параллелограмма с вершинами
в серединах сторон пространственного четырехугольника
ABDC, поэтому второй отрезок пересекает первый в той же

42

Глава 2. Параллельные прямые и плоскости

самой точке O (середине первого). Итак, все три отрезка,
а с ними и рассматриваемые в условии задачи три прямые
пересекаются в точке O. Требуемое доказано.
Следующие задачи нужно разобрать особо тщательно,
ибо это первые задачи, в которых применяется фундаментальный прием решения стереометрических задач:
сведение их к планиметрическим.
9. Через концы отрезка AB и его середину M проведены
параллельные прямые, пересекающие некоторую плоскость
в точках A1 , B1 и M1 . Найдите длину отрезка MM1 , если
отрезок AB не пересекает плоскость и если AA1 = 5 м,
BB1 = 7 м.
Решение. По предыдущей задаче, прямые AA1 , BB1
и MM1 лежат в одной плоскости β, поэтому точки A1 , B1
и M1 лежат на прямой A1 B1 пересечения плоскости β с данной плоскостью α. Следовательно, достаточно рассмотреть
картинку, получающуюся в плоскости β. (В этой задаче
и стереометрический чертеж достаточно «прозрачен», но
все-таки лучше сразу приучать школьников к отдельному изображению плоского сечения — в данном случае
рисунка.) По теореме Фалеса, M — середина отрезка A1 B1 ,
MM1 — средняя линия трапеции AA1 B1 B, и, по теореме
о средней линии, MM1 = x = 0,5(a + b) = 0,5(5 + 7) = 6 (м).
Стоит показать учащимся еще два способа решения:
провести прямую AB1 и дважды использовать теорему о средней линии треугольника (для треугольников
ABB1 и AB1 A1 ) или провести прямую AK параллельно
A1 B1 и использовать равенство сторон параллелограммов
AA1 = K1 M1 = KB1 и теорему о средней линии треугольника
ABK. Обязательно следует спросить, как (где) в решении
использовано условие, что отрезок AB не пересекает
плоскость. Существенно ли оно?
10. Через конец A отрезка AB проведена плоскость.
Через конец B и точку C этого отрезка проведены
параллельные прямые, пересекающие плоскость в точках
B1 и C1 . Найдите длину отрезка BB1, если: 1) CC1 = 8,1 см,
AB : AC = 11 : 9; 2) AB = 6 см, AC : CC1 = 2 : 5.

§ 13. Взаимное расположение двух прямых в пространстве

43

Решение. Идея решения и чертеж в обоих случаях
одинаковы: плоскость β, проходящая через параллельные
прямые BB1 и CC1 , содержит прямую AB и пересекает
данную плоскость по прямой AB1 так, что в плоскости β
получаются два подобных (даже гомотетичных) треугольника ACC1 и ABB1 (угол A у них общий, а равенство
углов C и B следует из параллельности прямых CC1
и BB1 ). Коэффициент подобия равен k = AB/AC = 11/9,
поэтому BB1 = CC1 · k = 9,9 (см). Далее можно записать
пропорцию: BB1 : AB = CC1 : AC, т. е. BB1 = 6 = 5 : 2, откуда
BB1 = 6 · 5 : 2 = 15 (см).
Упражнения
1(13.1) На примере тетраэдра ABCD (сделайте рисунок
самостоятельно) укажите пары скрещивающихся прямых.
2(13.2) Верен ли признак транзитивности для скрещивающихся прямых, т. е. верно ли, что прямые a и b
скрещиваются, если попарно скрещиваются прямые a, c
и b, c?
О т в е т: нет.
3(13.3) Верно ли, что если прямая c скрещивается с одной
из двух данных параллельных прямых, то она скрещивается и с другой?
О т в е т: пусть a k b и c скрещивается с прямой a, в этом
случае возможны два варианта: а) c скрещивается с прямой b, б) c пересекается с прямой b.
4(13.4) Какое множество точек в пространстве заполняют
все прямые, пересекающие сразу две данные параллельные
прямые?
О т в е т: плоскость, в которой лежат эти две параллельные
прямые.
5(13.5) По каждой из двух скрещивающихся нитей ползут
два муравья. Могут ли все четыре муравья оказаться
в одной плоскости?
О т в е т: нет, не могут. Если бы муравьи оказались в одной
плоскости в некоторый момент, то прямые, по которым
ползли муравьи, тоже бы оказались в одной плоскости,
что противоречит условию.

44

Глава 2. Параллельные прямые и плоскости

6(13.6) Можно ли провести плоскость через всякую: а) двузвенную, б) трехзвенную ломаную линию?
О т в е т: а) можно всегда провести плоскость через три
точки по аксиоме 1, взяв по одной точке на каждом звене
и угловую точку ломаной, тогда звенья ломаной будут
принадлежать этой плоскости по аксиоме 2.
7(13.7) Пусть через каждую из двух скрещивающихся
прямых a и b проведена плоскость. Тогда если эти
плоскости пересекаются по прямой c, то может ли прямая
c быть параллельной: а) прямой a; б) прямым a и b?
О т в е т: а) да; б) нет.
8(13.8) Четыре точки не лежат в одной плоскости. Могут
ли какие-нибудь три из них лежать на одной прямой?
О т в е т: не могут, так как, по следствию 1, через прямую,
содержащую три точки и четвертую точку, не лежащую
на этой прямой, можно провести единственную плоскость,
что противоречит условию.
9(13.9) Могут ли быть параллельны прямые, полученные
при пересечении двух пересекающихся плоскостей третьей
плоскостью?
О т в е т: да, например, ребра треугольной призмы могут
быть параллельны друг другу.
10(13.10) Прямая a пересекает плоскость α. Может ли
в плоскости лежать прямая, параллельная прямой a?
О т в е т: нет. Пусть прямая b принадлежит плоскости α,
b k a, тогда, по следствию 3, через эти прямые можно провести единственную плоскость, что противоречит условию.
11(13.11) Докажите, что все прямые, пересекающие две
данные пересекающиеся прямые и не проходящие через
точку их пересечения, лежат в одной плоскости.
Доказательство. Пусть данные прямые a и b пересекаются в точке C (рис. 2.6). Возьмем на прямых a и b
точки A и B соответственно, отличные от точки C. По
аксиоме 1, через точки A, B, C проходит единственная
плоскость α, причем, по аксиоме 2, прямые a и b лежат
в плоскости α. Пусть произвольная прямая l пересекает
прямые a и b соответственно в точках A1 и B1 . Точки A1
и B1 принадлежат плоскости α, так как в этой плоскости

§ 13. Взаимное расположение двух прямых в пространстве

45

лежат прямые a и b, и различны (в противном случае точки
A и B совпадают с точкой C, но прямая l не проходит через
точку C по условию), поэтому, по аксиоме 2, прямая l
лежит в плоскости α.
l
A

a
A1

C
B

α
B1

b

Рис. 2.6.

12(13.12) Три точки лежат в каждой из двух различных
плоскостей. Докажите, что эти три точки лежат на одной
прямой.
Доказательство. Данные три точки принадлежат линии
пересечения двух плоскостей, которая является прямой по
аксиоме 3.
13(13.14) Столяр с помощью двух нитей проверяет, будет
ли устойчиво стоять на полу стол, имеющий четыре ножки.
Как нужно натянуть нити?
Решение. Обозначим концы ножек стола через A, B, C,
D. Тогда необходимо натянуть нити по диагоналям AC
и BD четырехугольника ABCD (рис. 2.7). Если эти нити
пересекаются, то точки A, B, C, D лежат в одной плоскости,
и в этом случае стол будет стоять на полу устойчиво.
14(13.15) Даны точки A, B, C, не лежащие D
на одной прямой, и плоскость α, не
содержащая ни одну из этих точек. Пусть
O
A1 , B1 , C1 — точки пересечения прямых
BC, AC, AB с плоскостью соответственно. A
Докажите, что точки A1 , B1 , C1 лежат на
Рис. 2.7.
одной прямой.

C

B

Доказательство. Обозначим через α1 плоскость, проходящую через точки A, B, C (рис. 2.8). Тогда прямые AC, ВС

46

Глава 2. Параллельные прямые и плоскости
A
B
C

α1
C1

α A1

B1

Рис. 2.8.

и AB лежат в плоскости α1 и поэтому точки пересечения
этих прямых с плоскостью α лежат на прямой пересечения
плоскостей α и α1 .
15(13.16) Как могут быть расположены прямая a и плоскость α, если данная прямая и некоторая прямая, лежащая
в плоскости α, скрещиваются?
О т в е т : прямая a должна пересекать плоскость α или быть
параллельной плоскости α.
16(13.17) Докажите, что лежат в одной плоскости: а) все
параллельные прямые, пересекающие данную прямую;
б) все прямые, пересекающие данную окружность в двух
точках.
Доказательство. б) Все эти прямые лежат в плоскости
окружности.
17(13.18) Даны три попарно пересекающиеся плоскости.
Докажите, что если две из прямых пересечения этих
плоскостей пересекаются, то третья прямая проходит через
точку их пересечения.
Доказательство. Пусть плоскости α и β пересекаются
по прямой l3 , β и γ — по прямой l1 , γ и α — по прямой l2 .
Если прямые l1 и l2 пересекаются в точке A, то эта точка
является точкой пересечения плоскостей α, β и γ, а значит,
и плоскостей α и β, т. е. точка A лежит на l3 .
18(13.19) На рис. 2.9 три попарно пересекающиеся прямые
a, b, c пересекают плоскость в точках A, B, C соответственно. Правильно ли выполнен рисунок?
О т в е т:
рисунок выполнен неправильно, так как если
прямые a, b, c попарно пересекаются, то они лежат в одной

§ 13. Взаимное расположение двух прямых в пространстве

47

a

c
b
C
α

A

B

Рис. 2.9.

плоскости, и поэтому точки A, B, C должны лежать на
одной прямой.
19(13.20) Даны две непересекающиеся плоскости. Докажите, что прямая, пересекающая одну из двух плоскостей,
пересекает и другую.
Доказательство. Обозначим данную прямую через a,
данные плоскости — через α и β, а точку пересечения
прямой a с плоскостью α — через A (рис. 2.10). Возьмем
a
γ
α

A
b

β

B
c

Рис. 2.10.

в плоскости β произвольную точку B. Если эта точка лежит
на прямой a, то это означает, что прямая a пересекает
плоскость β, поэтому предположим, что точка B не лежит
на прямой a, и проведем через прямую a и точку B
плоскость γ. Тогда прямые b и c пересечения плоскости
с плоскостями a и β не пересекаются (в противном
случае пересекались бы плоскости a и β). Итак, прямые
b и c лежат в одной плоскости γ и не пересекаются,
т. е. прямые b и c параллельны. Предположим теперь, что
прямая a не пересекает плоскость β, тогда она не пересекает
и прямую c, откуда следует, что a k c. Таким образом, мы

48

Глава 2. Параллельные прямые и плоскости

получили, что в плоскости γ через точку A проходят две
различные прямые a и b (прямые a и b не могут совпадать,
так как прямая a пересекает плоскость α, прямая b
лежит в ней), параллельные прямой c, лежащей в той
же плоскости, что противоречит аксиоме о параллельных
прямых. Итак, предположение о том, что прямая не
пересекает плоскость β, приводит к противоречию, поэтому
прямая a и плоскость β пересекаются.
§ 14. ПАРАЛЛЕЛЬНОСТЬ ПРЯМОЙ И ПЛОСКОСТИ

Перейдем к рассмотрению взаимного расположения прямой и плоскости в пространстве. Для прямой a и плоскости α в принципе существуют три возможности: a лежит
в α (или α проходит через a), a пересекается с α (ровно
в одной точке) и, наконец, a не пересекается с α (т. е. α
и a не имеют общих точек). Мы уже видели, что первые
две возможности осуществимы. Осуществима ли третья
возможность? Как для данной плоскости α провести какуюнибудь не пересекающую ее прямую a?
Учащиеся (возможно, с подсказкой) дают соответствующее построение: нужно в плоскости α провести какуюнибудь прямую a1 , взять вне плоскости α точку A
и провести через точку A прямую a параллельно прямой
a1 , тогда прямая a не будет пересекать плоскость α.
Доказательство: прямые a и a1 лежат в одной плоскости β,
причем плоскости α и β различны и пересекаются по
прямой a1 ; если бы прямая a пересекала плоскость α
в какой-то точке B, то B обязательно лежала бы на
прямой a1 (ибо точка B принадлежала бы и плоскости α,
и плоскости β). Значит, прямые a и a1 пересекаются, что
противоречит построению (a k a1).
Далее учащимся можно предложить следующие задачи.
Задача 1. Через данную точку проведите прямую, параллельную каждой из двух данных пересекающихся плоскостей.
Решение. Для этого достаточно через данную точку провести прямую, параллельную прямой пересечения данных
плоскостей, и сослаться на теорему 18.

§ 14. Параллельность прямой и плоскости

49

Задача 2. Докажите, что через любую из двух скрещивающихся прямых можно провести плоскость, параллельную
другой прямой.
Решение. Пусть a и b — две скрещивающиеся прямые.
Возьмем на прямой a любую точку и проведем через
нее прямую b′ , параллельную прямой b. Проведем через
прямые a и b′ плоскость α. По теореме 18, она будет
параллельна прямой b.
Вопрос к решению: почему применима теорема 18?
Иначе, почему прямая b не будет лежать в проведенной
плоскости α?
О т в е т: потому что прямые a и b скрещиваются.
Задача 3. Даны четыре точки А, В, С, D, не лежащие
в одной плоскости. Докажите, что любая плоскость,
параллельная прямым AB и CD, пересекает прямые AC,
AD, BD и BC в вершинах параллелограмма.
Решение. Пусть некоторая плоскость α параллельна прямым AB и CD. По теореме 18, из параллельности прямой
AB и плоскости α следует, что плоскости трех точек
ABC и ABD пересекают плоскость α по прямым a и b,
параллельным прямой AB. Из теоремы 17 вытекает, что
прямые a и b параллельны между собой. Аналогично
из параллельности прямой CD и плоскости α следует,
что и плоскости ACD и BCD пересекают плоскость α по
параллельным прямым c и d. Каждая из точек пересечения
плоскости α с прямыми АС, AD, BD, ВС лежит на
плоскости α в пересечении каких-то двух из непараллельных прямых а, b, с, d. Например, точка пересечения
прямой AC с плоскостью α принадлежит плоскостям ABC
и ACD, поэтому она получается в пересечении прямых b
и c, где прямые b и c — линии пересечения плоскости α
с плоскостями ABC и ACD соответственно. Из попарной
параллельности прямых a и b, c и d и следует утверждение
задачи.
Вопрос к решению: почему мы имеем право применять — например, к прямым a и b — теорему 17? Иначе
говоря, не могут ли прямые a и b оказаться совпадающими? Ответ: нет, так как если прямая a совпадает

50

Глава 2. Параллельные прямые и плоскости

с прямой b, то обе эти прямые совпадают с прямой
AB, ибо плоскости ABC и ABD имеют единственную
общую прямую AB; однако тогда плоскость α проходит
через прямую AB, что противоречит условию задачи.
Более сложный вопрос: а почему вообще плоскость α,
параллельная прямым AB и CD, должна пересекать
прямые AC, AD, BD, ВС или же плоскости ABC, ABD,
ACD, BCD? Ответ можно дать, используя тот факт, что
все рассматриваемые в задаче плоскости α параллельны
между собой и параллельны плоскости параллелограмма
с вершинами в серединах сторон пространственного четырехугольника ACBD.
Вопросы для закрепления
1. Каким может быть взаимное расположение прямой
и плоскости в пространстве? (Описать, обосновать и показать.)
2. В каком случае прямая и плоскость называются параллельными? пересекающимися?
3. Как через данную точку провести какую-нибудь прямую,
параллельную данной плоскости?
4. Сформулируйте признак параллельности прямой и плоскости (теорему 18). (В формулировке важно подчеркнуть:
прямая, не лежащая в плоскости.)
5. Верна ли теорема, обратная сформулированной? Что
она гласит?
6. Что можно сказать о прямой пересечения плоскостей α
и β, если плоскость β проходит через прямую a, параллельную плоскости α?
7. Перечислите все возможные случаи расположения прямой и плоскости.
8. Прямая a скрещивается с некоторой прямой b, лежащей
в плоскости α. Может ли прямая a быть параллельной
плоскости α?
9. Следует ли из того, что прямая параллельна плоскости, то, что она параллельна каждой прямой этой
плоскости?
10. Сколько прямых, параллельных данной плоскости,
можно провести через точку, не лежащую в этой плоскости?

§ 14. Параллельность прямой и плоскости

51

11. Может ли прямая быть параллельной сразу двум
пересекающимся плоскостям?
Задачи для закрепления
1. Докажите, что если две плоскости, пересекающиеся
по прямой a, пересекают плоскость α по параллельным
прямым, то прямая a параллельна плоскости α.
Доказательство. Проведем доказательство от противного. Допустим, что плоскости β и γ пересекаются по
прямой a и пересекают плоскость α по параллельным
прямым b и c. Если прямая a не параллельна плоскости α,
т. е. пересекает плоскость α в некоторой точке A, то точка A
принадлежит всем трем плоскостям α, β и γ, а поэтому
и прямым b и c. Таким образом, прямые b и c пересекаются
в точке A в противоречие с условием.
2. Через данную точку проведите плоскость, параллельную
каждой из двух пересекающихся прямых. Всегда ли это
возможно?
Решение. Проведем через точку A прямые a1 и b1 ,
параллельные прямым a и b (считается, что точка A не
принадлежит ни прямой a, ни прямой b). Прямые a1 и b1
определяют плоскость α, так как они тоже пересекающиеся,
т. е. несовпадающие, как прямые a и b (почему?). Эта
плоскость будет параллельна прямым a и b, если прямые a
и b не лежат в ней (теорема 18). Если одна из этих
прямых лежит в плоскости α, то в ней лежит и вторая
прямая, что следует из теоремы 2. Но тогда точка A
лежит в плоскости α0 , определяемой прямыми a и b.
Значит, нужную плоскость можно построить (указанным
способом!) только в том случае, когда данная точка A не
лежит в плоскости α0 прямых a и b.
Более точно: наше построение дает нужную плоскость
только в случае A 6∈ α0 . Если же A ∈ α0, то плоскости α,
проходящей через A и параллельной прямым a и b,
вовсе не существует; однако это требует специального
доказательства. В самом деле, если такая плоскость α
существует, то она не совпадает с α0 , ибо лежащие
в плоскости α0 прямые a и b параллельны плоскости α.

52

Глава 2. Параллельные прямые и плоскости

Но тогда, согласно теореме 18, прямая c пересечения
плоскостей α и α0 (она существует, так как точка A
принадлежит обеим плоскостям) параллельна каждой из
двух пересекающихся прямых a и b, а это противоречит
теореме 15. Тем самым утверждаемое доказано.
Упражнения
1(14.1) Пусть прямая a пересекает плоскость. Верно ли,
что в плоскости α найдется прямая, параллельная прямой a?
О т в е т: нет.
2(14.2) Какие из вариантов а)–в) приведенных ниже утверждений, верны? Если одна из двух а) параллельных,
б) пересекающихся, в) скрещивающихся прямых пересекает данную плоскость, то и вторая прямая пересекает
эту плоскость.
О т в е т: а).
3(14.3) Пусть прямые a и b параллельны плоскости α.
Следует ли отсюда, что a k b?
О т в е т: нет.
4(14.4) Верно ли, что через точку A, не принадлежащую
данной плоскости α, можно провести единственную прямую, параллельную плоскости α?
О т в е т: нет.
5(14.5) Пусть плоскости α и β пересекаются, прямая a
лежит в плоскости α, а прямая b — в плоскости β. Могут
ли прямые a и b : а) пересекаться, б) лежать в одной
плоскости, в) быть параллельными, г) скрещиваться?
О т в е т: а) да; б) нет; в) да; г) да.
6(14.6) Плоскость γ пересекает две данные пересекающиеся
плоскости α и β соответственно по прямым a и b. Могут ли
прямые a и b : а) пересекаться, б) быть параллельными,
в) скрещиваться?
О т в е т: а) да; б) нет; в) нет.
7(14.7) Пусть EF — средняя линия трапеции ABCD с основаниями AB и CD. Следует ли из параллельности прямых
AB и CD данной плоскости α, что прямая EF параллельна
этой плоскости?
О т в е т: да.

§ 14. Параллельность прямой и плоскости

53

8(14.8) Плоскости α, β и γ попарно пересекаются, но не
имеют общих для всех трех плоскостей точек. Существуют
ли в пространстве прямые, параллельные всем трем
плоскостям?
О т в е т: да.
9(14.9) Как вы думаете, какое множество точек в пространстве заполняют все прямые, параллельные данной
плоскости и проходящие через точку A?
О т в е т: плоскость.
10(14.11) Проведите через точку M пространства плоскость, параллельную двум данным скрещивающимся прямым a и b.
Решение. Проведем через точку M прямые a1 и b1 , параллельные соответственно прямым a и b. Тогда плоскость,
проходящая через прямые a1 и b1 (а следовательно, и через
точку M их пересечения), параллельна каждой из прямых a
и b.
11(14.12) Постройте прямую, скрещивающуюся с каждой
из двух данных: а) пересекающихся; б) скрещивающихся
прямых.
Решение. а) Пусть α — плоскость, содержащая пересекающиеся прямые a и b (рис. 2.11). Возьмем в плоскости α
точку M, не лежащую ни на одной из прямых a и b, а вне
плоскости α — точку C. Тогда прямая CM скрещивается
с каждой из прямых a и b по признаку скрещивающихся
прямых.
C
α

M
a

b

Рис. 2.11.

б) Возьмем в пространстве точку M, не лежащую
на скрещивающихся прямых a и b, и проведем через

54

Глава 2. Параллельные прямые и плоскости

нее плоскость α, параллельную прямым a и b. Пусть
прямые a1 и b1 лежат в плоскости α, причем a1 k a, b1 k b
(рис. 2.12), тогда любая прямая плоскости α, проходящая
через точку M и отличная от прямых a1 и b1 , скрещивается
с каждой из прямых a и b.
a
a1
α

b1

M
b

Рис. 2.12.

12(14.13) Докажите, что: а) если плоскость пересекает
прямую, содержащую одно из оснований трапеции, то
она пересекает и прямую, содержащую другое основание;
б) если плоскость пересекает данную параболу в двух
различных точках, то она пересекает и ось параболы.
Решение. а) Предположив, что плоскость не пересекает
прямую, содержащую второе основание трапеции, т. е.
параллельна этой прямой, получим, что она не пересекает и прямую, содержащую первое основание трапеции,
поскольку если одна из двух параллельных прямых
параллельна данной плоскости, то и вторая прямая параллельна этой плоскости. Итак, мы получилипротиворечие
с условием задачи.
б) Пусть плоскость α, содержащая две точки A и B
параболы, пересекает плоскость β, содержащую параболу.
Тогда прямая пересечения этих плоскостей будет проходить
через две точки A, B и пересекать ось параболы. Следовательно, плоскость α тоже пересекает ось параболы.
13(14.14) Прямая a параллельна плоскости α. Верно ли,
что: а) прямая b, параллельная прямой a, параллельна
плоскости; б) прямая b, параллельная плоскости α, параллельна прямой a?
О т в е т: а) да; б) нет.

§ 14. Параллельность прямой и плоскости

55

14(14.15) Постройте прямую, проходящую через заданную
точку и параллельную двум данным пересекающимся
плоскостям.
Решение. Прямая, проходящая через заданную точку
параллельно линии пересечения плоскостей, будет параллельна этим плоскостям.
15(14.16) Докажите, что если три плоскости, не проходящие через одну прямую, попарно пересекаются, то прямые,
по которым они пересекаются, либо параллельны, либо
имеют общую точку.
Доказательство. Пусть плоскости α и β пересекаются
по прямой c, β и γ — по прямой a, γ и α — по прямой b.
Предположим, что какие-либо две из этих прямых, например прямые a и b, пересекаются в точке M. Тогда M ∈ a,
M ∈ b и, поскольку a = β ∩ γ, b = α ∩ γ, имеем M ∈ β, M ∈ γ
и M ∈ a, т. е. плоскости α, β и γ пересекаются в точке M.
Покажем, что если прямые a, b и c не имеют общих
точек, то они параллельны. В самом деле, предположив,
что прямая a пересекает плоскость α, получим, что
плоскости α, β и γ имеют общую точку, т. е. прямые
a, b и c пересекаются. Итак, в нашем случае a k α, но
тогда плоскости β и γ, проходящие через прямую a,
параллельную плоскости α, пересекают эту плоскость по
прямым c и b, параллельным прямой a, откуда следует,
что эти три прямые параллельны.
16(14.17) Приведите пример: а) трех таких прямых, которые нельзя пересечь никакой четвертой прямой; б) трех
плоскостей таких, что любая прямая пространства пересекает хотя бы одну из них.
О т в е т: а) три параллельные прямые, не лежащие в одной
плоскости; б) любые три плоскости, имеющие общую
точку.
17(14.18) Дана четырехугольная пирамида ABCDM,
в основании которой лежит трапеция ABCD (AB k CD).
Постройте прямые пересечения граней: а) ADM и BCM;
б) ABM и CDM.

56

Глава 2. Параллельные прямые и плоскости

Решение. а) Продолжим отрезки AD и BC до пересечения
в точке P (рис. 2.13). Так как прямая AD лежит в плоскости
ADM, а прямая BC — в плоскости BCM, то точка P принадлежит прямой пересечения этих плоскостей. Поскольку
точка M также принадлежит прямой пересечения указанных плоскостей, плоскости ADM и BCM пересекаются
по прямой PM.
б) Так как ABCD — трапеция, то
M
AB k CD, но линия пересечения двух
плоскостей, проходящих через две
параллельные прямые, параллельна
P
этим прямым, поэтому искомой пряD
C
мой является прямая, проходящая через
A
B точку M и параллельная прямой AB.
Рис. 2.13.

18(14.20) Дан тетраэдр ABCD и точки
K, M, P, лежащие на ребрах AD, BD, CD соответственно.
Постройте прямую пересечения плоскостей KMP и ABC.
Решение. Пусть ABCD — данный тетраэдр (рис. 2.14).
Выберем на одном из его ребер (на рис. 2.14 — BD) какуюлибо точку K и проведем через нее прямые KN и KL,
параллельные ребрам AD и BC соответственно. Покажем,
что плоскость, проходящая через прямые KN и KL,
пересекает тетраэдр ABCD по параллелограмму KLMN,
где M — точка пересечения этой плоскости с ребром AC.
Действительно, KN k AD, поэтому прямая KN параллельна
плоскости ACD, откуда следует, что KN k LM. Аналогично,
KL k NM. Итак, мы доказали, что четырехугольник KLMN
является параллелограммом.
D
L
M
A

K
N
B

Рис. 2.14.

C

Далее нам встретятся несколько
задач, в которых требуется построить сечение многогранника (точнее —
изображение сечения на изображении многогранника; более подробно
об этом поговорим в § 16 и далее
в 11 классе) некоторой плоскостью.
Договоримся линию пересечения секущей плоскости с гранью многогранника называть следом сечения.

§ 14. Параллельность прямой и плоскости

57

Построить сечение означает построить все его следы.
При этом разрешенные действия таковы: а) имеем право
провести прямую через две данные точки; б) имеем право
отметить точку пересечения двух данных прямых (разумеется, если таковая имеется); в) можно также провести через
данную точку прямую, параллельную данной прямой.
К примеру, категорически воспрещается «строить»
точку пересечения данной прямой и данной плоскости
«просто так», не прибегая к каким-либо дополнительным
соображениям.
Вообще же, главное, к чему нужно стремиться, строя
сечение, — это искать пары точек, которые одновременно
принадлежали бы какой-либо грани и секущей плоскости, ведь тогда прямая, проходящая через эти точки,
обязательно будет содержать след сечения, являясь линией
пересечения грани многогранника и секущей плоскости.
19(14.21) Дан куб ABCDA1B1 C1 D1 . Найдите сечение этого
куба плоскостью, проходящей через вершины B, D1 , A1 .
Решение. Плоскость сечения проходит через прямую A1 D1
и точку B на параллельной ей прямой BC, поэтому она
совпадает с плоскостью, содержащей параллельные прямые
BC и A1 D1 . Для построения сечения остается только
соединить точки B и A1 , C и D1 (рис. 2.15).
D1

A1

C1

B1
D

A

C

B

Рис. 2.15.

58

Глава 2. Параллельные прямые и плоскости

§ 15. ПАРАЛЛЕЛЬНОСТЬ ПЛОСКОСТЕЙ

Рассмотрим теперь взаимное расположение двух плоскостей.
Согласно аксиоме 2, если две различные плоскости
имеют хотя бы одну общую точку, то они пересекаются
по прямой — в этом случае плоскости называются просто
пересекающимися. Логически возможен еще один случай:
общих точек вообще нет, т. е. плоскости не пересекаются —
в этом случае они называются параллельными.
Две плоскости называются параллельными, если они
не пересекаются.
Возникает вопрос о существовании параллельных плоскостей. Поставим его иначе. Пусть даны плоскость α
и точка A вне плоскости α. Как построить плоскость β,
проходящую через точку A и параллельную плоскости α?
Всегда ли это возможно? Нетрудно догадаться, как можно
построить такую плоскость β. Проведем через точку A две
различные прямые b1 и b2 , параллельные плоскости α.
Плоскость, проходящая через прямые b1 и b2 , и будет
искомой. Действительно, если бы плоскость β пересекалась
с плоскостью α по некоторой прямой c, то, по аксиоме
параллельных, хотя бы одна из прямых b1 или b2 пересекала бы прямую c, а значит, пересекала бы и плоскость α,
что противоречит выбору прямых b1 и b2 .
Таким образом, доказано существование плоскости,
параллельной данной плоскости и проходящей через
данную точку (вопрос о единственности потом мы тоже
обсудим), и получен признак параллельности плоскостей:
если плоскость параллельна двум пересекающимся
прямым, лежащим в другой плоскости, то эти две
плоскости параллельны.
В учебнике эта теорема сформулирована по-иному.
Теорема 22 (признак параллельности плоскостей). Если
две пересекающиеся прямые одной плоскости параллельны другой плоскости, то эти плоскости параллельны.
Вопрос: существенно ли в этих двух признаках параллельности плоскостей, что прямые пересекаются?

§ 15. Параллельность плоскостей

59

Ответ: конечно, ибо можно взять
β
две плоскости α и β, пересекающиеся
по прямой c, а в плоскости β — две
c
прямые b1 и b2 , параллельные прямой
α
b1
c (рис. 2.16); тогда прямые b1 и b2
параллельны плоскости α, но плоскость
b2
β отнюдь не параллельна плоскости α.
Рис. 2.16.
Следующую задачу целесообразно
разобрать с помощью учащихся.
Задача 1. Даны две скрещивающиеся прямые. Как провести через них две параллельные плоскости?
Решение. Пусть a и b — данные скрещивающиеся прямые.
Через произвольную точку прямой a проведем прямую b′ ,
параллельную b, а через произвольную точку прямой b
проведем прямую a′ , параллельную a. Теперь проведем
две плоскости: одну через прямые a и b′ , а другую через
прямые b и a′ . По теореме 22, эти плоскости параллельны.
В первой из них лежит прямая a, а во второй — прямая b.
Вопросы к решению: не может ли оказаться так,
что плоскости совпадут (тогда теорема 22 неприменима)?
В решении неявно использована теорема 21 (где, в каком
месте?), а применима ли она? Скажем, не может ли
прямая b′ оказаться лежащей в плоскости прямых b и a′ ?
Ответ в обоих случаях: не может, иначе мы получим
противоречие с тем, что прямые a и b скрещиваются.
Задача 2. Плоскости α и β параллельны плоскости γ. Могут
ли плоскости α и β пересекаться?
Решение. Плоскости α и β не могут пересекаться. Если
бы плоскости α и β имели общую точку, то через эту
точку проходили бы две плоскости (α и β), параллельные
плоскости γ. А это противоречит теореме 23.
Следующие задачи рекомендуются для самостоятельного
решения.
Задача 3. Через вершины параллелограмма ABCD, лежащего в одной из двух параллельных плоскостей, проведены
параллельные прямые, пересекающие вторую плоскость

60

Глава 2. Параллельные прямые и плоскости

в точках A1 , B1 , C1 , D1 . Докажите, что четырехугольник
A1 B1 C1 D1 — тоже параллелограмм.
Решение. Пользуясь условием, теоремой 11 и теоремой
4,
устанавливаем
параллельность
прямых:
A1 B1 k AB k CD k C1D1 и аналогично A1 D1 k B1 C1 , поэтому
A1 B1 C1 D1 — параллелограмм.
Задача 4. Даны две параллельные плоскости α1 и α2
и точка A, не лежащая ни в одной из этих плоскостей.
Через точку A проведена произвольная прямая. Пусть
X1 и X2 — точки пересечения ее с плоскостями α1 и α2 .
Докажите, что отношение длин отрезков AX1 : AX2 не
зависит от взятой прямой.
Решение. Проведем через точку A другую прямую и обозначим через Y1 и Y2 точки пересечения ее с плоскостями
α1 и α2 . Проведем через прямые AX1 и AY1 плоскость.
Она пересечет плоскости α1 и α2 по параллельным прямым
X1 Y1 и X2 Y2 (теорема 11). Отсюда следует подобие треугольников AX1 Y1 и AX2 Y2 . А из подобия треугольников
следует пропорция, т. е. отношения AX1 : AX2 и AY1 : AY2
одинаковы для обеих прямых.
Заметим, что возможен еще случай, когда точка A
лежит между плоскостями α1 и α2 ; рассуждения остаются
справедливыми и в этом случае. Подобие треугольников
выводится тоже из признака подобия по двум углам.
Задача 5. Три прямые, проходящие через одну точку, пересекают данную плоскость в точках A, B, C, а параллельную
ей плоскость — в точках A1 , B1 , C1 . Докажите подобие
треугольников ABC и A1 B1 C1 .
Решения задач 3 и 5 не должны затруднить учащихся.
В решениях задач 4 и 5 следует требовать четких
ссылок на признаки подобия треугольников, которые стоит
предварительно повторить: по трем сторонам, по двум
сторонам и углу между ними и по двум углам. При
разборе решений имеет смысл вспомнить определение
подобных фигур, преобразования подобия и гомотетию
как частный случай преобразования подобия. Отметим,

§ 15. Параллельность плоскостей

61

что в задаче 5 речь идет о подобных треугольниках, не
лежащих в одной плоскости; можно считать, что и в этом
случае для доказательства подобия можно пользоваться
планиметрическими признаками, однако полезно указать
учащимся и на гомотетию в пространстве, отображающую
плоскость ABC на плоскость A1 B1 C1 .
Задача 6. Через данную точку пространства проведите
прямую, пересекающую каждую из двух скрещивающихся
прямых. Всегда ли это возможно?
Решение. Это классическая задача, несколько неожиданная по формулировке. Ее решение можно показать «на
пальцах»: если посмотреть «со стороны» на две скрещивающиеся прямые, то (как правило!) мы увидим их как
бы пересекающимися — вот и проведем прямую по лучу
из «точки зрения» (это данная точка) к видимой «точке
пересечения». Такой взгляд «со стороны» подсказывает
строгое решение.
Пусть a, b — данные скрещивающиеся прямые, C —
данная точка. Искомая прямая c вместе с каждой из
прямых a, b определяет плоскость; обозначим эти плоскости соответственно α и β. Обе они содержат точку C
и, следовательно, однозначно определяются точкой C
и прямыми a, b и без прямой c. Наоборот, эти плоскости
α и β, которые мы можем построить по точке C и прямым
a, b, в пересечении дадут прямую c.
Если окажется, что прямая c пересекает прямые a и b, то
c — искомая прямая. Если же прямая c будет параллельна
прямой a или b, то получается, что нужной прямой не
существует.
Если получилось, что прямая c параллельна прямой a,
то это означает параллельность прямой a и плоскости β.
Но тогда точка C принадлежит плоскости, проведенной
через прямую b параллельно прямой a. (Из теоремы 7
нетрудно вывести, что для скрещивающихся прямых a
и b этими условиями — проходить через b и быть параллельной прямой a — плоскость определяется однозначно.)
Аналогично, если точка C лежит в плоскости, проведенной
через прямую a параллельно прямой b, решения тоже не
существует. Конечно, эти рассуждения справедливы лишь

62

Глава 2. Параллельные прямые и плоскости

тогда, когда точка C не лежит ни на прямой a, ни на
прямой b. В этих двух случаях можно провести бесконечно
много прямых, удовлетворяющих условию задачи.
Вопросы для закрепления
1. Каким может быть взаимное расположение двух плоскостей в пространстве? (Описать, обосновать и показать.)
2. В каком случае две плоскости называются параллельными? пересекающимися?
3. Как через данную точку провести плоскость, параллельную данной плоскости?
4. Сформулируйте признак параллельности плоскостей
(в двух вариантах — с двумя и с четырьмя прямыми).
5. Существенно ли в признаке параллельности плоскостей,
что две прямые пересекаются?
6. Сформулируйте теорему о пересечении двух параллельных плоскостей третьей плоскостью.
7. Может ли прямая (или плоскость) пересекать одну из
двух параллельных плоскостей, но не пересекать другую?
8. Что можно сказать о двух различных плоскостях,
параллельных третьей плоскости?
9. Сформулируйте теорему об отрезках параллельных прямых, заключенных между параллельными плоскостями.
10. Сформулируйте теорему о (существовании и единственности) параллельной плоскости.
11. Сколько различных плоскостей, параллельных данной
плоскости, можно провести в пространстве?
12. Верно ли, что всякая прямая, лежащая в одной из двух
параллельных плоскостей, параллельна другой плоскости?
13. Параллельны ли две данные плоскости, если третья
плоскость пересекает их по параллельным прямым?
14. Две пересекающиеся плоскости пересекают третью
плоскость по двум прямым. Будут ли эти прямые параллельными?

§ 15. Параллельность плоскостей

63

Задачи для закрепления
1. Докажите, что если прямая пересекает одну из двух
параллельных плоскостей, то она пересекает и другую.
Решение. Пусть плоскости α и β параллельны, прямая c
пересекает плоскость α в точке a. Допустим, эта прямая
не пересекается с плоскостью β. Возьмем в плоскости β
точку B и проведем плоскость γ через прямую c и точку B.
Тогда плоскость γ пересекается с плоскостями α и β по
параллельным прямым a и b. С другой стороны, прямая c
параллельна плоскости β, а поэтому прямая c параллельна
и прямой b. Итак, в плоскости γ через точку A к прямой b
проведены две различные параллельные прямые a и c —
противоречие.
Вопрос при разборе: пусть плоскость α параллельна
плоскости β, а прямая c параллельна плоскости α. Каково
взаимное расположение прямой c и плоскости β?
Ответ: если прямая c не лежит в плоскости β, то
прямая c параллельна плоскости β. Таким образом,
получен своеобразный аналог теоремы 17: если плоскость β
и не лежащая в ней прямая c параллельны некоторой
плоскости α, то они (c и β) параллельны между собой.
2. Плоскости α и β пересекаются. Докажите, что любая
плоскость γ пересекает хотя бы одну из плоскостей α, β.
Решение. Если бы плоскость γ не пересекалась ни с одной
из плоскостей α, β, то плоскости α и β были бы
параллельны в противоречие с условием задачи.
3. Через вершины треугольника ABC, лежащего в одной из
двух параллельных плоскостей, проведены параллельные
прямые, пересекающие вторую плоскость в точках A1 , B1 ,
C1 . Докажите равенство треугольников ABC и A1 B1 C1 .
Решение. Из условия задачи и теоремы 24 следует,
что четырехугольники ABB1 A1 , BCC1B1 и CAA1 C1 суть
параллелограммы (их противолежащие стороны попарно
параллельны), поэтому AB = A1B1 BC = B1C1 , AC = A1C1
и требуемое следует из признака равенства треугольников
по трем сторонам.

64

Глава 2. Параллельные прямые и плоскости

При обсуждении этой задачи в классе следует вспомнить
все признаки равенства треугольников: первый — по двум
сторонам и углу между ними, второй — по стороне и двум
прилежащим к ней углам, третий — по трем сторонам.
4. Докажите, что если четыре прямые, проходящие через
точку A, пересекают плоскость α в вершинах параллелограмма, то они пересекают любую плоскость, параллельную α и не проходящую через A, тоже в вершинах
параллелограмма.
Решение. Если O — данная точка, ABCD — данный параллелограмм, то нужно рассмотреть плоскости OAB, OBC,
OCD и ODA и воспользоваться теоремами 24 и 17.
5. Даны две параллельные плоскости. Через точки A
и B одной из этих параллельных плоскостей проведены
параллельные прямые, пересекающие вторую плоскость
в точках A1 и B1 . Чему равен отрезок A1 B1 , если AB = a?
Решение. ABB1 A1 — параллелограмм, поэтому A1 B1 =
= AB = a.
6. Даны три параллельные плоскости: α1 , α2 , α3 . X1 ,
X2 , X3 — точки пересечения этих плоскостей с произвольной прямой. Докажите, что отношение длин отрезков
X1 X2 : X2 X3 не зависит от прямой, т. е. одинаково для
любых двух прямых.
Решение. Пусть другая прямая пересекает плоскости
α1 , α2 , α3 в точках Y1 , Y2 , Y3 . Проведем через точку
X1 прямую, параллельную второй прямой, и обозначим ее точки пересечения с α2 и α3 через Z2 и Z3 .
Тогда X1 X2 : X2 X3 = X1 Z2 : Z2 Z3 (почему?). X1 Z2 = Y1Y2
и Z2 Z3 = Y2Y3 , поэтому X1 X2 : X2 X3 = Y1Y2 : Y2Y3 , что и требовалось установить. Рассуждение, конечно, не зависит от
порядка «следования» плоскостей α1 , α2 , α3 в пространстве.
7. Даны четыре параллельные прямые. Докажите, что если
какая-нибудь плоскость пересекает эти прямые в вершинах
параллелограмма, то любая плоскость, не параллельная
данным прямым, пересекает их в вершинах некоторого
параллелограмма.

§ 15. Параллельность плоскостей

65

Решение. Пусть a, b, c, d — данные прямые, пересекающиеся с плоскостью α в вершинах параллелограмма ABCD.
Пусть вторая плоскость β пересекает эти прямые в точках
A1 , B1 , C1 , D1 соответственно. Плоскости, проходящие
через пары параллельных прямых a, b и c, d, т. е.
плоскости ABB1 A1 и CDD1 C1 , параллельны, поскольку
прямые AB и CD, a и d параллельны. Следовательно,
плоскость β пересекает эти плоскости по параллельным
прямым A1 B1 и C1 D1 . Точно так же доказывается, что
и прямые B1 C1 и A1 D1 параллельны. Таким образом,
A1 B1 C1 D1 — параллелограмм.
Упражнения
1(15.1) Через всякую ли а) точку, б) прямую в пространстве, не лежащую в данной плоскости, можно провести
плоскость, параллельную этой плоскости?
О т в е т: а) да; б) нет.
2(15.2) Вспомните признак параллельности плоскостей.
Является ли требование пересечения прямых в этом
признаке существенным, т. е. верно ли, что если две
прямые одной плоскости параллельны другой плоскости,
то эти плоскости параллельны?
О т в е т: требование пересечения прямых в признаке
параллельности прямых является существенным, так как,
например, если две параллельные прямые одной плоскости
параллельны двум параллельным прямым другой плоскости, то такие плоскости могут пересекаться по линии,
параллельной рассматриваемым прямым.
3(15.3) Верно ли, что произвольная плоскость пересекает
хотя бы одну из двух данных пересекающихся прямых?
О т в е т: нет.
4(15.4) Верно ли, что плоскость, пересекающая одну
из двух скрещивающихся прямых, пересечет и вторую
прямую?
О т в е т: нет.
5(15.5) Пусть плоскости α и β параллельны прямой c.
Следует ли отсюда, что плоскости α и β параллельны?
О т в е т: нет.

66

Глава 2. Параллельные прямые и плоскости

6(15.6) Покажите, что противоположные грани куба лежат
в параллельных плоскостях.
Решение. Противоположные грани куба лежат в параллельных плоскостях по признаку параллельности двух
плоскостей.
7(15.7) Сколько плоскостей, параллельных данной плоскости α, можно провести через данную прямую a, если
a k α?
О т в е т: одну.
8(15.8) Можно ли провести плоскость γ, параллельную
двум данным пересекающимся плоскостям α и β?
О т в е т: нет.
9(15.9) Могут ли быть равны отрезки непараллельных прямых, заключенных между параллельными плоскостями?
О т в е т: да.
10(15.10) Всегда ли парабола, пересекающая одну из
параллельных плоскостей, пересекает и вторую?
О т в е т: да.
11(15.11) Прямые a и b пересекают две параллельные
плоскости γ1 и γ2 . Длины отрезков этих прямых, заключенные между плоскостями γ1 и γ2 , равны. Являются ли
прямые a и b параллельными?
О т в е т: прямые a и b не обязаны быть параллельными.
Проведем, например, через стороны AB и A1 B1 прямоугольника AA1 B1 B параллельные плоскости α и β. Тогда
AB1 = BA1 как диагонали прямоугольника, причем прямые
AB и BA1 пересекаются (рис. 2.17).
A

B

α
O

β A1

B1

Рис. 2.17.

§ 15. Параллельность плоскостей

67

12(15.12) Какое множество точек в пространстве образуют
все плоскости, проходящие через данную прямую и пересекающие параллельную этой прямой плоскость?
О т в е т: все точки пространства, за исключением точек
плоскости, проходящей через данную прямую и параллельной данной плоскости.
13(15.13) Может ли в сечении куба плоскостью получиться
а) правильный треугольник; б) правильный пятиугольник?
О т в е т: а) может, например сечение A1 BD (рис. 2.18);
б) не может, так как пятиугольник, являющийся сечением
куба, должен иметь две пары параллельных сторон.
D1

A1

C1

B1

C
D
A

B

Рис. 2.18.

14(15.15) Какие правильные многоугольники могут получиться в сечении куба плоскостью?
О т в е т: правильный треугольник, квадрат и правильный
шестиугольник.
15(15.16) Какую форму имеет сечение куба плоскостью,
проходящей через а) ребро; б) диагональ грани; в) диагональ куба?
О т в е т: а) прямоугольник; б) треугольник, трапеция или
прямоугольник; в) параллелограмм или прямоугольник.
16(15.17) В
основании
четырехугольной
пирамиды
ABCDM лежит параллелограмм (рис. 2.19). Постройте
сечение этой пирамиды плоскостью, проходящей через
диагональ BD основания и параллельной ребру AM.

68

Глава 2. Параллельные прямые и плоскости
M
P
C

D
O
A

B

Рис. 2.19.

Решение. Плоскость AMC пересекает плоскость сечения
по прямой, параллельной AM; поэтому для построения
сечения достаточно через точку пересечения диагоналей
параллелограмма O провести прямую, параллельную AM.
Эта прямая пересечет ребро MC в точке P, совпадающей
с его серединой. Таким образом, в сечении получился
треугольник DBP.
17(15.19) Постройте сечение пирамиды ABCDM плоскостью, проходящей через точки D и K и параллельной
ребру MC (рис. 2.20).
M
E

A

D
C

P
K
B

N

Рис. 2.20.

Решение. Это плоскость DEPK.
Проведем прямую через K параллельно MC до пересечения с BM в точке P; N = AB ∩ DK; E = AM ∩ NP.
18(15.20) Постройте сечение куба ABCDA1B1 C1 D1 плоскостью AKM, где точка K принадлежит ребру BC, а точка
M — грани DCC1D1 (рис. 2.21).

§ 15. Параллельность плоскостей

69

R
D1

P

C1

M

N

Q
A1

B1
D

C
E
K

A

B

Рис. 2.21.

Решение. Это плоскость AKNPQ; E = AK ∩ DC; N =
= ME ∩ CC1 ; P = EN ∩ D1 C1 ; R = EP ∩ DD1; Q = AR ∩ A1D1 .
19(15.21) Дан куб ABCDA1B1 C1 D1 , точка P лежит на
1

диагонали BD грани ABCD, причем BP = BD, а точка M
3
совпадает с серединой диагонали BD1 куба. Установите
форму сечения куба плоскостью AMP.
О т в е т: параллелограмм.
20(15.22) В кубе ABCDA1B1 C1 D1 на ребре BB1 взята
точка K, а точка M принадлежит грани DCC1 D1 (рис. 2.22).
Найдите сечение куба плоскостью AKM.
D1

C1
K1
B1
K

A1

Q
M

D
P
A

C

B

Рис. 2.22.

Решение. Построим точку K1 ∈ CC1 ; KK1 k B1 C1 . Проведем
через M прямую l, параллельную прямой DK1 ; P = DC ∩ l;
Q = CC1 ∩ l. Искомое сечение AKQP.

70

Глава 2. Параллельные прямые и плоскости

§ 16. ПАРАЛЛЕЛЬНОЕ ПРОЕКТИРОВАНИЕ

Рассмотрение основных свойств параллельного проектирования целесообразно предварять наводящими вопросами.
Отметим некоторые свойства изображения фигуры на
плоскости, вытекающие из описанного ее построения.
Свойство I
Вопрос: чем может быть изображен на плоскости отрезок?
Иначе говоря, какой фигурой может оказаться параллельная проекция отрезка? (Следует отметить, что в случае,
когда отрезок параллелен прямой проектирования h, он
изобразится точкой.) Как обосновать ответ? Обоснование
базируется на следствии 11.
Прямолинейные отрезки фигуры изображаются на
плоскости чертежа отрезками. Действительно, все прямые, проектирующие точки отрезка AC, лежат в одной
плоскости, пересекающей плоскость чертежа α по прямой
A1 C1 . Произвольная точка B отрезка AC изображается
точкой B1 отрезка A1 C1 .
З а м е ч а н и е . В только что доказанном свойстве и далее
предполагается, конечно, что проектируемые отрезки не
параллельны направлению проектирования.
В действительности этим рассуждением доказано только
то, что проекцией каждой точки отрезка AC является
точка отрезка A1 C1 . Доказательство же того, что каждая
точка отрезка A1 C1 получается как проекция точки отрезка
AC, опущено. Это можно установить, взяв произвольную
точку B1 отрезка A1 C1 и проведя через нее прямую,
параллельную прямой h : она будет лежать во вспомогательной плоскости проектирования и пересечет отрезок AC
в некоторой точке B, которая и проектируется в точку B1 .
Обсуждение свойства I целесообразно завершить весьма
существенным его следствием:
многоугольники (плоские или пространственные) изображаются на плоскости чертежа многоугольниками
(или в исключительных случаях отрезками).
Интересный вопрос: может ли при параллельном проектировании многоугольника получиться многоугольник

§ 16. Параллельное проектирование

71

с меньшим числом сторон? Если многоугольник пространственный, т. е. его стороны (или вершины) не лежат в одной
плоскости, то ответ положительный: число сторон после
проектирования может уменьшиться (достаточно спроектировать многоугольник параллельно одной из его сторон).
Для плоских многоугольников ответ отрицательный.
Свойство II
Вопрос: как изобразятся при параллельном проектировании параллельные прямые (отрезки)? Полный ответ:
параллельными или совпадающими прямыми (параллельными или лежащими на одной прямой отрезками). Строгое
доказательство этого факта существенно использует признак параллельности плоскостей (теорему 22) и теорему 24
о пересечении двух параллельных плоскостей третьей.
Параллельные отрезки фигуры изображаются на
плоскости чертежа параллельными отрезками или
отрезками, лежащими на одной прямой. Действительно,
пусть AC и A′ C′ — параллельные отрезки фигуры. Прямые
A1 C1 и A′1 C′1 параллельны, так как они получаются при
пересечении параллельных плоскостей с плоскостью α.
Первая из этих плоскостей проходит через прямые AC
и AA1 , а вторая — через прямые A′ C′ и A′ A′1 .
Вопросы: почему указанные плоскости параллельны?
Всегда ли это так (для параллельных AC и A′ C′ )? Какие
теоремы используются при доказательстве свойства II?
Ответ: кроме упомянутых теорем 22, 24 нужна еще
теорема 17 — признак параллельности прямых, который
обеспечивает параллельность прямых проектирования.
Свойство III
Вопросы: сохраняются ли длины отрезков при параллельном проектировании? Приведите пример. Сохраняются ли
отношения длин? Приведите пример. Оказывается, при
дополнительных ограничениях на отрезки отношение их
длин все-таки сохраняется.
Отношение отрезков одной прямой или параллельных прямых при параллельном проектировании сохраняется.

72

Глава 2. Параллельные прямые и плоскости

Покажем, например, что AB : BC = A1B1 : B1 C1 .
Проведем через точку B прямую, параллельную A1 C1 .
Треугольники BAA2 и BCC2 подобны. Из подобия треугольников следует пропорция.
Вопросы: почему указанные треугольники подобны? Как
изменится доказательство в том случае, когда рассматриваются не прилежащие друг к другу отрезки одной прямой?
параллельные отрезки различных прямых?
Отметим, что доказательства основных свойств параллельной проекции запоминать учащимся не нужно, однако
эти свойства I, II, III (их обоснование) вполне могут быть
предложены в качестве задач (при опросе по задачам, на
контрольной работе, на зачете).
Задачи 1, 2, 3 можно предложить для самостоятельного
решения или же решить устно всем классом,
Задача 1. Дана параллельная проекция треугольника. Как
построить проекции медиан этого треугольника?
Решение. При параллельном проектировании сохраняется
отношение отрезков прямой. Поэтому середина стороны
треугольника проектируется в середину проекции этой
стороны. Следовательно, проекции медиан треугольника
будут медианами его проекции.
Эта задача имеет довольно
неожиданное и красивое применение. Очевидно, в правильном (равC
ностороннем) треугольнике медианы пересекаются в одной точке —
h
в центре треугольника. Но произвольный треугольник ABC можно
B
C1
спроектировать в правильный треA
угольник ABC1 (рис. 2.23), достаточно построить такой правильный
треугольник в плоскости ABC1, не
Рис. 2.23.
проходящей через точку C, и проектировать параллельно прямой h = CC1. Рассматривая эту
проекцию, из результата первой задачи выводим, что
медианы любого треугольника пересекаются в одной точке.
Задача 2. Может ли при параллельном проектировании
параллелограмма получиться трапеция? Объясните ответ.

§ 16. Параллельное проектирование

73

О т в е т: не может. Так как при параллельном проектировании параллельные прямые изображаются параллельными,
параллельной проекцией параллелограмма будет параллелограмм (или отрезок, если плоскость проектируемого
параллелограмма параллельна прямой проекции h). Эту
задачу можно дать сразу же после рассмотрения свойства II
параллельного проектирования.
Задача 3. Докажите, что параллельная проекция центрально-симметричной фигуры также является центрально-симметричной фигурой.
Решение. Так как при параллельном проектировании
середина отрезка переходит в середину его проекции, проекция центра симметрии фигуры будет являться центром
симметрии проекции этой фигуры.
Если позволяет время, урок можно закончить построением изображений квадрата или правильного шестиугольника по заданным изображениям каких-нибудь трех
характерных точек.
Вопросы для закрепления
1. Чем задается параллельная проекция? Как построить
проекцию данной точки на данную плоскость параллельно
данной прямой? Что такое проекция (пространственной)
фигуры?
2. Что можно сказать о параллельной проекции прямой?
отрезка?
3. Что можно сказать о параллельных проекциях параллельных прямых? отрезков?
4. Сохраняются ли при параллельном проектировании
длины отрезков? отношения длин отрезков? Приведите
примеры. В каком случае отношение длин отрезков при
параллельном проектировании все-таки сохраняется?
Задачи для закрепления
1. Дана параллельная проекция треугольника. Чем изобразится проекция средней линии треугольника?

74

Глава 2. Параллельные прямые и плоскости

О т в е т: проекция средней линии является средней линией
проекции треугольника, так как середины отрезков проектируются в середины проекций этих отрезков.
2. Может ли проекция параллелограмма при параллельном
проектировании быть квадратом?
Решение. Построим в плоскости α квадрат ABCD, проведем через прямую AB отличную от плоскости α плоскость β, и в этой плоскости построим не являющийся
квадратом параллелограмм ABC1 D1 (рис. 2.24). Четырехугольник CDD1 C1 — параллелограмм, поэтому прямые
DD1 и CC1 параллельны. Следовательно, при проектировании параллелограмма ABC1D1 на плоскость α параллельно
прямой h = CC1 получится как раз квадрат ABCD. (Наоборот, «пристраивая» по общей стороне к произвольному
параллелограмму лежащий в другой плоскости квадрат,
точно так же можно доказать, что параллельной проекцией
любого параллелограмма может быть квадрат.)
C1
β
D1

A

B
α

C

D

Рис. 2.24.

3. Дана параллельная проекция окружности и ее диаметра.
Как построить проекцию перпендикулярного диаметра?
Эта задача может оказаться трудной для учащихся,
поэтому ее условие можно снабдить наводящим вопросом:
каким свойством обладает диаметр, перпендикулярный
данному? Ответ: второй диаметр делит пополам любую
хорду, параллельную данному диаметру. Отсюда ясно
решение: проведем хорду CD, параллельную данному
диаметру AB, и найдем середину M этой хорды. Тогда
прямая OM, где O — центр окружности, являющийся

§ 17. Центральное проектирование

75

серединой диаметра, содержит перпендикулярный AB диаметр. Все эти построения осуществляются на изображении
(рис. 2.25).
A1

C1

O1

D1
B1

Рис. 2.25.

§ 17. ЦЕНТРАЛЬНОЕ ПРОЕКТИРОВАНИЕ

Центральное проектирование и связанные с ним вопросы
проективной геометрии, в частности теорема Дезарга,
предлагаются в учебнике факультативно для изучения
сильными учащимися. Можно в качестве метода ознакомления с данной темой предложить им написать реферат
«Теорема Дезарга и ее применение к решению задач из
курса школьной геометрии».
Целью исследования является изучение темы «Теорема
Дезарга и ее применение к решению задач из курса
школьной геометрии». В рамках достижения поставленной
цели необходимо поставить и решить следующие задачи.
1. Изучить теоретические аспекты основ проективной
геометрии.
2. Показать эффективность решения задач из курса
школьной геометрии с использованием теоремы Дезарга.
3. Определить класс задач, которые решаются с использованием теоремы Дезарга и основных свойств проективной
геометрии, и привести общие рекомендации для решения
этих задач.
Задачи для закрепления
1. Докажите, что существует проективное отображение,
которое три данные точки одной прямой переводит в три
данные точки другой прямой.

76

Глава 2. Параллельные прямые и плоскости

Решение. Обозначим данные прямые через l0 и l, данные
точки на прямой l0 — через A0 , B0 , C0 , данные точки
на прямой l — через A, B, C. Пусть l1 — произвольная
прямая, не проходящая через точку A. Возьмем произвольную точку O0 , не лежащую на прямых l0 и l1 .
Обозначим через P0 центральное проектирование прямой l0
на прямую l1 с центром в точке O0 , а через A1 , B1 ,
C1 — проекции точек A0 , B0 , C0 . Пусть l2 — произвольная
прямая, проходящая через точку A, не совпадающая
с прямой l и не проходящая через A1 . Возьмем некоторую
точку O1 на прямой AA1 и рассмотрим центральное
проектирование P1 прямой l1 на l2 с центром в O1 .
Обозначим через A2 , B2 , C2 проекции точек A1 , B1 ,
C1 . Ясно, что A2 совпадает с A. Наконец, пусть P2 —
проектирование прямой l2 на прямую l, которое в том
случае, когда прямые BB2 и CC2 не параллельны, является
центральным проектированием с центром в точке пересечения этих прямых, а в том случае, когда прямые BB2 и CC2
параллельны, является параллельным проектированием
вдоль одной из этих прямых. Композиция P2 ◦ P1 ◦ P0
является требуемым проективным преобразованием.
2. а) Даны прямые a, b, c, d, проходящие через одну точку,
и прямая l, через эту точку не проходящая. Пусть A,
B, C, D — точки пересечения прямой l с прямыми a,
b, c, d соответственно. Докажите, что (abcd) = (AB, CD),
sin(a, c) · sin(b, d)

где (abcd) =
. б) Докажите, что двойное
sin(b, c) · sin(a, d)
отношение четверки точек сохраняется при проективных
преобразованиях.
Решение. а) Обозначим точку пересечения четырех данных прямых через O; пусть H — проекция этой точки на
прямую l и h = OH. Тогда
2SOAC = OA · OC sin(a, c) = h · AC,
2SOBC = OB · OC sin(b, c) = h · BC,
2SOAD = OA · OD sin(a, d) = h · AD,
2SOBD = OB · OD sin(b, d) = h · BD.

§ 17. Центральное проектирование

77

Поделив первое равенство на второе, а третье — на четвертое, получаем
OA · sin(a, c) AC
,
=
BC
OB · sin(b, c)

OA · sin(a, d) AD
.
=
BD
OB · sin(b, d)

Деля получившиеся равенства, получаем |(AB,CD)|=|(abcd)|.
Для доказательства того, что числа (AB, CD) и (abcd)
имеют одинаковый знак, можно, например, выписать
все возможные способы расположения точек на прямой
(24 способа) и в каждом случае убедиться в том, что
(ABCD) положительно тогда и только тогда, когда пара
прямых a, b не разделяет пару прямых c, d. Результат
б) является непосредственным следствием задачи а).
3. Докажите, что если (AB, CX) = (AB, CY), то X = Y (все
точки попарно различны, кроме, быть может, точек X и Y,
и лежат на одной прямой).
Решение. Пусть a, b, c, x, y — координаты точек A, B,
C, X, Y. Тогда
x−a : c−a = y−a : c−a .
x−b c−b
y−b c−b

Следовательно, поскольку все точки
различны,
(x − a)(y − b) = (x − b)(y − a). Раскрывая скобки и приводя
подобные члены, получаем ax − bx = ay − by. Сокращая это
равенство на (a − b), получаем x = y.
4. Проекцией точки A из точки O на плоскость P
называется точка A′ , в которой прямая OA пересекает
плоскость P. Проекцией треугольника называется фигура,
состоящая из всех проекций его точек. Какими фигурами
может быть проекция треугольника, если точка O не лежит
в его плоскости?
Решение. Пусть ABC — данный треугольник. Рассмотрим
полный трехгранный угол OABC с вершиной O, состоящий
из двух трехгранных углов (ребрами одного угла являются
лучи OA, OB, OC, а ребрами другого — их продолжения).
Проекция треугольника ABC на плоскость P совпадает
с пересечением плоскости P и полного трехгранного
угла OABC. В зависимости от взаимного расположения
плоскости и трехгранного угла возникают следующие

78

Глава 2. Параллельные прямые и плоскости

варианты. 1. Плоскость P параллельна двум ребрам и пересекает третье. В проекции получается угол. 2. Плоскость P
параллельна одному ребру и пересекает два других, причем
оба — с одной стороны от вершины O. В проекции получается полоса, ограниченная двумя параллельными прямыми
и пересекающими их третьей прямой. 3. Плоскость P
параллельна одному ребру и пересекает два других, причем
по разные стороны от вершины O. В проекции получаются
два угла, у которых сторона одного служит продолжением
стороны другого, а две другие стороны параллельны
и противоположно направлены. 4. Плоскость P пересекает
все три ребра, причем все три — с одной стороны от
вершины O. В проекции получается треугольник. 5. Плоскость P пересекает все три ребра, причем два — с одной
стороны от вершины O, а одно — с другой. В проекции
получается фигура, состоящая из угла и бесконечной
фигуры, которая ограничена продолжениями сторон этого
угла и прямой, их пересекающей.
5. Докажите, что если плоскости α1 и α2 пересекаются,
то центральное проектирование α1 на α2 с центром O
задает взаимно однозначное отображение плоскости α1
с выкинутой прямой l1 на плоскость с выкинутой прямой l2 , где l1 и l2 — прямые пересечения плоскостей α1
и α2 соответственно с плоскостями, проходящими через O
и параллельными α2 и α1 . При этом на l1 отображение не
определено.
Решение. Прямые, проходящие через O и параллельные
плоскости α1 (соответственно α2 ), пересекают плоскость α2
(соответственно α1 ) в точках прямой l2 (соответственно l1 ).
Поэтому если точка лежит на одной из плоскостей α1 , α2
и не лежит на прямых l1 , l2 , то определено проектирование ее на другую плоскость. Ясно, что разные точки
проецируются в разные.
6. Докажите, что геометрическое место точек пересечения
диагоналей четырехугольников ABCD, у которых стороны
AB и CD лежат на двух данных прямых l1 и l2 , а стороны
BC и AD пересекаются в данной точке P, является прямой,
проходящей через точку Q пересечения прямых l1 и l2 .

§ 17. Центральное проектирование

79

Решение. Рассмотрим проективное преобразование, для
которого прямая PQ является исключительной. Образы l′1
и l′2 прямых l1 и l2 при этом преобразовании параллельны, а образами рассматриваемых четырехугольников
являются параллелограммы, у которых две стороны лежат
на прямых l′1 и l′2 , а две другие стороны параллельны
некоторой фиксированной прямой (бесконечно удаленная
точка этой прямой является образом точки P). Ясно,
что геометрическим местом точек пересечения диагоналей
таких параллелограммов является прямая, равноудаленная
от прямых l′1 и l′2 .
7. Пусть O — точка пересечения диагоналей четырехугольника ABCD, а E, F — точки пересечения продолжений
сторон AB и CD, BC и AD соответственно. Прямая EO
пересекает стороны AD и BC в точках K и L, а прямая FO
пересекает стороны AB и CD в точках M и N. Докажите,
что точка X пересечения прямых KN и LM лежит на
прямой EF.
Решение. Сделаем проективное преобразование с исключительной прямой EF. Тогда четырехугольник ABCD перейдет в параллелограмм, а прямые KL и MN — в прямые,
параллельные его сторонам и проходящие через точку
пересечения диагоналей, т. е. в средние линии. Поэтому
образы точек K, L, M, N являются серединами сторон
параллелограмма и, следовательно, образы прямых KN
и LM параллельны, т. е. точка X переходит в бесконечно
удаленную точку, а значит, X лежит на исключительной
прямой EF.
8. Прямые a, b, c пересекаются в одной точке O.
В треугольниках A1 B1 C1 и A2 B2 C2 вершины A1 и A2
лежат на прямой a; B1 и B2 — на прямой b; C1 и C2 — на
прямой c. A, B, C — точки пересечения прямых B1 C1
и B2 C2 , C1 A1 и C2 A2 , A1 B1 и A2 B2 соответственно.
Докажите, что точки A, B, C лежат на одной прямой
(Дезарг).
Решение. Сделаем проективное преобразование с исключительной прямой AB. Образы точек при этом преобразовании будем обозначать буквами со штрихом. Рассмотрим

80

Глава 2. Параллельные прямые и плоскости

гомотетию с центром в точке O′ (или параллельный перенос, если O′ — бесконечно удаленная точка), переводящую
точку C′1 в C′2 . При этой гомотетии отрезок B′1 C′1 перейдет
в отрезок B′2 C′2 , поскольку B′1 C′1 k B′2 C′2 . Аналогично C′1 A′1
перейдет в C′2 A′2 . Поэтому соответственные стороны треугольников A′1 B′1 C′1 и A′2 B′2 C′2 параллельны, т. е. все три
точки A′ , B′ , C′ лежат на бесконечно удаленной прямой.

Глава 3

ВЕКТОРЫ И КООРДИНАТЫ
В ПРОСТРАНСТВЕ

Приступая к теме «Векторы и координаты в пространстве»,
учитель должен будет сделать акцент на чрезвычайную
прикладную ее значимость. Мы имеем в виду традиционные приложения координат и векторов не только
к решению задач самой геометрии, но и к другим областям
математики и физики, что стало возможным благодаря
развитию ЭВМ и современных численных методов программирования.
Введение координат, рассмотрение векторов и их
координат дают возможность использования современной
вычислительной техники в самых различных геометрических с внешней точки зрения ситуациях.Именно
поэтому столь важно ознакомление в курсе средней школы
с координатами и векторами.
§ 18. ПОНЯТИЕ ВЕКТОРА В ПРОСТРАНСТВЕ.
ЛИНЕЙНЫЕ ОПЕРАЦИИ НАД ВЕКТОРАМИ
И СКАЛЯРНОЕ ПРОИЗВЕДЕНИЕ ВЕКТОРОВ

Мы рекомендуем теоретический материал дать относительно конспективно, рассмотреть понятие вектора в пространстве и связанные с ним понятия абсолютной величины, направления, равенства векторов; координат вектора; суммы векторов; произведения вектора на число;
коллинеарность векторов; скалярное произведение векторов и его свойства.
В пространстве, как и на плоскости, вектором называется направленный отрезок. Буквально так же, как и на
плоскости, определяются основные понятия для векторов
в пространстве: абсолютная величина вектора, направление
вектора, равенство векторов.

82

Глава 3. Векторы и координаты в пространстве

−−→
Под направленным отрезком AB понимается отрезок
AB, один из концов которого A называется началом,
а второй конец B — концом (направленного отрезка, т. е.
−−→
вектора AB). На чертеже обычно конец вектора помечается
стрелкой. Абсолютной величиной или модулем |AB| вектора
−−→
AB называется длина отрезка AB. Направление вектора
−−→
AB определяется направлением луча AB (с началом A).
При этом два луча считаются одинаково направленными
(имеющими одно и то же направление), если они совмещаются параллельным переносом, и противоположно
направленными, если каждый из них одинаково направлен
с лучом, дополнительным другому.
−−→
−−→
Два вектора AB и CD называются равными, если они
совмещаются некоторым параллельным переносом так,
что начало a совмещается с началом C, а конец B —
с концом D. Из этого определения очевидно, что равные
векторы одинаково направлены и равны по абсолютной
величине, причем верно и обратное: если векторы имеют
одно и то же направление и равные абсолютные величины,
то они равны.
Правило треугольника придает геометрический смысл
сумме векторов, дает геометрический, или графический,
способ сложения векторов безотносительно к координатам,
т. е. инвариантно по отношению к выбору системы
координат. Правда, координаты участвуют в определении
параллельного переноса, фигурирующего в определениях
равных векторов и одинаково направленных лучей, но
анализ свойств параллельного переноса дает возможность
установить, что и понятие параллельного переноса может
быть введено инвариантно, независимо от выбора декартовой системы координат. Эти замечания позволяют утверждать, что сумма векторов — направленных отрезков —
как направленный отрезок не зависит от выбора системы
~ ~b выражается через
координат. Скалярное произведение a
~
~
~
~
длины векторов a, b и a + b, а поэтому тоже не зависит
от выбора системы координат.
Произведением вектора a
~ на число p называется
вектор p~
a. Абсолютная величина вектора p~
a равна |p k a
~ |,

§ 18. Понятие вектора в пространстве

83

а направление совпадает с направлением вектора a
~ , если
p > 0, и противоположно направлению вектора a
~ , если p < 0.
Вопросы для закрепления
1. Что называется вектором в пространстве? Что такое
нулевой вектор? абсолютная величина вектора? направление вектора?
2. В каком случае векторы называются равными? Как
сформулировать условие равенства двух векторов через их
абсолютные величины и направления?
~ и ~b? Как
3. Как определяется сумма двух векторов a
записывается правило треугольника отыскания суммы
двух векторов?
4. Как определяется произведение вектора на число? Как
геометрически связаны между собой векторы a
~ и p~
a?
5. Могут ли векторы, лежащие на скрещивающихся прямых, быть коллинеарными?
6. Приведите примеры трех векторов:
а) лежащих на одной прямой;
б) лежащих в одной плоскости;
в) не лежащих в одной плоскости.
7. Могут ли быть коллинеарными векторы, лежащие на
противоположных ребрах тетраэдра?
Упражнения
−−→ −−→
1(18.1) Всегда ли верно, что если векторы AB и CD равны,
то ACDB — параллелограмм?
−−→ −−→
−−→
−−→
Решение. Если AB = CD и AB не совпадает с CD,
то из определения вектора отрезки AB и CD равны
и параллельны, тогда отрезки тоже равны и параллельны.
Следовательно, ABCD — параллелограмм.
~
2(18.2) Какому условию должны удовлетворять векторы a,
~b и ~c пространства, чтобы из них можно было образовать
треугольник?

84

Глава 3. Векторы и координаты в пространстве

~ ~b, ~c должны удовлетворять условиям
Решение. Векторы a,
~ < |~b| + |~c|, |~b| < |a|
~ + |~c|, |~c| < |a|
~ + |~b|.
|a|
~ и c~ пространства попарно не
3(18.4) Пусть векторы a
~, b
коллинеарны. Докажите, что:
~ + ~b| 6 |a|
~ + |~b|,
а) |a
~
~
~
~ + |b|
~ + |~c|.
б) |a + b + c| 6 |a|
Решение. Справедливость этих неравенств следует из
того, что длина отрезка прямой всегда меньше ломаной,
соединяющей концы этого отрезка.
4(18.5) Каким условиям должны удовлетворять векторы
~ ~b и ~c пространства, чтобы
a,
~ + ~b = a
~ − ~b;
а) a
~ 2 = |~
a|;
б) a
~
~
~
~
~
~
~ + |~b|;
г) |a + b| = |a|
в) |a + b| = |a − b|;
~ + ~b)~c = a
~ ~c − ~b~c;
~ + ~b + ~c| = |a|
~ + |b|
~ + |~c|?
е) |a
д) (a
О т в е т:
а) равенство верно всегда;
б) ~b = ~0;
~ и ~b ортогональны;
в) векторы a
~ и ~b сонаправлены;
г) векторы a
~
д) векторы b и ~c ортогональны, либо ~c = ~0;
~ ~b, ~c сонаправлены.
е) векторы a,
~ ⊥ ~b, ~b ⊥ ~c. Верно ли, что a
~ ⊥ ~c?
5(18.6) Пусть a
О т в е т: нет.
6(18.7) Используя формулу из задачи 16 в п. 100 учебника, докажите, что диагональ AC1 параллелепипеда
ABCDA1B1 C1 D1 проходит через точки пересечения медиан
треугольников A1 BD и D1 CB1 и делится этими точками
на три равных отрезка.
Доказательство.
Пусть
M — точка
пересечения
медиан треугольника A1 BD, d = AC, тогда, используя
формулу из задачи 16 п. 100 учебника, получаем
AM = (AB + AD + AA)/3 = AC1/3 = d/3. Аналогично, если
N — точка пересечения медиан треугольника D1 CB1 , то
C1 N = d/3; MN = d/3.
~ ~b и ~c параллельны одной
7(18.8) Докажите, что векторы a,
плоскости, если:

§ 18. Понятие вектора в пространстве

85

а) один из этих векторов нулевой;
б) два из этих векторов коллинеарны;
~ + β~b.
в) существуют такие числа α и β, что ~c = αa
Доказательство. а) Пусть ~c = 0, тогда можно представить
этот вектор как сумму противоположных векторов k~
a − k~
a.
~ и ~b, будет
Тогда плоскость, параллельная векторам a
параллельна и вектору ~c.
~ и ~b коллинеарны, тогда плоскость,
б) Пусть векторы a
~ и ~c, будет параллельна и векпараллельная векторам a
тору ~b.
~ ~b, ~c лежат в плоскости
~ + q~b, то векторы a,
в) Если ~c = pa
параллелограмма, сторонами которого являются векторы
~ и q~b, ~c — диагональ этого параллелограмма.
pa
8(18.9) Докажите правило параллелепипеда сложения трех
векторов в пространстве: направленная диагональ параллелепипеда, построенного на трех данных неколлинеарных
векторах, как на ребрах, является суммой этих векторов.
Доказательство.
Пусть
дан
параллелепипед
−−→
ABCDA1B1 C1 D1 , ребра которого являются векторами AB,
−−→ −−→
AD, AA1 , тогда по правилу параллелограмма сложения
векторов в плоскости основания параллелепипеда мы полу−−→ −−→ −−→
чим AC = AB + AD. Применяя то же правило для векторов
−−→
−−→
−−→
−−→ −−→ −−→
AC и CC1 , получим окончательно AC1 = AB + AD + AA1 .
9(18.10) Пусть векторы e~1 , e~2 и e~3 не параллельны
одной плоскости. Докажите, что для любого вектора a
~
в пространстве существуют числа x1 , x2 и x3 такие, что
a
~ = x1 e~1 + x2 e~2 + x3e~3 .
Доказательство. Если для любого произвольного вектора a
~ построить некоторый произвольный параллелепипед
−−→
~ то, исходя из
ABCDA1B1 C1 D1 с диагональю AC1 = a,
решения задачи 9, можно сделать вывод, что существует
разложение данного вектора в виде суммы трех векторов.
10(18.25) В
основании
четырехугольной
пирамиды
SABCD лежит прямоугольник ABCD. Докажите, что
SA2 + SC2 = SB2 + SD2 .

86

Глава 3. Векторы и координаты в пространстве

Доказательство. Пусть точка O — проекция вершины S пирамиды на плоскость основания, тогда
условие SA2 + SC2 = SB2 + SD2 эквивалентно условию
AO2 + CO2 = BO2 + DO2. Проектируя точку O на стороны
основания и применяя теорему Пифагора, получим тождество.
§ 19. КОМПЛАНАРНОСТЬ. БАЗИС И КООРДИНАТЫ
В ПРОСТРАНСТВЕ

Понятие базиса на плоскости и в пространстве является
фундаментальным в изучении основ векторной геометрии.
Важно заострить внимание учащихся на том, что любая
пара неколлинеарных векторов плоскости e~1 ; e~2 задает
ее базис и любой вектор плоскости (в том числе
и базисный) можно по этому базису разложить. К примеру,
e~1 = 1~
e1 + 0~
e2 . Полезно отметить, что базисный вектор не
может быть нулевым, поскольку нулевой вектор коллинеарен любому вектору. Неплохо проиллюстрировать разло~ плоскости по выбранному
жение произвольного вектора a
базису e~1 ; e~2 чертежом. Для этого
~e2
следует все три вектора e~1 , e~2 и a
~
привести к одному началу, которым
B может служить произвольная точка O
x2~e2
~a
плоскости (рис. 3.1) и, используя
A
O
~e1 правило параллелограмма, по котоx1~e1
−−→ −−→
рому a
~ = OA + OB, показать, что сущеРис. 3.1.
ствуют числа x1 ; x2 , удовлетворяющие
−−→
−−→
равенствам OA = x1e~1 ; OB = x2e~2 . Такая наглядность в представлении базиса плоскости облегчит переход к пониманию
базиса пространства и свойства компланарности векторов.
Опять же с помощью чертежа надо показать принцип раз~ пространства по трем базисным
ложения любого вектора a
векторам e~1 ; e~2 ; e~3 . Для этого представить вектор a
~ в виде
диагонали параллелепипеда (лучше не прямоугольного),
двенадцать ребер которого задаются тройкой векторов
e~1 ; e~2 ; e~3 . При разъяснении свойства компланарности
векторов важно сделать акцент на том, что, в соответствии
с определением, хотя бы один из компланарных векторов

§ 19. Компланарность

87

должен выражаться через остальные. Именно «хотя бы»
~ ~b; ~c,
один, а не любой. К примеру, на рис. 3.2 векторы a;
приведенные к одному началу в точке O, компланарны,
~ ~b, так как a
~ k ~b,
но вектор ~c невозможно выразить через a;
~ + 0~c.
в то время как ~b = −2a
Имеет смысл также обратить внимание учащихся на то, что бази~c
сов плоскости и пространства существует сколько угодно, но каждый из
них должен состоять соответственно ~
O
~a
из двух неколлинеарных и из трех b
некомпланарных векторов. Перед тем,
Рис. 3.2.
как перейти к вопросу о координатах
вектора в заданном базисе, надо обязательно остановиться
на таком важном свойстве базиса, как упорядоченность
системы векторов его составляющих. Благодаря этому свойству координатная запись вектора позволяет однозначно
перейти к разложению данного вектора по базису. Важно
отметить, что любая перестановка в тройке базисных
векторов приводит к созданию нового базиса. Пусть,
~ ~b; ~c не компланарны, т. е. могут
к примеру, векторы a;
~ ~b; ~c и ~c; a;
~ ~b —
составить базис пространства. Тогда a;
два разных базиса. В первом для вектора p
~ = (x, y, z) имеет
~ + yb
~ + z~b.
~ + z~c, во втором ~p = x~c + ya
место разложение ~p = xa
Для решения задачи 12 к § 19 полезно четко сформулировать определение линейно зависимой (линейно независимой) системы векторов. Научить школьника давать
лаконичные и грамотные определения математическим
языком — большая заслуга преподавателя. В данном случае это позволит перейти к общепринятому в курсе
высшей математики определению базиса как упорядоченной линейно независимой максимально полной системе
векторов рассматриваемого пространства.

Вопросы для закрепления
1. Как определяются координаты точки в пространстве
(в данной системе координат)?

88

Глава 3. Векторы и координаты в пространстве

2. Для любой ли тройки чисел {x, y, z} существует точка
с этими координатами? Как ее построить? (Более трудный
вопрос: могут ли две различные точки иметь одинаковые соответственные координаты? Его можно дать как
задачу.)
3. Чему равны координаты середины отрезка с данными
концами? (Ответ: полусуммам соответственных координат
концов отрезка.)
4. Как по координатам четырех точек A, B, C и D
установить, является ли четырехугольник ABCD параллелограммом?
Задачи для закрепления
1. В тетраэдре ABCD точки M и N являются соответственно точками пересечения медиан граней ADB и BDC.
Докажите, что MN k AC и найдите отношение длин этих
отрезков (рис. 3.3).
A

M
B
C

N

E
D

Рис. 3.3.

Доказательство. M — точка пересечения медиан тре−−−→
−−−→
угольника; ABD ⇒ AM = 2ME;
N — точка пересечения медиан треугольника BCD ⇒
−−→
−−→
⇒ CN = 2NE;
−−−→ ~
−→
~ = 1 eC
~ − 1 eA
~ = 1 (eC
~ − eA)
~ =1 −
MN = eN − eM
AC;
3
3
3
3


−−−→ 1 −−→
MN
1
MN = AC ⇒ MN k AC;
= .
3

AC

3

§ 19. Компланарность

89

2. ABCD — правильный тетраэдр, в котором AK = KD,
CL = LB. Отрезок KL — средняя линия тетраэдра. Дока−−→ 1 −−→ −−→
жите, что KL = (AC + DB).
2

Доказательство.
−−→ −−→ −−→ −−→
KL = KA + AC + CL,
−−→ −−→ −−→ −−→
KL = KD + DB + BL,
−−→ −−→ ~
−−→ −−→
KA = KD = 0, CL + BL = ~0

9
?
?
?
?
?
=


?
?
?
?
?
;


−−→ −−→ −−→ −−→
−−→ −−→
⇒ 2KL = AC + DB ⇒ KL = 1 AC + DB .
2

3. Докажите, что все средние линии тетраэдра пересекаются в одной точке и делятся этой точкой пополам.
−−→ −
−−→ 1 −−→
Доказательство. KN = ML = DC.
2
Пусть O — середина отрезка KL. Докажем, что O —
середина отрезка MN (рис. 3.4).
D
K

M
O

A
D1

N

B
L

C

Рис. 3.4.

−−−→ −
−−→ −−→ 1 −−→ −−→
MO = ML + LO = DC + LO,
2
−−→ −−→ −−→ −−→ 1 −−→
ON = OK + KN = OK + DC,
2
−−→ −−→
OK = LO

9
?
?
?
?
?
?
= −−−→ −−→
⇒ MO = ON.
?
?
?
?
?
?
;

То есть точка O — середина отрезка MN.
Аналогично это утверждение доказывается и для отрезка
PQ, где P — середина отрезка DC, Q — середина отрезка
AB.

90

Глава 3. Векторы и координаты в пространстве

§ 20. ПРЯМОУГОЛЬНЫЕ КООРДИНАТЫ В ПРОСТРАНСТВЕ

Данный параграф имеет очень важное практическое содержание. В нем приведены фактически все формулы, необходимые для решения задач методами векторной алгебры
и аналитической геометрии. Они составляют главный «рабочий инструмент» школьника в этой науке, поэтому изучению и тщательному освоению данного материала надо
уделить особое внимание. Учащиеся должны не только разобраться в теории, но и хорошенько запомнить все формулы,
приведенные в учебнике. Этому может способствовать проведение письменных теоретических работ, как, например,
это делается для усвоения формул тригонометрии.
Вопросы для закрепления
1. Как вводится декартова система координат в пространстве? Из чего она состоит? (Ответ: из начала координат,
координатных осей и плоскостей.)
2. Укажите координаты проекций (оснований перпендикуляров, проведенных из данной точки A(x, y, z) на координатные оси x, y, z и плоскости xy, yz, xz).
3. Чему равно расстояние от начала координат до данной
точки A(x, y, z)?
4. Что такое скалярное произведение двух векторов? Перечислите его основные свойства.
5. Что такое скалярный квадрат? По каким формулам
вычисляются скалярные квадраты суммы векторов? разности векторов?
6. Каков геометрический смысл скалярного произведения
векторов? Что такое угол между двумя векторами? (В ответе
нужно уточнить: между ненулевыми векторами.)
7. Что означает перпендикулярность векторов? Каким
образом можно установить перпендикулярность векторов?
найти угол между векторами?
Упражнения
1(20.8) В тетраэдре ABCD плоские углы при вершине D
прямые. Докажите, что углы между биссектрисами DD1 ,
DB1 и DC1 этих углов, взятыми попарно, равны 60◦ .

§ 20. Прямоугольные координаты в пространстве

91

Доказательство. Пусть D — начало прямоугольной
системы координат. Оси координат совпадают с боковыми
ребрами тетраэдра, исходящими из вершины D, тогда
−−→
−−→
−−→
координаты векторов DA1 и DB1 будут: DA1 (a/2, 0, a/2),
−−→ −−→
DA1 · DB1
−−→
DB1 (b/2, b/2, 0), угол ∠A1 DB1 = α; cos α =
= 1/2;
|DA1 | · |DB1 |

α = 60◦ . Аналогичный результат мы получим при рассмотрении других пар биссектрис.

2(20.9) Докажите, что одна из вершин куба и центры трех
граней куба, проходящих через эту вершину, являются
вершинами правильного тетраэдра.
Доказательство. Пусть длина стороны квадрата равна
a, точки √M, N, K — центры граней,
тогда AC = AD√1 =

= D1 C = a 2, MN = NK = MK = a 2; DK = DM = DN =√
a 2.
Следовательно, все ребра тетраэдра DMKN равны a 2.
3(20.10) В параллелепипеде ABCDA1B1 C1 D1 плоские углы
при вершине A равны. Из точки A проведены биссектрисы соответствующих плоских углов, которые пересекают диагонали граней параллелепипеда A1 B, BD и DA1
в точках E1 , E2 и E3 соответственно. Докажите, что
если тетраэдр AE1 E2 E3 — правильный, то параллелепипед ABCDA1B1 C1 D1 — куб.
Указание. Эту задачу можно рассматривать как обратную
к задаче 9.
4(20.12) Параллельны ли следующие прямые:
а) x = 1 + 2t, y = 1 − t, z = t и x = −3 − 2t, y = t, z = −t;
б) x = 2 − t, y = 1 + 2t, z = 1 и

в)

x−1
z+2
=y+5=
;
2
−3

x+1
y−3
z+1
=
=
и x = −2t, y = 1 − 5t, z = −3 + 3t?
2
5
−3

Решение. Рассматривая направляющие векторы прямых,
можно ответить на вопрос о параллельности прямых.
~ = {2; −1; 1}, ~b = {−2; 1; −1}, т. е. a
~ = −~b; следоваа) a
тельно, прямые параллельны.
~ = {−1; 2; 0}, ~b = {2; 1; −3}, следовательно, прямые не
б) a
параллельны.

92

Глава 3. Векторы и координаты в пространстве

~ = {2; 5; −3}, ~b = {−2; −5; 3}, т. е. a
~ = −~b; следовав) a
тельно, прямые параллельны.
5(20.13) Составьте уравнение прямой, проходящей через
две точки A и B:
а) A(1; 1; 2), B(−1; 2; 3);
б) A(−1; 3; 4), B(2; 1; −3);
в) A(0; 1; −1), B(1; 1; 2).
Решение. Составим уравнения прямых, воспользовавшись
каноническим уравнением прямой и уравнением прямой
в параметрической форме из п. 120.
x−1
= (y − 1) = (z − 2);
−2
x+1
y−3
z−4
б)
=
=
;
3
−2
−7

а)

в) x = t; y = 1; z = −1 + 3t.
6(20.15) Найдите углы между прямыми:
а)

x−1
y+2
z−3
x+2
y+1
z−2
=
=
и
=
=
;
2
3
−1
−1
2
3
8
8

>
>
< x = 1+t
б) > y = 1 − t
>
: z = 2t

>
>
< x = 2−t
и > y = 3t
>
: z = −1 + t.

Решение. Угол между прямыми равен углу между их
направляющими.
~ = {2; 3; −1}, ~b = {−1; 2; 3};
а) a
−2 + 6 − 3

1

= ;
cos α =
14
14
α = arccos (1/14);
~ = {1; −1; −2}, ~b = {−1; 3; 1};
б) a
~ · ~b
a
2
=− √ ;
~
~
66
|a| · |b|

−2
α = arccos √
.
66

cos(α) =

Глава 4

ПЕРПЕНДИКУЛЯРНОСТЬ ПРЯМОЙ
И ПЛОСКОСТИ

§ 21. ПРИЗНАК ПЕРПЕНДИКУЛЯРНОСТИ ПРЯМОЙ
И ПЛОСКОСТИ

В процессе освоения теоретического материала § 21 важно
акцентировать внимание учащихся на достаточности критерия перпендикулярности прямой и плоскости. Полезно
в качестве примера рассмотреть векторное доказательство этого признака. Вообще говоря, привлечение векторного «инструментария» во многих случаях позволяет
школьнику, не обладающему развитым пространственным
воображением, успешно решать многие стереометрические
задачи. Поэтому, наряду с геометрическими построениями, ниже рассматриваются приемы решения задач
средствами векторной алгебры и аналитической геометрии.
В частности, векторные решения задач 21.17 и 21.21
предвосхищают использование теоремы о трех перпендикулярах, которая изучается позже в § 22 учебника,
а задачи 11–13 относятся непосредственно к категории
задач аналитической геометрии.

Упражнения и задачи для закрепления
1(21.4). Дан куб ABCDA1B1 C1 D1 . Установите, является ли
прямая BD перпендикулярной к плоскости ACC1 .
О т в е т: является.
2(21.5). Можно ли провести:
а) прямую, перпендикулярную одновременно данной
плоскости и пересекающей эту плоскость прямой?
б) прямую, перпендикулярную двум пересекающимся
плоскостям?

94

Глава 4. Перпендикулярность прямой и плоскости

Решение. а) Пусть прямая l1 (рис. 4.1) перпендикулярна
плоскости α и прямой l2 , пересекающей эту плоскость,
~ ~b — единичные базисные векторы плоскости α; ~c, d
~—
a,
единичные направляющие векторы соответственно пря~ ~b, d
~ образуют пространственмых l1 и l2 . Векторы a,
ный базис, в котором вектор ~c можно представить:
~c = λa
~ + ν~b + µd.
~ Умножим это равенство скалярно на ~c,
~ · ~c + ν~b · ~c + µd
~ · ~c. В силу перпендикулярполучим ~c · ~c = λa
~
~
~
~
~
ности векторов a · c = b · c = d · ~c = 0. Отсюда ~c = ~0. Это противоречит условию.
l1

~c

l2

~
d

~b
~a

α

Рис. 4.1.

О т в е т: нельзя.
~1 и n
~ 2 — соответственно нормальные векторы
б) Пусть n
пересекающихся плоскостей α и β, a
~ — направляющий
вектор прямой l, которая перпендикулярна обеим плоскостям. Тогда найдутся не равные нулю числа λ1 и λ2 , для
которых a
~ = λ1 · n
~1 и a
~ = λ2 · n
~ 2 . Отсюда можно заключить,
что n
~2 =

λ1
n
~ , т. е. векторы n
~1 и n
~ 2 коллинеарны. Но это
λ2 2

означает параллельность плоскостей α и β.
О т в е т: нельзя.
3. Используя векторы, докажите признак перпендикулярности прямой и плоскости.

Решение. Пусть дана плоскость α (рис. 4.2) и прямые
a, b, c, d, причем прямые b, c лежат в плоскости α
и пересекаются (b ∩ c = O). Прямая a перпендикулярна
прямым b и c (a ⊥ b, a ⊥ c).
О прямой d известно лишь, что она лежит в плоскости α.
~ ~b, ~c, d
~ для всех четырех
Введем направляющие векторы a,
прямых.

§ 21. Признак перпендикулярности прямой и плоскости

95

~a
~
d

d

b
cO

α

~c
a

~b

Рис. 4.2.

Поскольку прямая b не параллельна прямой c, векторы
~b, ~c задают базис плоскости α. Имеет место разложение
~ по этому базису в виде d
~ = xa
~ + y~b. Умножим
вектора d
~
~
~ = xa
~ · ~b + ya
~ · ~c.
последнее равенство скалярно на a: a · d
Учитывая, что вектор a
~ по условию перпендикулярен
~ ·d
~ = 0, но это означает,
как ~b, так и ~c, получим a
что прямая a перпендикулярна произвольной прямой d,
лежащей в плоскости α.
4. Все грани параллелепипеда ABCDA1B1 C1 D1 — равные
ромбы (рис. 4.3). Углы между ребрами, содержащими
точку A, равны. Выяснить, будет ли прямая A1 C перпендикулярна прямой D1 B1 .
D1

C1
O

A1
B1

~c
D
C

~b
A

~a

B

Рис. 4.3.

Решение. Способ 1. Треугольники AA1 D1 и AA1 B1
равны. Значит, AB1 = AD1. Пусть O — середина отрезка
B1 D1 . Тогда прямая AO в равнобедренном треугольнике
D1 AB1 перпендикулярна B1 D1 . Аналогично можно доказать, что прямая CO перпендикулярна B1 D1 . Значит,

96

Глава 4. Перпендикулярность прямой и плоскости

прямая B1 D1 перпендикулярна плоскости AA1 C1 C и, как
следствие, прямой A1 C.
Способ 2. Не нарушая общности рассуждений, примем длины всех ребер параллелепипеда за 1. Пусть
~
~b, ~c — соответственно единичные векторы ребер
a,
−−−→
−−→
−−−→ −−−→
~, −
AB, AD, AA1 . D1 B1 = ~a − b
A1 C = ~a + ~b − ~c, D1 B1 · A1 C =
~2 + a
~ · ~b − a
~ · ~c − a
~ · ~b − ~b2 + ~b · ~c. Так как a
~ 2 = ~b2 , a
~ · ~c = ~b · ~c,
=a
−−−→ −−−→
−−−→
−−−→
то D1 B1 · A1 C = 0 и D1 B1 ⊥ A1 C.
О т в е т: A1 C ⊥ D1B1 .
5(21.16). Дана правильная четырехугольная пирамида
SABCD с основанием ABCD, причем ее боковые грани —
остроугольные треугольники. Постройте сечение пирамиды
плоскостью, проходящей через точку A и перпендикулярной ребру SB.
Решение. В треугольнике ASB из точки A проведем
перпендикуляр AM к стороне SB. В силу равенства
треугольников ASB и CSB перпендикуляр, восстановленный из точки M в плоскости треугольника SBC,
проходит через точку C (рис. 4.4). Таким образом, ребро SB
перпендикулярно двум пересекающимся прямым MC и MA
плоскости сечения ACM. Отсюда ACM — искомое сечение.
S

M
C

D

B

A

Рис. 4.4.

6(21.12). Докажите, что если все диагонали параллелепипеда равны, то этот параллелепипед прямоугольный.

§ 21. Признак перпендикулярности прямой и плоскости

97

Решение. Рассмотрим диагональное сечение ACC1 A1
параллелепипеда ABCDA1B1 C1 D1 (рис. 4.5). Поскольку
диагонали параллелограмма ACC1 A1 равны, он является
прямоугольником и AA1 ⊥ AC. Аналогично DD1 ⊥ BD, но
AA1 k DD1 , поэтому AA1 ⊥ BD.
D1

C1
B1

A1

D

A

C
B

Рис. 4.5.

Итак, AA1 ⊥ AC, AA1 ⊥ BD, т. е. прямая AA1 перпендикулярна плоскости ABCD, откуда следует, что AA1 ⊥ AB,
AA1 ⊥ AD.
Точно так же можно показать, что ребро AB перпендикулярно плоскости ADD1 A1 , поэтому данный параллелепипед является прямоугольным.
7(21.13). Все диагонали параллелепипеда равны, а одна из
них, выходящая из некоторой вершины параллелепипеда,
перпендикулярна плоскости, которая содержит концы трех
ребер, выходящих из этой же вершины. Докажите, что
данный параллелепипед является кубом.
Решение. Пусть в параллелепипеде ABCDA1B1 C1 D1 диагональ AC1 перпендикулярна плоскости A1 BD и диагонали
параллелепипеда равны.
Из предыдущей задачи следует, что этот параллелепипед прямоугольный. Введем систему координат, как это
показано на рис. 4.6. Для краткости изложения обозна−−→
−−→
−−→
−−→
~ Тогда d~ = ~a + b
~ + ~c,
чим AB = ~a, AD = ~b, AA1 = ~c, AC1 = d.
~ −b
~ = d~ · b
~ ), d~ ⊥ (b
~ − ~c) ⇒ d~ · a
~ = d~ · ~c. Умножим равенство
d~ ⊥ (a
~
~
~ потом на b
~
~
~ и ~c, учтем
d = a + b + c скалярно сначала на a,

98

Глава 4. Перпендикулярность прямой и плоскости
z

C1

B1

A1
D1

~
d
x

~c

C

B

~a
y

A

~b

D

Рис. 4.6.

~ b,
~ ~c попарно ортогональны.
при этом, что векторы a,
2
2
~ = a~ , d~ · b
~ = b~ , d~ · ~c = c~ 2 .
Получим d~ · a
2
2
2
Отсюда a~ = b~ = c~ , значит, ABCDA1B1 C1 D1 — куб.
8(21.17). Постройте сечение куба плоскостью, проходящей
через середину одной из его диагоналей и перпендикулярной этой диагонали.
Решение. В плоскости BDD1 (рис. 4.7) через середину O
прямой BD1 проведем прямую LN, перпендикулярную
BD1 .
z

D1

C1
N
F

A1

B1

H

P

O
K
T
A

y

D

C

E

L

B

x

Рис. 4.7.

Через точки L, O, N проведем прямые TE, KP,
HF, параллельные A1 C1 , построим сечение TKHFPE,
параллельное A1 C1 .
Покажем, что оно является искомым сечением. Действительно, по построению прямая BD1 перпендикулярна

§ 21. Признак перпендикулярности прямой и плоскости

99

LN. Перпендикулярность прямых BD1 и KP докажем,
введя систему координат с центром в точке A. Примем
−−→
−−→
длину ребра куба за 1. Тогда BD1 = (1; 1; 1), KP = (1; 1; 0),
−−→
−−→ −−→
−−→
BD1 · KP = −1 + 1 + 0 = 0. Следовательно, BD1 ⊥ KP. Таким
образом, по признаку перпендикулярности прямой и плоскости TKHFPE — искомое сечение.
9(21.19). В правильной треугольной пирамиде через сторону основания и середину противоположного бокового
ребра проведена плоскость, которая оказалась перпендикулярной этому ребру. Найдите высоту пирамиды, если
длина стороны основания пирамиды равна 1.
Решение. Введем систему координат, как это показано
AB = a, TE = h. Тогда
на рис.√ 4.8. Введем обозначения:



a 3
a 3
a
h
; 0 , T 0;
; h , L 0; √ ;
; A
2
6
3 2

−−→
a
BT = 0; − √ ; h .
3

B 0;

z

a
; 0; 0 ,
2

T

L
y
B
C

E
O
A

x

Рис. 4.8.

Уравнение плоскости,
проходящей через L и AC:

a
h
y− √ +h z−
= 0.

a
−√
3

2

3

Так как координаты точки А удовлетворяют
данному
r
a2

h2

2

уравнению, получим

= 0, h = a
.
3
2
3
r
О т в е т: a

2
.
3

100

Глава 4. Перпендикулярность прямой и плоскости

10(21.21), Мат. ф-т МГПУ. В прямоугольном параллелепипеде AB = BC = a; AA1 = b. Найдите площадь сечения,
проходящего через вершину A и перпендикулярного диагонали BD1 .
Решение. Введем систему координат, как это показано на
рис. 4.9. В этой системе координат: A(0; 0; 0), B(a; 0; 0),
D1 (0; a; b), C(a; a; 0).
B∗
z

D1

C1
L

A1

B1

N
y
D

C
O

A

x

B

Рис. 4.9.

−−→
Отсюда BD1 = (−a; a; b).
Уравнение плоскости сечения: −ax + ay + bz = 0.
Очевидно, что координаты точки C удовлетворяют этому
уравнению: −a · a + a · a + b · 0 = 0.
Следовательно, основание параллелепипеда пересекается плоскостью сечения по прямой AC. Обозначим B∗ точку пересечения плоскости сечения с прямой BB1, O — середину отрезка AC. Пусть B∗ (a; 0; z∗ ).
Тогда из уравнения секущей плоскости вытекает, что
a2

−a · a + a · 0 + b · z∗ = 0 ⇒ z∗ =
.
b
2
a
Иначе z∗ = b ·
. Если a 6 b,
b

то

сечение

имеет



форму треугольника AB C. Если a > b, то сечением
является трапеция ANLC. Так как AB = BC, то из
равенства △ABB∗ = △CBB∗ следует, что △AB∗ C —
равнобедренный, а значит, B∗ O — его высота. Из
прямоугольного треугольника B∗s
BO найдем гипотенузу
по
r

2
2 2
2 + 2a2
p
a
a
a
b
+
=
.
формуле B∗ O = BO2 + (BB∗)2 =
2

b

b

2

§ 21. Признак перпендикулярности прямой и плоскости 101
1 √
a
SAB∗ C = a 2 · ·
2
b

дения

SANLC

p

p
b2 + 2a2
a2 b2 + 2a2

=
.
2b
2

учтем,
a2

SNB∗ L
(B B∗ )2
= 1 ∗2 =
SAB∗ C
(BB )

чим
= SAB∗ C

b

что

2
−b

a2
b

2

Для

нахож-

△NB∗ L ∼ △AB∗C,

поэтому

. После преобразований полу-

2
b2
SNB∗L = SAB∗ C · 1 − 2 ; SANLC = SAB∗ C − SNB∗ L =
ap
p

2
b
a2 b2 + 2a2 b4
b3 b2 + 2a2
1 − 1 + 2 , SANLC =
· 4=
.
2
2b

a

a

2a

p

О т в е т: при a 6 b площадь сечения равна
a > b площадь сечения равна

p
b3 b2 + 2a2
2a2

a2 b2 + 2a2
, при
2b

.

11. Составьте уравнение плоскости, проходящей через
точку A(1; 2 : 3) и перпендикулярной:
а) оси OX; б) оси OY; в) оси OZ; г) прямой OA.
Решение. а) Нормальный вектор искомой плоскости
коллинеарен вектору ~i = (1; 0; 0), поэтому 1(x − 1) +
+ 0(y − 2) + 0(z − 3) = 0 или x − 1 = 0, x = 1. Аналогичное
решение для случаев б) и в).
О т в е т: а) x = 1; б) y = 2; в) z = 3; г) 1(x − 1) + 2(y − 2) +
+ 3(z − 3) = 0, x + 2y + 3z − 14 = 0.

12. Плоскость x + y + z = 1 пересекает оси координат в точках A, B и C. Напишите уравнение высоты тетраэдра
OABC, проходящей через точку O.

~ данной плоскости имеет
Решение. Нормальный вектор n
~
координаты n = (1; 1; 1) и является направляющим вектором искомой прямой, поэтому уравнение этой прямой
x
y
z
имеет вид = = или x = y = z.
1

1

1

О т в е т: x = y = z.
13. Найдите уравнение прямой, симметричной прямой
6x = 3y = 2z относительно плоскости x + y + z − 6 = 0.

102

Глава 4. Перпендикулярность прямой и плоскости

Решение. Для решения достаточно найти пару точек
пространства, симметричных двум точкам заданной прямой относительно плоскости x + y + z − 6 = 0. Выберем
в качестве этих двух точек начало координат O(0; 0; 0)
и точку M пересечения прямой 6x = 3y = 2z с плоскостью x + y + z − 6 = 0. Прямая, перпендикулярная данной
плоскости и проходящая через начало координат, имеет
уравнение x = y = z и пересекает плоскость x + y + z − 6 = 0
в точке A(2; 2; 2). Тогда точка O1 , симметричная точке
O(0 : 0; 0) относительно плоскости x + y + z − 6 = 0, имеет
−−→ −−→
координаты (4; 4; 4), поскольку OA = AO1 .
Подставив найденные из уравнения 6x = 3y = 2z переменные y = 2x, z = 3x в уравнение x + y + z − 6 = 0, получим
6x − 6 = 0, т. е. x = 1; y = 2x = 2, z = 3x = 3. Итак, прямая 6x = 3y = 2z пересекает данную плоскость в точке
M(1; 2; 3), поэтому прямая, симметричная этой прямой
относительно плоскости x + y + z − 6 = 0, проходит через
точки M(1; 2; 3) и O1 (4 : 4; 4). Так как направляющий
−−−→
вектор этой прямой MO = (3; 2; 1), то можно составить
ее уравнение:
О т в е т:

x−1
y−1
=
= z − 3.
3
2

x−1
y−1
=
= z − 3.
3
2

14(21.22). Точка P равноудалена от вершин A и C1 куба
ABCDA1B1 C1 D1 и середин A2 и B2 его ребер AA1 и BB1.
Найдите длину отрезка AP, если длина ребра куба равна 1.
Решение. Введем систему координат по аналогии с задачей 8(21.17). Поскольку точка P одинаково удалена от A
и от C1 , она лежит в плоскости α, перпендикулярной AC1 ,
проходящей через середину O отрезка AC1 .
−−−→
Так как AC1 = (1; 1; 1) и O(0,5; 0,5; 0,5), то уравнение
плоскости α имеет вид: 1(x − 0,5) + 1(y − 0,5) + 1(z − 0,5) = 0
или x + y + z = 1,5.
В то же время координаты точки P(x, y, z) должны
удовлетворять уравнению плоскости x = 0,5, проходящей
через середину отрезка, соединяющего точки A2 и B2 ,
и перпендикулярной прямой A2 B2 . Учтем также равенство
PA = PA2, т. е. x2 + y2 + z2 = x2 + y2 + (z − 0,5)2 . Таким обра-

§ 22. Перпендикуляр и наклонная 103

зом, координаты
точки P должны удовлетворять системе
8
>
x
+
y
+ z = 1,5,
>
<
x = 0,5,
Отсюда находим координаты
уравнений >
> 2
:
z = (z − 0,5)2 .
искомой точки P: x = 0,5; y = 0,75; z = 0,25. Следовательно,
r
p
7
2
2
2
длина отрезка AP составляет x + y + z =
.
8
r
7
.
8

О т в е т:

§ 22. ПЕРПЕНДИКУЛЯР И НАКЛОННАЯ.
УГОЛ МЕЖДУ ПРЯМОЙ И ПЛОСКОСТЬЮ

Одной из важных тем данного параграфа является «Уравнение плоскости в пространстве» и связанные с ней понятия.
Здесь для поиска наиболее рациональных решений следует
обратить внимание учащихся на то, что плоскость задается
~ и любой своей точлюбым своим нормальным вектором n
кой M0 . Расстояние ρ от произвольной точки пространства
M∗ до заданной плоскости α есть абсолютная величина
−−−−−→
~
проекции вектора M0 M∗ на любой нормальный вектор n
этой плоскости.
−−−−−→
~ · M M∗
n






0
,
Согласно формуле ρ(M∗ , α) = Пр~ M0 M∗ =
n

~
n

величина ρ не изменится при инверсии направления
нормального вектора или при изменении его длины.
В расчетах в качестве M0 надо выбирать наиболее удобную
по координатной записи точку.
Поясним сказанное на примере.
Задача.
В
прямоугольном
параллелепипеде
ABCDA1B1 C1 D1 точка P — середина ребра CC1 , AB = a;
AD = b; AA1 = c. Найдите расстояние от точки A до
плоскости BA1 P.
Решение. Введем систему координат, как это показано на рис. 4.10. Тогда A(0; 0; 0), B(a; 0; 0), A1 (0; 0; c),

c
P a; b;
. Среди множества векторов, нормальных плос2

~ = (m; l; 1). Найдем m
кости BA1 P , выделим вектор n

104

Глава 4. Перпендикулярность прямой и плоскости
z

D1

C1

A1

B1
P
α

y

D

C

A

x

B

Рис. 4.10.


−−→ ~
−−→
c
~
~
и l из условий n ⊥ BP; n ⊥ BA1. Имеем: BP = 0; b;
;
2

−−−→
BA1 = (−a; 0; c).
c
l·b+ =0
2

Тогда

−→
~ ·−
n
BP = 0;

−m · a + c = 0.

и

−−→
~ ·−
n
BA1 = 0.

Отсюда

Поэтому


~= c ;− c ;1 .
n
a
2b

−−→ −−→
Используя формулы ρ(A, α) = Прn~ AB ; AB = (a; 0; 0), получим ρ(A, α) =

−→
~ ·−
n
AB
~
n

c

=s
1+

c

О т в е т: s
1+

c2
a2

c2
a2

.
+

c2
4b2

.
+

c2
4b2

Отметим, что и остальные теоретические положения
данного параграфа (угол между прямой и плоскостью и его
свойство, теорема о трех перпендикулярах) играют первостепенную роль в практике решения задач. Это наглядно
демонстрируют приведенные ниже решения. Добавим, что
задачи под номерами 1–9 фактически являются опорными
для многих других, а утверждения, сформулированные
в задачах 3–6, будут востребованы и в дальнейшем при
изучении темы «Комбинации многогранников и круглых
тел».
Упражнения и задачи для закрепления
1. Доказать теорему о трех перпендикулярах, используя
векторы.

§ 22. Перпендикуляр и наклонная 105

Решение. Докажем, что наклонная перпендикулярна прямой, лежащей в плоскости, тогда и только тогда, когда
этой прямой перпендикулярна проекция наклонной.
Пусть HB — проекция наклонной AB на плоскость α, e~ — направляющий вектор прямой l ∈ α (рис. 4.11).
−−→
−−→
−−→ −−→ −−→
Поскольку BH + HA + AB = ~0, имеем ~e · BH + ~e · HA +
−−→
−−→
−−→
−−→
+ ~e · AB = 0. Так как ~e · HA = 0, то ~e · BH = −~e · AB.
A

H

~e

l

B

Рис. 4.11.

Следовательно, l ⊥ BH ⇔ l ⊥ AB.
2(22.4). Докажите, что для расстояний ρ от концов отрезка
AB до плоскости α, проходящей через точку M, для которой
|AM| : |MB| = λ, верна формула ρ(A, α) = λ · ρ(B, α).

3(22.5). Найдите следующие геометрические места точек:
а) оснований наклонных одинаковой длины, проведенных из данной точки к данной плоскости;
б) точек, удаленных на одинаковое расстояние от данной плоскости.
О т в е т: а) окружность, б) плоскость, параллельная данной плоскости.
4(22.7). Докажите, что если в пирамиде все ребра наклонены под одним углом к основанию, то около основания
можно описать окружность, причем высота пирамиды
попадает в центр этой окружности.
5(22.8). Дана пирамида, в основании которой лежит трапеция. Могут ли быть равны:
а) все боковые ребра этой пирамиды;
б) все высоты боковых граней, опущенные их вершины
пирамиды?

106

Глава 4. Перпендикулярность прямой и плоскости

О т в е т: такая пирамида существует а) в случае, когда
трапеция равнобедренная; б) в случае, когда суммы длин
противоположных сторон трапеции равны.
6(22.9). Найти геометрическое место оснований перпендикуляров, опущенных из данной точки A на все прямые,
лежащие в заданной плоскости и пересекающиеся в одной
точке B.
О т в е т: окружность. Центр этой окружности является
серединой отрезка, соединяющего точку B с основанием
перпендикуляра, опущенного из точки A на заданную
плоскость, сам же отрезок является ее диаметром.
7(22.11). Наклонная l образует равные углы с тремя
попарно непараллельными прямыми l1 , l2 , l3 , лежащими
в одной плоскости. Докажите, что наклонная перпендикулярна этой плоскости.
Решение. Пусть e~1 , e~2 , e~3 , e~ — соответственно единичные
направляющие векторы попарно непараллельных прямых l1 , l2 , l3 и наклонной l. Тогда, согласно условию, e~1 · e~ = e~2 · e~ = e~3 · e~. Отсюда e~(e~1 − e~3 ) = 0 и e~(e~1 − e~2 ) = 0.
Поскольку векторы e~1 − e~2 , e~1 − e~3 не параллельны и ненулевые, их можно принять в качестве базисных векторов
плоскости. Тогда e~ ⊥ α, а значит, l ⊥ α.
8(22.12). Докажите, что скрещивающиеся ребра правильной треугольной пирамиды попарно перпендикулярны.
9(22.15). Плоские углы при вершине D тетраэдра ABCD —
прямые. Докажите, что основание высоты, опущенной
из вершины D этого тетраэдра, совпадает с точкой
пересечения высот основания.
Решение. В треугольнике DAB проведем высоту DL
(рис. 4.12). Тогда плоскость CDL перпендикулярна прямой
AB. Действительно, поскольку CD ⊥ DAB, DL является
проекцией наклонной CL на плоскость DAB и по теореме
о трех перпендикулярах AB ⊥ CL. Учитывая, что AB ⊥ DE,
заключаем, что AB ⊥ CDL. Пусть DE — высота треугольника DLC.

§ 22. Перпендикуляр и наклонная 107
D

B
L

E

A

C

Рис. 4.12.

Тогда DE ⊥ AB и DE ⊥ LC, следовательно, DE — высота
пирамиды, опущенная из вершины D на основание ABC.
Но CL — одна из высот основания. Аналогичным образом
можно доказать, что точка E как основание высоты пирамиды принадлежит и двум другим высотам треугольника
ABC.
Значит, точка E является ортоцентром основания пирамиды.
10(22.17). Докажите, что если S1 , S2 , S3 — площади
боковых граней тетраэдра, у которого все плоские углы,
примыкающие к вершине, прямые, а S — площадь его
основания, то S21 + S22 + S23 = S2 .
Решение. Для доказательства используем рис. 4.12
и начальные выкладки решения задачи 9, согласно
которым высота DE данного тетраэдра проходит через
ортоцентр основания. Введем обозначения AD = a,
BD = b,
CD = c.
В
треугольнике
ABD
AB · DL =
= a · b ⇒ DL =

a·b
a·b
=p
.
AB
a2 + b2

В

треугольнике

△CDL

1
1 DL · CD
CL · AB =
· AB =
2
2
DE
DL
1 CL
1
· CD · AB.
SABC =
· CD · AB = CL · AB =
DE
2 CD r
2


1
a2 b2 √ 2
2
2
2
2
2
CD + DL · a + b . SABC =
c + 2 2 a + b2 =
2
a +b

DL · CD = CL · DE.

SABC =

1
2
1
=
2
1 √ 2 2
=
c a + c2b2 + a2b2 . Но c2 a2 = 4S21 ; c2 b2 = 4S22 ; a2 b2 = 4S23 .
2

=

Таким образом, S2 = S21 + S22 + S23 .

108

Глава 4. Перпендикулярность прямой и плоскости

11(22.23). Дан куб ABCDA1B1 C1 D1 с ребром 1. Найдите
расстояние от точки A1 до плоскости ACD1 .
Решение. Способ 1. Поскольку A1 O1 = O1D (рис. 4.13),
расстояние от точки A1 до плоскости AD1 C равно расстоянию от точки D до той же плоскости (см. задачу 2).
Рассмотрим треугольник ODD1. Высота DH этого треугольника будет перпендикуляром к плоскости AD1 C,
так как DH лежит в плоскости ODD1 , перпендикулярной AC (поскольку AC ⊥ DD1, AC ⊥ D1 O). Следовательно, DH ⊥ AC, DH ⊥ DO. Но в прямоугольном треугольнике ODD1 выполняется
OD · DD1 = OD1 · DH. Отсюда

ρ(D, AD1 C) = DH =

z

2
1
1
v
 √ 2 = √3 .
2 u
u
t
2
12 +
2
D1

C1

A1

B1
O1

H
y

D

C
O

A

B

x

Рис. 4.13.

~ = (x; y; 1) является нормальным
Способ 2. Пусть вектор n
для плоскости AD
1 C. Найдем его координаты из условия
8−−


>
−−→ ~ −−→ −−→

⇒ n = (1;−1;1).
y+1=0
: AD = (0; 1; 1)
Чтобы воспользоваться формулой расстояния от точки
до плоскости, выберем на плоскости AD1 C точку A(0; 0; 0);
−−→
~ · AA
n
−−→
−−→
1
1
AA1 = (0; 0; 1). ρ(A, AD1 C) = Прn~ AA1 =
=√ .
~
n

1

О т в е т: √ .
3

3

§ 22. Перпендикуляр и наклонная 109

12(22.24). Из некоторой точки на плоскость опущен перпендикуляр и проведены две наклонные, которые составляют с перпендикуляром углы 45◦ , а между собой —
угол 60◦ . Найти угол между проекциями этих наклонных.
Решение. Опустим из точки A перпендикуляр на плоскость
α (рис. 4.14). Треугольники △AOB, △AOC равны между
собой, являются равнобедренными √и прямоугольными.
Если принять AO = 1, то AB = AC = 2.
A

O
B
α

C

Рис. 4.14.

Треугольник
ABC является равносторонним, поэтому

BC = 2, а значит, BC2 = 2 = BO2 + OC2.
Отсюда ∠BOC = 90◦ .

О т в е т: 90◦ .

13(22.25). Найдите плоский угол при вершине правильной
треугольной пирамиды, если этот угол равен углу между
боковым ребром и плоскостью основания пирамиды.
Решение. В правильной пирамиде ABCD (рис. 4.15)
со стороной основания 1 и высотой DO длина
1

отрезка AO равна √ . Обозначим ∠OAD = φ. В треугольнике

BDC

3

высота

DK

является

биссектрисой,

1
поэтому
∠BDK = ∠OAD.
Отсюда
боковое
ребро
2
BK
1
DB =
 , т. е. DB =
 . Поскольку
1
1
sin
∠OAD
2 sin
∠OAD
2
2
1
1
AD = DB, верно равенство √
=
.
1
3 cos ∠OAD
2 sin
∠OAD
2

110

Глава 4. Перпендикулярность прямой и плоскости
D
ϕ
2

B
O

ϕ
A

K
C

Рис. 4.15.
φ

Тогда 3 cos2 φ = 4 sin2 . Используя тригонометрические
2
2
формулы, запишем 3 cos
√ φ − 2(1 − cos φ) = 0.
Отсюда cos ∠OAD =

О т в е т: arccos



7−1
.
3

7−1
.
3

14(22.26). Угол между боковым ребром и плоскостью
основания правильной треугольной пирамиды DABC с вершиной D равен 60◦ . Через точку A проведена плоскость,
перпендикулярная биссектрисе угла D треугольника BDC.
В каком отношении линия пересечения этой плоскости
с плоскостью BDC делит площадь грани BDC?
Решение. Пусть условию задачи отвечает сечение AMN
(рис. 4.16). Ясно, что MN k BC. Введем систему координат,
как это показано на рис. 4.16. Приняв длину отрезка
AO за 1, запишем координаты всех необходимых
√ точек:


0,5
λ 3
A(0; −1; 0), L(0; 0,5; 0), D(0; 0; 3), K 0;
;
, где
1+λ 1+λ
LK
λ=
(формулы деления отрезка в данном отношении).
KD
S
Тогда искомым будет отношение MDN .
SBMNC

Запишем его в виде

SMDN
1
1
= S 1
=
=
.

SBDC − SMDN
(1 + λ)2 − 1
LD 2
BDC
−1
−1
SMDN
KD

§ 22. Перпендикуляр и наклонная 111

−−→ −−→
В соответствии с условием задачи имеем AK ⊥ DL,
−−→ −−→
поэтому
λ
найдем
из
равенства
AK · DL = 0.
√ 

−−→ −−→
0,5
λ 3
AK = 0;
;
,
DL = (0; 0,5; 3).
AK · DL = 0 ⇒
1+λ

1+λ

⇒ 0,5 · (1,5 + λ) = 3λ ⇒ λ = 0,3.
Тогда

1

(1 + λ)2 − 1

=

100
.
69

z
D

M

O

B

N

K

y

L
C

A

x

Рис. 4.16.

О т в е т: 100:69 или 69:100.
15(22.27). Точка M — середина ребра AD единичного куба
ABCDA1B1 C1 D1 . Через середину K отрезка B1 M перпендикулярно к нему проводится плоскость α (рис. 4.17).
Найдите расстояние от центра куба до плоскостиα.
D1

z

C1

A1

B1
K
M

A

y
C

D
B

Рис. 4.17.

x

112

Глава 4. Перпендикулярность прямой и плоскости

Решение. Для решения этой задачи не обязательно
строить сечение. Достаточно использовать формулу
−−→
ρ(O, α) = Пр−−−−→ KO .
MB1

Здесь O — центр куба. В соответствии с введенной системой координат: M(0; 0,5; 0), B1 (1; 0; 1), O(0,5; 0,5; 0,5).
Координаты точки K вычисляются по формулам деления отрезка в данном отношении при λ = 1. То есть
K(0,5; 0,25; 0,5).
−−→ −−−→
|KO · MB1 |
0,125
1
=√
= .
Тогда ρ(O, α) = −−−−→
12
2,25
|MB1 |

О т в е т: 1/12.
16(22.28). В сечении прямоугольного параллелепипеда
с квадратным основанием плоскостью получается ромб
с острым углом 60◦ . Под каким углом пересекают плоскость
сечения боковые ребра параллелепипеда?
Решение. Способ 1. На рис. 4.18 показано сечение прямоугольного параллелепипеда — ромб AB1 C1 D1 . В треугольнике ACC1 проведем высоту CH. Покажем, что отрезок
CH перпендикулярен плоскости сечения. Действительно,
прямая AC является проекцией прямой CH на плоскость
ABC. Но AC ⊥ DB и прямая DB параллельна D1 B1 , отсюда
AC ⊥ D1B1 , значит, по теореме о трех перпендикулярах
CH ⊥ D1B1 .
Учитывая, что CH ⊥ AC1 , получим, что прямая CH
перпендикулярна плоскости AD1 B1 .
z

D1

C1
C1

B1
A1

A

D1

H

D

y B1

B

Рис. 4.18.

C
x

§ 22. Перпендикуляр и наклонная 113

Для нахождения угла CC1 H примем длину стороны
ромба за 1. Треугольник AD1 B1 является равносторонним, так как угол A по условию равен 60◦ , поэтому
1
D1 B1 = DB = 1 ⇒
√AC = DB = 1. Длина диагонали AC ромба
1
1
AD CB равна 3. Поэтому в прямоугольном
треугольнике

AC1 C синус угла AC1 C равен 1/ 3.
Способ 2. Введем систему координат, как это показано на
рис. 4.18. Нормальный вектор сечения можно представить
~ = (x; y; 1). Найдем этот вектор из условия, что
в виде n



−→
1
~ ⊥ AB1 , n
~ ⊥−
n
AD1 . Приняв
√ длину AB за 1 и учтя, что
1
1
1
1
D B = DB = AB , AC = 3, из треугольников ABC, ACC1 ,

1
1
получим AB = √ , BB1 = √ , CC1 = 2 · BB1 = 2.
2
2

1
1
Запишем координаты точек: A(0; 0; 0), B1 √ ; 0; √ ,
2
2

1
1
D1 0; √ ; √ .
2
8 2−−→
x
1
>
1
>
~
>

−−→1
y
1
~ · AD
>
=0⇒ √ + √ =0⇒y=1
: n
2

2

~ боковых ребер имеет координаты
Единичный вектор k
~
k = (0; 0; 1). Тогда угол φ, под которым боковые ребра
пересекают плоскость ромба, вычисляется по формуле
φ = arcsin

~ ·k
~
n
~ · k
~
n

1

= arcsin √ .
3

1

О т в е т: arcsin √ .
3

17. Точка P лежит на ребре AB правильного тетраэдра
ABCM, причем BP : PA = 2 : 1. Найдите угол между прямой
PC и плоскостью AMC.
Решение. Способ 1. В треугольнике OME из точки K
проведем перпендикуляр KT к гипотенузе ME. Докажем,
что он будет являться перпендикуляром к плоскости AMC.
Действительно, проекцией прямой KT на плоскость ABC
является прямая BE. Она же является проекцией ребра

114

Глава 4. Перпендикулярность прямой и плоскости

BM на плоскость ABC. Но BE ⊥ AC, значит, по теореме
о трех перпендикулярах KT ⊥ AC, отсюда KT ⊥ ABC.
Для расчета угла KCT примем длину ребра правильного тетраэдра за 1. Из подобия треугольников OKP
и EKC, зная, что BP : PA = 2 : 1, нетрудно заключить, что
OK : KE = 2 : 3.√Тогда BE : KE = 15 : 3, или BE : KE = 5 : 1.
Значит, KE = 3/10. Тогда в прямоугольном
треуголь√

2
нике KEC выполняется равенство KC = KE + EC2 = 7/5.
Отметим, что отношение высоты тетраэдра, опущенной из
к длине перпендикуляра KT
точки B на плоскость AMC,

2

равно 5, поэтому KT = √ . Отсюда синус угла KCT равен




5 3

2

21


2

. Угол KCT равен arcsin √

21

.

Способ 2. Введем систему координат, как это показано
на рис. 4.19, и примем длину ребра тетраэдра за 1. Тогда
можно записать координаты
для решеr  точек, необходимых


2
1
1
1
, C − √ ; ; 0 , P 0; − ; 0 ,
ния задачи: M 0; 0;
3
3
2 3 2

1
E − √ ; 0; 0 .
2 3

z
M

A
T
P
x

E
O

K

B

C
y

Рис. 4.19.

~ плоскости можно задать коордиНормальный вектор n
натами (x; y; 1).

§ 22. Перпендикуляр и наклонная 115

−−→
−−−→
~ из условий, что n
~ · EC
~ · EM
Найдем n
= 0, n
r = 0,
√ 

−−−→
1
2 −−→
1
x
2
EM = √ ; 0; √ , EC = 0; ; 0 . При этом √ +
= 0,
2
3
2 3
3
2 3


−→
1
1
5
~ = (−2 2; 0; 1), −
y = 0, n
PC = − √ ; ; 0 .
2

2 3

6



1

(−2
2)
·




~ · PC|
|n
2 3
r
,
Искомый угол φ = arcsin
−−→ = arcsin
~
28
|n| · |PC|

36
r
2
φ = arcsin
.
21

r

О т в е т: arcsin

2
.
21

18. В основании прямой призмы ABCMA1B1 C1 M1 лежит
равнобедренная трапеция ABCM, причем AB и MC —
ее основания. MA = AB = CB = 4, CM = 8. Боковое ребро
призмы равно 4. Вычислить угол между прямой AA1
и плоскостью M1 A1 B.
Решение. Способ 1. Сечение призмы плоскостью M1 A1 B
представляет собой параллелограмм KM1 A1 B, KB k M1 A1 ,
MK = KC. Проведем в плоскости нижнего основания
призмы из точки A перпендикуляр AP к прямой BK.
По теореме о трех перпендикулярах, прямая BK перпендикулярна плоскости APA1.
В плоскости APA1 из точки A опустим перпендикуляр
AE на прямую A1 . Отрезок AE перпендикулярен как PA1 ,
так и BK. Следовательно, он перпендикулярен плоскости
сечения KM1 A1 B. Отсюда вытекает, что PA1 — проекция
наклонной AA1 . Таким образом, искомым углом является
угол AA1 P. В прямоугольном треугольнике APA1 его
нетрудно рассчитать. Действительно, угол AKP в прямо—
угольном треугольнике AKP равен 60◦ , так как ABKM


3
= 2 3.
ромб с острым углом 60◦ и AP = AK · sin 60◦ = 4 ·


AP
2 3
3
=
=
,
Поэтому tg ∠AA1P =
AA1
4
2

3
Искомый угол равен arcsin
.
7

2

а sin ∠AA1P =



3
.
7

116

Глава 4. Перпендикулярность прямой и плоскости

Способ 2. Треугольник KBC равносторонний. Это позволяет ввести систему координат с осью ординат, проходящей
через середину отрезка KC и началом в точке B (рис. 4.20).


−−→
В данной системе координат BK = (2,2 3, 0) = 2(1, 3, 0),
−−−→
BA1 = (4,0, 4) = 4(1,0, 1).
z
B1

A1

M1

C1

E
B
x

A
P

M

K

y

C

Рис. 4.20.

Пусть нормальный вектор сечения запишется в виде
~ = (x∗ , y∗ , 1).
n
−−→
−−−→

~ ⊥ BK
~ ⊥ BA
~ −−→
Ясно, что n
и n
1 . Тогда n · BK = 0 ⇒ x +
√ ∗



1
~ · BA = 0 ⇒ x∗ + 1 = 0. Отсюда x∗ = −1, y∗ = √ ,
+ 3y = 0 и n
1
3

1
~ = −1, √ , 1 . Направляющий вектор прямой AA — это
n
1
3

~ = (0,0, 1). Искомый угол найдем по формуле
вектор k
r
φ = arcsin

~
~ ·n
k
1
= arcsin √
= arcsin
~
~
7/3
|k| · |n|

r

О т в е т: arcsin

3
.
7

3
.
7

19. Дан куб ABCDA1B1 C1 D1 . Вычислите угол между
плоскостью A1 BC1 .
прямой AD1 и r
О т в е т: arcsin

2
.
3

20. Точка P — середина ребра AD прямоугольного
параллелепипеда ABCDA1B1 C1 D1 с отношением ребер
AB : AD : AA1 = 1 : 2 : 1.

§ 23. Связь между параллельностью прямых и плоскостей 117

Найдите угол
r между прямой PC1 и плоскостью CDA1.
О т в е т: arcsin

3
.
5

§ 23. СВЯЗЬ МЕЖДУ ПАРАЛЛЕЛЬНОСТЬЮ ПРЯМЫХ
И ПЛОСКОСТЕЙ И ПЕРПЕНДИКУЛЯРНОСТЬЮ
ПРЯМОЙ И ПЛОСКОСТИ

Содержание данного раздела в учебнике представляет собой
ряд следствий из разобранных ранее теорем и определений.
Однако сформулированные в нем утверждения играют роль
ключевых элементов в стереометрии. Поэтому целесообразно представить их учащимся в виде задач. Отметим
при этом, что выбор векторных решений во многих случаях обусловлен стремлением авторов настоящего пособия
избежать дублирования учебника. Желательно, чтобы учащиеся усвоили как чисто геометрические доказательства,
представленные в учебнике, так и векторные.

Упражнения и задачи для закрепления
1. Докажите, что две прямые, перпендикулярные одной
плоскости, параллельны между собой.
~ b,
~ ~c, d~ — единичные векторы, причем
Решение. Пусть a,
~ и b
~ — базисные векторы плоскости α; ~c и d~ — направa
ляющие векторы двух прямых l1 и l2 , перпендикулярных
плоскости α.
l1
α

~a
ϕ

l2

~c

~b

Рис. 4.21.

~
d

118

Глава 4. Перпендикулярность прямой и плоскости

~ b,
~ ~c образует пространственный
Тройка векторов a,
базис. Поэтому найдутся числа λ, β, γ, для которых
~ + βb
~ + γ~c.
d~ = λa
~ потом
Умножим это равенство скалярно сначала на a,
~ ·a
~ ·b
~ = λa
~ + βb
~ + γ~c · a,
~ d~ · b
~ : d~ · a
~ = λa
~ + βb
~ + γ~c · b
~.
~·a
~·b
на b
~
~
~
~
Отсюда следует, что 0 = λ + βb · a и 0 = λa · b + β.
~ и b
~ — единичные векторы, поэтому
Учтем, что a
~
~ и b.
~ = cos φ. Здесь φ — угол между векторами a
~ Тогда
a·b
~
либо λ = β = 0, либо cos φ = 1. Если cos φ = 1, то векторы a
~ коллинеарны, но тогда они не могут быть базисными.
и b
Следовательно, λ = β = 0.
Это означает, что d~ = γ~c , т. е. l1 k l2 .
2. Докажите, что если одна из двух параллельных прямых
перпендикулярна плоскости, то и другая перпендикулярна
этой плоскости.
Решение. Пусть прямая l1 параллельна l2 и перпен~ и b
~ задают базис
дикулярна плоскости α, векторы a
плоскости, векторы ~c и d~ являются направляющими
векторами соответственно прямых l1 и l2 . Тогда ~c · a
~ = 0,
~c · b
~ = 0. Но в силу параллельности прямых d~ = k · ~c, k ∈ R.
~ = 0 и k~c · b
~ = 0. Значит, l ⊥ α.
Следовательно, k~c · a
2
3. Докажите, что прямая и плоскость, перпендикулярные
данной прямой, параллельны.
Решение. Пусть имеются прямые l, l1 и плоскость α,
для которых l ⊥ l1 , l ⊥ α. Докажем, что l1 k α. Пусть ~c, c~1 —
единичные направляющие векторы соответственно прямых
~ b
~
~ — базис плоскости α. Тогда a,
l и l1 , а векторы a,
~ ~c — базис пространства и имеет место разложение
b,
~ + βb
~ + γ~c.
c~1 = λa
Умножим последнее равенство скалярно на ~c. Так как
~
~
~ · ~c = 0, получим ~c · c~ = γ. Поскольку ~c ⊥ c~ , получаем
a·c=b
1
1
~ + βb,
~ т. е. l k α.
γ = 0. Значит, ~c = λa
1
4. Докажите, что все точки прямой, параллельной плоскости, удалены от этой плоскости на одинаковое расстояние.

§ 23. Связь между параллельностью прямых и плоскостей 119

~ — нормальный вектор плоскости α,
Решение. Пусть n
A и B — произвольные точки прямой l, которая параллельна плоскости α (рис. 4.22). Расстояние от точки A до
−−→
плоскости α вычисляется по формуле ρ(A, α) = |Прn~ AC|.
B

A

l

~
n
α
C

Рис. 4.22.

Здесь C — произвольная точка плоскости α. Для точки
−−→
B эта формула запишется в виде ρ(B, α) = |Прn~ BC|.
−−→
−−→ −−→
−−→
~ ⊥ AB,
Учтем, что BC = −(CA + AB) и n
поэтому






~ · BC| = |n
~ · AC|.
|n
Отсюда ρ(B, α) =

−−→
~ · AC|
|n
= ρ(A, α).
~
|n|

5. Докажите, что все точки данной плоскости находятся на
одинаковом расстоянии от параллельной ей плоскости (это
расстояние называется расстоянием между параллельными
плоскостями).
6. Докажите, что отношение длин параллельных наклонных, проведенных из точек A и B к плоскости α, равно
отношению длин их проекций A1 A2 и B1 B2 и отношению
длин перпендикуляров AA2 , BB2, опущенных соответственно из точек A и B на плоскость α.
Указание. Для доказательства рассматривается подобие
треугольников AA1 A2 и BB1 B2 .
7. Могут ли: а) две пересекающиеся плоскости быть
перпендикулярными одной прямой; б) две пересекающиеся прямые быть перпендикулярными одной плоскости;

120

Глава 4. Перпендикулярность прямой и плоскости

в) существовать две плоскости, одна из которых перпендикулярна данной прямой, а другая параллельна данной
прямой?
Решение. а) Если плоскости пересекаются, то их нормальные векторы не могут быть коллинеарны. В то же время
направляющий вектор прямой, перпендикулярной каждой
из плоскостей, должен быть коллинеарен соответствующему нормальному вектору каждой плоскости. Поскольку
для положительного ответа на вопрос требуется коллинеарность всех трех векторов, две пересекающиеся плоскости
не могут быть перпендикулярными одной прямой.
О т в е т: нет.
б) Для положительного ответа на вопрос требуется
коллинеарность направляющих векторов прямых и нормального вектора плоскости.
О т в е т: нет.
в) Если плоскости перпендикулярны, то любая прямая, параллельная одной из них, будет перпендикулярна
другой.
О т в е т: да.
8. Расстояние между параллельными плоскостями равно a.
Отрезок длины b упирается своими концами в обе плоскости. Докажите, что длины проекций отрезка на обе
плоскости√ одинаковы. Чему они равны?
О т в е т:
b2 − a2 .
9. Отрезок длины 13 пересекает плоскость. Найдите длину
отрезка, соединяющего основания перпендикуляров, опущенных из концов отрезка на плоскость, если длины
перпендикуляров равны 7 и 5.
О т в е т: 5.
10(23.9). Дан куб ABCDA1B1 C1 D1 . Постройте сечение куба
плоскостью, проходящей через середину M ребра AB
и перпендикулярной прямой A1 B.
Решение. Искомая плоскость α (рис. 4.23) пересекает
грань AA1 B по прямой MK, перпендикулярной A1 B. Так
как прямая BC перпендикулярна A1 B, то она должна

§ 23. Связь между параллельностью прямых и плоскостей 121

быть параллельна плоскости сечения. Поэтому грани BB1C
и ABC пересекаются ею по прямым соответственно KL
и MN, KL k MN k BC. Таким образом, MKLN — искомое
сечение.
B1

C1
D1

A1
K

L

B
C
M

N

A

D

Рис. 4.23.

11. Найдите расстояние от центра куба ABCDAB1C1 D1 до
плоскости A1 BD, если ребро куба равно 1.
Решение. Способ 1. Пусть точки O и O1 — соответственно
центры нижнего и верхнего оснований куба. В прямоугольном треугольнике A1 O1 O из точки O1 опустим перпендикуляр O1 K на гипотенузу OA1 . Нетрудно показать,
что его длина есть расстояние от точки O1 до плоскости
A1 BD. Действительно, прямая BD перпендикулярна диагональной плоскости AA1 C (BD ⊥ AC, BD ⊥ OO1), а значит,
BD ⊥ OA1.
Отсюда O1 K ⊥ OA1 и O1 K ⊥ BD, т. е. O1 K — перпендикуляр к плоскости A1 BD. Его длину нетрудно найти
из равенства OO
· O1 A1 = O1K · OA1 , поскольку OO1 = 1;
√1
1

3

OA = √ ; OA1 = √ .
2

2
1
Тогда O1 K = √ . Но так как длина наклонной OO1 в два
3

раза больше длины наклонной O2 O, (O2 — центр куба), то
1

ρ(O2 , A1 BD) = √ .
2 3

Способ 2. Воспользуемся
−−−→
= |Прn~ OO2 |.

формулой

ρ(O2 , A1 BD) =

122

Глава 4. Перпендикулярность прямой и плоскости

−−−→
Заметим, что вместо OO2 может быть взят вектор
−−−→
−−−−→
~
BO2 или A1 O2 и т. д. В качестве нормального вектора n
−−−→
−−−→
возьмем вектор, перпендикулярный как BA1 , так и DA1 .
Введем систему координат, как это показано на рис. 4.24.
−−→
−−−→
~ = (x; y; 1), −
Пусть n
DA1 = (0; 1; 1), BA1 = (1; 0; 1), тогда
8

−−→
>
~ ·−

x+1=0
:n
~ ·−
BA1 = 0

−−−→ ~
|OO2 · n|
−−−→
1
1
OO2 = 0; 0;
, и поэтому ρ(O2 , A1 BD) =
= √ .
~
|n|

2

z

B1

2 3

A1
O1

C1

D1
K

O2
B

y
A
O

C

D

x

Рис. 4.24.

12(23.11). В параллелепипеде ABCDA1B1 C1 D1 диагональ
AC1 равна d и образует с плоскостью B1 CD1 угол 30◦ .
Найти расстояние от середины отрезка A1 C1 до плоскости
A1 BD.
Решение. Нетрудно видеть, что плоскости B1 CD1 и A1 BD
параллельны и середина отрезка A1 C1 принадлежит плоскости B1 CD1 . Поэтому задача состоит в нахождении
расстояния между этими параллельными плоскостями.
Рассмотрим диагональное сечение CC1 A1 A параллелепипеда (рис. 4.25). Оно представляет собой параллелограмм,
диагональ AC1 которого имеет длину d и образует с параллельными плоскостями B1 CD1 и A1 BD угол 30◦ . Если
AC ∩ BD = O, A1 C1 ∩ B1 D1 = O1 , то диагональ AC1 параллелепипеда пересекает указанные параллельные плоскости

§ 23. Связь между параллельностью прямых и плоскостей 123
B1

A1
O1
C1

D1
L

B

N
A

O
D

C

Рис. 4.25.

в точках L и N, для которых N = AC1 ∩ A1 O, L = AC1 ∩ CO1.
Для параллелограмма AA1 C1 C выполняется равенство
C1 L = LN = NA, в этом легко убедиться, применив теорему
Фалеса. Действительно, поскольку CO1 k OA1 и CO = OA,
1

имеем C1 L = LN и LN = NA. Отсюда LN = d. Учитывая,
3
что угол наклона LN к указанным плоскостям равен 30◦ ,
расстояние между плоскостями равно LN · sin 30◦ =
О т в е т:

d
.
6

d
.
6

13(23.13). Вершины A, B и C треугольника ABC находятся
на расстояниях соответственно a, b и c от плоскости
α и расположены по одну сторону от этой плоскости.
Докажите, что расстояние от центра этого треугольника
до плоскости α равно

1
(a + b + c).
3

Решение. Пусть A1 , B1 , C1 — основания перпендикуляров, опущенных на плоскость из точек соответственно A, B,
C (рис. 4.26). O1 — основание перпендикуляра, опущенного
из точки O пересечения медиан треугольника.
Тогда AA1 = a, BB1 = b, CC1 = c.
Поскольку все перпендикуляры к одной плоскости
параллельны между собой, последовательно применяя
теорему Фалеса, можно показать, что точка O1 является
точкой пересечения медиан треугольника A1 B1 C1 .
Напомним, что в любом треугольнике ABC выполняется
−−→ −−→ −−→
равенство OA + OB + OC = ~0, где O — точка пересечения
медиан.

124

Глава 4. Перпендикулярность прямой и плоскости
B
O

A

C
B1
O1

A1

C1

Рис. 4.26.

−−−→ −−→ −−→
Поэтому справедливы утверждения OO1 = OA + AA1 +
−−−→
−−−→ −−→ −−→
−−−→
−−−→ −−→ −−→
−−−−→
+ A1 O1 , OO1 = OB + BB1 + B1 O1 , OO1 = OC + CC1 + C1O1 .
−−−→ −−−→ −−−→ −−−→
Сложив эти равенства, получим 3 · OO1 = AA1 + BB1 + CC1.
−−−→ −−→ −−−→
Поскольку векторы OO1 , AA1 , CC1 параллельны и сонаправлены, OO1 =
О т в е т: OO1 =

1
(a + b + c).
3

1
(a + b + c).
3

З а м е ч а н и е . В случае, если какая-либо из вершин треугольника, например A, будет располагаться по одну
сторону от плоскости α, а две оставшиеся — по другую, ясно
−−−→
−−→ −−−→
что вектор AA1 будет антинаправлен векторам BB1 , CC1 ,
поэтому для расстояния OO1 будет выполняться равенство
OO1 =

1
|b + c − a|.
3

14(23.14). Существуют ли в пространстве четыре попарно
перпендикулярные скрещивающиеся прямые?
~ b,
~
Решение. Допустим, что такие прямые существуют и a,
~c, d~ — их попарно перпендикулярные ненулевые направляющие векторы. Известно, что три линейно независимых
вектора в пространстве задают базис. Попарно ортогональные векторы линейно независимы, поэтому, например
~ b,
~ + βb
~ ~c задают базис. Тогда d~ = λa
~ + γ~c, здесь
векторы a,
~
λ, β, γ ∈ R. Умножим последнее равенство скалярно на d.
~
~
~
~
~
~
~
~
Получим d · d = γd · a + βd · b + γd · c. В силу ортогональности
векторов это означает, что d~ · d~ = 0. Но d~ не является

§ 24. Двугранный угол. Угол между плоскостями 125

нулевым вектором, поэтому в пространстве не существуют
четырех попарно перпендикулярных скрещивающихся прямых.
О т в е т: нет.
15. Найдите геометрическое место точек, удаленных от
плоскости α, задаваемой уравнением x + 2y − 2z − 5 = 0, на
расстояние 2.
Решение. Пусть точка M(x, y, z) принадлежит искомому множеству точек. Возьмем произвольную точку P

−−→
на плоскости α. Например, P(3, 1, 0). При этом MP =
= (x − 3; y − 1; z). Нормальный вектор к плоскости α можно
~ = (1, 2, −2). Используя формулу ρ(M, α) =
задать как n

−−→
= Прn~ MP , получим
−−→
~ ·−
n
MP
~
n

(

=2⇔

,x + 2y − 2z − 11 = 0,
|x − 3 + 2(y − 1) − 2z|
=2⇔,
* x + 2y − 2z + 1 = 0.
3

Полученные выражения суть уравнения плоскостей, параллельных данной плоскости α и находящихся по разные
стороны от нее.

§ 24. ДВУГРАННЫЙ УГОЛ. УГОЛ МЕЖДУ ПЛОСКОСТЯМИ

Основной целью данного параграфа является развитие
у учащихся навыка изображать двугранные углы и вычислять их величину с помощью теоремы о трех перпендикулярах. Поясним на примере.
Задача. В кубе ABCDA1 B1 C1 D1 с ребром a точка M —
середина ребра CD. Найдем величину двугранного угла
между плоскостями C1 AM и AMB. Ребром искомого
двугранного угла является прямая AM, а гранями —
полуплоскости α = AMC1 и β = AMB с границей AM
(рис. 4.27). Заметим, что ребро CC1 перпендикулярно грани
β, причем прямая CC1 пересекает плоскость α в точке C1 ,
а плоскость β — в точке C. Проведем CH ⊥ AM и, соединив

126

Глава 4. Перпендикулярность прямой и плоскости

точки C1 и H, получим линейный угол C1 HC искомого
двугранного угла.
ABCD
Для
вычислений
рассмотрим
плоскость
(рис. 4.28). Так как CH ⊥ AH, то △CMH ∼ △AMD, причем
∠MCH = ∠MAD. Поэтому
AD = CH ⇒ r
AM
CM

a

a2
a2 +
4

a
= CH
a ⇒ CH = √ .
5

2

Значит, линейный
угол ∠C1 HC двугранного угла C1 AMB

равен arctg 5.
B1

C1

A1
D1
C

B
M

C

B

H

M H
A

D

Рис. 4.27.

A

D

Рис. 4.28.

Для решения можно использовать и другой прием,
основанный на том, что линейный угол двугранного угла
равен углу между нормальными векторами его граней.
Второй важной темой параграфа выступает раздел
о скрещивающихся прямых. Здесь важно подчеркнуть
принцип построения общего перпендикуляра к двум скрещивающимся прямым l1 , l2 , который состоит из трех
этапов.
1. Через одну из прямых (l1 ) строится плоскость α,
параллельная другой прямой (l2 ).
2. Из произвольной точки прямой l2 опускается перпендикуляр на плоскость α, и через основание этого перпендикуляра в плоскости α проводится прямая, параллельная l2 .
3. Фиксируется точка L ∈ l1 , из точки L опускается перпендикуляр на прямую l2 , который и является искомым.

§ 24. Двугранный угол. Угол между плоскостями 127

Применение этого алгоритма построения общего перпендикуляра к двум скрещивающимся прямым наглядно
продемонстрировано в решении задачи 6(24.20).
Важно обратить внимание учащихся на второй способ
построения общего перпендикуляра к двум скрещивающимся прямым, основанный на знании основ аналитической геометрии (см. § 22). Он также рассмотрен
в решении задачи 6(24.20). Достоинством второго способа
является несложное логическое содержание, отсутствие
геометрических построений. Приятным следствием такого
подхода к решению будет простота вычисления длины
этого перпендикуляра, т. е. расстояния между скрещивающимися прямыми.
Однако для решения задач, связанных с нахождением
расстояния между скрещивающимися прямыми, как правило, не требуется строить общий к ним перпендикуляр,
поэтому поступают проще. Через одну из прямых строят
плоскость, параллельную другой прямой, и описанными
в § 22 способами определяют расстояние от второй прямой
до построенной плоскости.
Применение формулы расстояния от точки до плоскости
на векторной основе позволяет унифицировать большинство типов таких задач по методам их решения.
В целом следует отметить, что использование теоретического материала данного параграфа на практике
теснейшим образом связано с вопросами параллельности плоскостей, параллельностью прямой и плоскости,
теоремой о трех перпендикулярах. Поэтому эффективное
решение предлагаемых ниже задач возможно только при
условии серьезного изучения предшествующих тем.
Упражнения и задачи для закрепления
1(24.26). Боковое ребро правильной четырехугольной
пирамиды наклонено к плоскости основания под углом 45◦ .
Найдите угол наклона к основанию боковой грани.
Решение. В правильной четырехугольной пирамиде
ABCKM (рис. 4.29) угол наклона боковой грани к основанию измеряется величиной линейного угла OPM, где
MP — апофема боковой грани BMC. Действительно, по

128

Глава 4. Перпендикулярность прямой и плоскости

теореме о трех перпендикулярах, OP ⊥ BC, поэтому OP
и MP являются перпендикулярами к прямой BC, по
которой боковая грань пересекает основание.
M
Если принять длину высоты MO
пирамиды за 1, то в треугольнике
MOC с углом C = 45◦ сторона OC
равна 1. Поскольку ABCK — квад45◦

K

1

C

рат, OP = √ , отсюда тангенс угла
2

1

A

MPO равен √ .

P

O

2

B

1

О т в е т: arctg √ .
2

Рис. 4.29.

2. В прямоугольном параллелепипеде ABCDA1 B1 C1 D1 заданы ребра AB = 4, AD = 5
и AA1 = 1. Найдите угол между плоскостями A1 BD и C1 BD.
Решение. Cпособ 1. Заметим, что двугранные углы, образованные плоскостями BDA1, BDC1 с плоскостью ABC,
вместе с углом между BDA1 , BDC1 в сумме составляют
развернутый угол, т. е. 180◦ . Для вычисления величины
двугранного угла между плоскостями ABD и BDA1 проведем AH ⊥ BD. Тогда, по теореме о трех перпендикулярах,
A1 H ⊥ BD (рис. 4.30).
B1

C1

z
A1

D1

y
B

C

H
A

x

D

Рис. 4.30.

Следовательно, угол AHA1 — линейный для указанного
двугранного угла. Пусть ∠ADB = γ.
Тогда tg γ =
4

sin γ = √

41

.

AB
4
= ,
AD
5

cos2 γ =

1

1 + tg2 γ

=

25
,
41

5

cos γ = √

41

,

§ 24. Двугранный угол. Угол между плоскостями 129
4

AH = AD · sin γ = 5 · √

Отсюда
из

41

прямоугольного


20

=√

треугольника

41

.

После

AA1 H

этого

находим

AA1
41
tg ∠AHA1 =
=
.
AH
20

Аналогично получается, что величина двугранного √
угла
41
между плоскостями BDC1 и ABC также равна arctg
.
20
Поэтому угол θ между √ плоскостями A1 BD и√ C1 BD
π − 2 · arctg

равен

либо

либо

2 · arctg



41
20

в



41
,
20

если

противном


 2 · 41
41
20 = 40 41 > 0,
=
tg 2 arctg
41
20
359
1+
400

40 41
либо θ = arctg
.
359


π − 2 · arctg
случае.

π
41
< ,
20
2

Так

то θ = 2 · arctg

как


41
,
20

Cпособ 2. В соответствии с заданной на рис. 4.30
системой координат запишем A(0; 0; 0), B(0; 4; 0),
−−→
C(5; 4; 0), D(5; 0; 0), A1 (0; 0; 1), C1 (5; 4; 1), BD = (5; −4; 0),
−−−→
−−−→
A1 D = (−5; 0; 1),
DC1 = (0; 4; 1).
Обозначим
через
~
n1 = (m; n; 1) нормальный вектор к плоскости A1 BD, через
−−−→
n
~ 2 = (t; l; 1) — к плоскости C1 BD. Имеем: n
~ 1 ⊥ A1 D,


−−→
−−→
5m − 4n = 0,
1 ;1 ,n
n
~ 1 ⊥ BD ⇒
⇒n
~1 = 1
;
~

DC,
2
5 4
−5m + 1 = 0


−−→
5t − 4l = 0,
n
~ 2 ⊥ BD ⇒
⇒n
~2 = − 1 ; − 1 ; 1 .
5
4
4l + 1 = 0

Отсюда cos θ = cos(~
n1 , n
~ 2) =

1
1
+1

25 16
359
=
. Используя
1
1
441
+
+1
25 16
1
2



тригонометрическую формулу 1 + tg θ =
, нетрудно
cos2 θ
убедиться, что данный результат совпадает полученным
выше.



41
40 41
О т в е т: 2 · arctg
или arctg
.
20
359

130

Глава 4. Перпендикулярность прямой и плоскости

3(24.12). Докажите, что прямая пересечения двух плоскостей, перпендикулярных третьей плоскости, перпендикулярна этой плоскости.
Решение. Способ 1. Пусть плоскости β и γ перпендикулярны плоскости α, пересекаются по прямой l и пересекают
плоскость α по прямым b и c соответственно (рис. 4.31).
β

b1

c1
l
γ
α

B

c
C

b

Рис. 4.31.

Возьмем на прямых b и c точки B и C и проведем
через эти точки в плоскостях β и γ прямые b1 , c1 ,
перпендикулярные соответственно прямым b, c. Тогда
прямые b1 и c1 перпендикулярны плоскости α и, следовательно, b1 k c1 . Плоскость β проходит через прямую b1 ,
параллельную плоскости γ, поэтому прямая b1 параллельна
линии пересечения этих плоскостей l. Итак, мы получили,
что l k b1 , но b1 ⊥ α и, значит, l ⊥ α, что и требовалось
доказать.
Способ 2. Пусть a
~ — направляющий вектор прямой l, n
~ 1,
n
~ 2, n
~ — соответственно нормальные векторы плоскостей β,
γ и α. По условию, n
~1 ⊥ n
~ и n
~2 ⊥ n
~ . Но это означает, что
n
~1 k α и n
~ 2 k α. Следовательно, n
~1 и n
~ 2 — базисные векторы
плоскости α. Поскольку a
~ ⊥n
~1 и a
~ ⊥n
~ 2, a
~ ⊥ α. Отсюда l ⊥ α,
что и требовалось доказать.
4(24.13). Найдите угол между гранями правильного тетраэдра.
Решение. Опустим из вершины D высоту DO
(рис. 4.32). Пусть плоскость DCO пересекает ребро AB

§ 24. Двугранный угол. Угол между плоскостями 131

в точке C1 . Так как точка O совпадает с центром треугольника ABC,
то C1 — середина AB и, поскольку
треугольники ABD и ABC правильные, DC1 ⊥ AB, CC1 ⊥ AB. Таким
A
образом, угол DC1 O является линейным углом двугранного угла между
плоскостями ABD и ABC. Учи1
1
тывая, что C1 O = C1 C = C1 D, из
3
3
прямоугольного треугольника C1 OD

D

C
α

O
B

Рис. 4.32.

C O
1 C D
1
1
находим cos α = 1 = · 1 = , откуда α = arccos .
C1 D
3 C1 D
3
3
1
О т в е т: arccos .
3

5(24.16).
костями
а) ACC1
г) ABC1

Дан куб (рис. 4.33). Найдите угол между плосα и β, где α и β — соответственно плоскости:
и DBB1 , б) ABC1 и ABC, в) BC1 D и ABC,
и BC1 D.
B1

z

C1
D1

A1

B

y

A

C
D

x

Рис. 4.33.

Решение. Пусть ребро куба имеет длину 1. Введем систему
координат, как показано на рис. 4.33. Обозначим через n
~ 1,
n
~ 2 соответственно нормальные векторы плоскостей α и β.
Тогда:
dβ) = 90◦ ;
а) n
~ 1 = (1; −1; 0), n
~ 1 = (1; 1; 0), n
~1 · n
~ 2 = 0 ⇒ (α,
dβ) = 45◦ ;
б) n
~ 1 = (1; 0; −1), n
~ 2 = (0; 0; 1), ⇒ (α,
dβ) = arccos √1 ;
в) n
~ = (1; 1; −1), n
~ = (0; 0; 1), ⇒ (α,
1

2

г) Способ
1. n
~ 1 = (1; 0; −1),
r
= arccos

2
.
3

3

n
~ 2 = (1; 1; −1),

dβ) =
⇒ (α,

132

Глава 4. Перпендикулярность прямой и плоскости

Способ 2. Угол между плоскостями равен углу между
перпендикулярами к этим плоскостям. Диагональ A1 C
перпендикулярна плоскости BC1 D, диагональ B1 C перпендикулярна плоскости ABC1 , следовательно, угол между
плоскостями ABC1 и BC1 D равен углу между прямыми A1 C
и B1 C (рис. 4.33). Из прямоугольного треугольника A1r
CB1
(∠A1 B1 C = 90◦ )
откуда α = arccos
О т в е т:

90◦ ;

cos α = cos ∠A1 CB1 =

находим
r

45◦ ;

2
.
3

1
arccos √ ; arccos
3

r

B1 C
=
A1 C

2
,
3

2
.
3

6(24.20). Постройте общий перпендикуляр скрещивающихся диагоналей смежных граней куба и вычислите
кратчайшее расстояние между этими диагоналями, приняв
длину ребра куба за 1.
Решение. Способ 1. Выберем в качестве смежных граней
куба AA1 K1 K и AA1 B1 B, при этом скрещивающимися
являются диагонали AB1 и KA1 . Из точки A1 проведем
прямую A2 A1 параллельно прямой AB1 (рис. 4.34).
B1

z

C1

A1
K1
E
P
y
M

C

B

A
K

x

T
A2

Рис. 4.34.

Диагональ AB1 параллельна плоскости A1 A2 K, поставим целью опустить перпендикуляр из точки A на
эту плоскость. Для этого рассмотрим пирамиду A1 A2 KA
с вершиной в точке A. Нетрудно видеть, что эта пирамида

§ 24. Двугранный угол. Угол между плоскостями 133

является правильной, поэтому ее высота, опущенная из
вершины A, попадает в центр правильного треугольника
A1 A2 K. Если A1 T — медиана этого треугольника, то его
центром является точка M ∈ A1 T, для которой A1 M = 2MT.
Через точку M как через основание перпендикуляра
проведем прямую ME, параллельную AB1 . Поскольку
E ∈ KA1 , построив отрезок EP, параллельный AM, можно
утверждать, что он является общим перпендикуляром
к скрещивающимся диагоналям AB1 и KA1 . Для нахождения длины отрезка EP учтем, что EP = AM. Высоту
AM пирамиды A1 A2 KA можно найти из pпрямоугольного треугольника MAA1√по формуле AM = AA21 − MA21.
2
2
3
1 √
Здесь MA1 = · TA1 = ·
· KA1 = √ · 2. Окончательно,
3
1
AM = EP = √ .
3

3

2

3

Способ 2. Введем систему координат, как показано на
рис. 4.34. В данной системе координат параметрические
уравнения прямых KA1 и AB1 соответственно имеют вид
x = t; y = 0; z = t + 1, t ∈ R и x = 0; y = p; z = p, p ∈ R.
Обозначим через EP общий перпендикуляр к указанным прямым. Тогда E(t; 0; t + 1), P(0; p; p) и коор−−→
−−→
динаты вектора EP имеют вид EP = (t; p; −1 + t + p).
−−→ −−−→
−−→ −−−→
−−−→
Учтем, что EP ⊥ KA1 и EP ⊥ AB1, KA1 = (−1; 0; 1),
−−→
−−→ −−−→
−−→ −−−→
AB1 = (0; 1; 1), получив EP · KA1 = 0 и EP · AB1 = 0. Тогда
1

1

t − 1 + t + p = 0 и p − 1 + t + p = 0. Отсюда p = , t = ,
3
3


1
2
1 1
E
; 0;
и P 0; ;
. Теперь, зная координаты точек
3

3

3

3

E и P, нетрудно построить перпендикуляр EP к скрещивающимся диагоналям куба. Длина этого перпендику−−→
1 1
1
ляра равна длине вектора EP = − ; ; −
, а именно
3 3
3
r
1
1
1
1
+ + =√ .
9
9
9
3
1

О т в е т: √ .
3

134

Глава 4. Перпендикулярность прямой и плоскости

7(24.22). В параллелепипеде ABCKA1 B1 C1 K1 диагональ
AC1 длины a наклонена к плоскости BKA1 под
углом 45◦ . Найдите расстояние между прямыми A1 B
и B1 C.
Решение. Прямые A1 B и B1 C лежат в параллельных
плоскостях A1 BK и B1 CK1 , поэтому расстояние между
этими прямыми равно расстоянию между плоскостями
A1 BK и B1 CK1 . Пусть диагональ AC1 пересекает плоскости A1 BK и B1 CK1 в точках M и T соответственно
(рис. 4.35).
A1

B1
O1

K1
C1

T
M
A

B
O

K

C

Рис. 4.35.

Вследствие параллельности прямых A1 O и O1 C имеем:
1

MT = TC1 и MT = AM, отсюда AM = MT = TC1 = a. Учи3
тывая, что точки M и T принадлежат соответственно
плоскостям A1 BK и B1 CK1 и отрезок MT образует
с этими плоскостями угол 45◦ , найдем расстояние
между

плоскостями A1 BK и B1 CK1 :
О т в е т: a

1
2
a · sin 45◦ = a
.
3
6



2
.
6

8(24.23∗ ). Дан куб ABCDA1B1 C1 D1 , ребро которого
равно a. Найдите расстояние между прямыми BD и D1 P,
где P — середина ребра CD, и постройте общий перпендикуляр этих прямых.

§ 24. Двугранный угол. Угол между плоскостями 135
C1

D1
O1
A1

B1

N

K
P

C
E

O

B

D
M
A

L

Рис. 4.36.

Решение. Способ 1. Обозначим через L точку пересечения
прямых D1 P и C1 C (рис. 4.36) и соединим центр O1 грани
A1 B1 C1 D1 с точкой L.
Поскольку прямая D1 P лежит в плоскости D1 O1 L,
а DB k D1O1 , расстояние между прямыми DB и D1 P равно
расстоянию от любой точки прямой DB до плоскости
D1 O1 L. Пусть прямые O1 L и AC пересекаются в точке E.
Проведем в треугольнике O1 OE высоту OK и покажем, что
прямая OK перпендикулярна плоскости D1 O1 L. В самом
деле, прямая D1 B1 перпендикулярна плоскости ACC1 A1
и, значит, D1 B1 ⊥ OK.
В то же время, OK ⊥ O1 L по построению. Итак,
прямая OK перпендикулярна двум пересекающимся
прямым плоскости D1 O1 L, а следовательно, и самой
этой плоскости. Поскольку точка O лежит на прямой DB, расстояние от этой прямой до плоскости
D1 O1 L равно длине перпендикуляра OK. Из равенства
прямоугольных треугольников D1 DP и LCP вытекает,
что D1 P =√PL, D1 D = CL = 1, поэтому ∠O1 OE = ∠LCE,
OE = EC =

∠KOE = α,

2
.
4

Обозначим ∠OO1E = α (рис. 4.37), тогда

O1 K = OK · ctg α,

EK = OK · tg α,

O1 K
=
EK

136

Глава 4. Перпендикулярность прямой и плоскости


=

OO1
OK · ctg α
= ctg2 α =
OK · tg α
OE

2

2
1

2
4

=

p
= 8, OK = O1 K · EK =



√ 1
2 2
= 8 · EK · EK = 2 2EK = 2 2 · O1 E =


9

O1
α

K
α
O

E

1+

1
1
= .
8
3

Таким образом, найдено расстояние
между скрещивающимися прямыми DB
и D1 P. Осталось построить общий перпендикуляр этих прямых (отрезок OK не
является общим перпендикуляром, так как
он не пересекает прямую D1 P). Предположим, что общий перпендикуляр прямых DB и D1 P пересекает эти прямые
в точках M и N соответственно. Тогда
MN k OK и MN = OK, поэтому четырехугольник MOKN является параллелограммом и NK k MO k D1B1 .
Поскольку

Рис. 4.37.

NKL

9

r

O K

= 1 =8 и
EK
подобны. Из этого

D1 N
=
NP

NK k D1 B1 k PE,

треугольники D1 O1 L и
подобия получим, что

EK + O1 E
NK
LK
EK + EL
EK + 9EK
5
=
=
=
=
= .
D 1 O1
O1 L
2 · O1 E
2 · O1 E
2 · 9EK
9
5
5
Отсюда NK = D1 O1 = DO, но MO = NK, поэтому
9
9
4
2
DM = DO − MO = DO = DB.
5
9

Итак, точка N делит отрезок D1 P в отношении 8 : 1,
а точка M — отрезок DB в отношении 2 : 7.

Способ 2. Напишем
прямых DB и D1 P
нат

(рис. 4.36).

D(0; 0; 0), DB:
p
2

y= ,
Пусть

параметрические
уравнения
заданной системе коорди
1
D1 (0; 0; 1),
P 0; ; 0 ,
B(1; 1; 0),
в

2

x = t,

y = t, z = 0, t ∈ R, D1 P: x = 0,

−−→
−−−→
1
z = 1 − p, p ∈ R. DB = (1; 1; 0), D1 P = 0; ; −1 .
2

NM — общий

перпендикуляр

этих

прямых,

§ 24. Двугранный угол. Угол между плоскостями 137



−−−→
p
N ∈D1 P, M∈DB, M(t; t; 0), N 0; ; 1−p . Тогда NM=
2
8−−−→ −−→
>



>
−−−→ −−−→
2
:NM·D P=0
1
8
8
p
p

>
>
>
>
t= ,
<
< t+t− 2 =0,
8
2
2 2
4
⇒> p p
⇒p= , t= , M
; ;0 ,
⇒> t p
9
9
9 9
>
: − −p+1=0
: − +1−p=0 >
8
4
2 4

4 1
N 0; ;
.
9 9
Зная координаты точек N и M, нетрудно постро
−−−→ 2
2
1
ить перпендикуляр NM. При этом NM=
;− ;−
=
9
9
9
−−−→
1
= (2; −2; −1). Длину вектора NM рассчитаем по формуле
9
−−−→ 1 √ 2 2
1
|NM|=
2 +2 +1= .
9
3
1
О т в е т:
.
3

9(24.24). В основании прямоугольного параллелепипеда
лежит квадрат, длина стороны которого равна 1, длины
боковых ребер равны 2. Найдите расстояние между прямыми AC и D1 B.
Решение. Способ 1. Пусть диагонали основания AC и DB
пересекаются в точке O (рис. 4.38). Опустим из точки O
перпендикуляр OP на прямую D1 B. Тогда отрезок OP —
общий перпендикуляр прямых AC и D1 B.
z
D1

C1

A1
B1
P

D

C
y

A
x

O
B

Рис. 4.38.

138

Глава 4. Перпендикулярность прямой и плоскости

В самом деле, OP⊥D1B по построению и OP⊥AC,
так как прямая AC перпендикулярна плоскости BDD1.
Из подобия прямоугольных треугольников
BDD1 и OPB

2
OP
OB
OP
1
2
находим
=
или
= √ , откуда OP= √ .
DD1 D1 B
2
6
3

~
n,
Способ 2. Вектор
перпендикулярный
прямым
AC
и
D1 B,
можно
записать
в
виде
−−→
~
~ −−→
~ −−−→
n=(x;
y; 1). Тогда
 n·AC=0 и n·D1 B=0, AC=(−1; 1; 0),
−−−→
−x+y=0, ~
D1 B=(1; 1; −1)⇒
⇒n=(1; 1; 1). Расстояние между
x+y−2=0
прямыми AC и D1 B вычисляется как абсолютная
−−→
−→
~ (можно −
~
величина проекции AB на n
BC на n).
−−→ ~
−−→ |AB·n| 1
ρ(AC, D1 B)=|Прn~ AB|=
=√ .
О т в е т:

~
|n|

1
√ .
3

3

10(24.25). Дана правильная четырехугольная пирамида
ABCKM с вершиной в точке M, в которой высота равна
стороне основания. Найдите расстояние между прямыми
AB и CM и постройте общий перпендикуляр этих прямых,
если AB=1.
Решение. Способ 1. Прямая AB параллельна плоскости
CMK, содержащей прямую CM (рис. 4.39), поэтому расстояние между прямыми AB и CM равно расстоянию от
прямой AB до плоскости CMK.
z
M

P1
E

K

x

P

H

Рис. 4.39.

C
y

O
A

H1

B

§ 24. Двугранный угол. Угол между плоскостями 139

Пусть P и E — середины отрезков AB и CK. Опустим из точки P перпендикуляр PP1 на прямую ME.
Поскольку ребро CK перпендикулярно плоскости PME,
имеем PP1 ⊥CK. Но отрезок PP1 перпендикулярен ME по
построению, поэтому PP1 ⊥CMK. Проведем через точку
P1 прямую P1 H1 , параллельную CK, и через точку H1
пересечения этой прямой с ребром CM — прямую H1 H,
параллельную PP1. Отрезок HH1 будет общим перпендикуляром прямых AB и CK. Вычислим его длину, учитывая,
треугольника MOE
что HH1 =PP1. Из прямоугольного
s


2

1
5
находим: ME= MO2 +OE2 = 1+
= . В треугольнике
2

PME PP·ME=MO·PE⇒PP1=
Поэтому
ром
точки


2 5
HH1 =
.
5

2


MO·PE 1·1
2 5
= √ ·2=
.
ME
5
5

Найдем отношение, в котоH
и
H√
делят
ребра
пирамиды.
1

p
5
△PP1 E: P1 E= PE2 −PP21 = , △MOE: ME= MO2 +OE2 =
5
5 MP1 3 MH1 3
= ⇒
= ⇒
= . Аналогичным образом доказы2
P1 E 2
H1 C 2
AH
вается, что
=4.
HB


Способ 2. В заданной (рис. 4.39) системе координат
запишем
координаты
вершин
пирамиды
A(0,5; −0,5; 1), B(0,5; 0,5; 0), C(−0,5; 0,5; 1), M(0; 0; 1).
−−→
−−−→
−−→
При этом AB=(0; 1; 0), CM=(0,5; −0,5; 1), AB=(0; 1; 0),
−−−→
CM=(0,5; −0,5; 1). Параметрические уравнения прямых
AB и CM имеют вид соответственно x=0,5; y=t; z=0
и x=−0,5p; y=0,5p; z=−p+1. Пусть LN — общий перпендикуляр к AB и CM, причем L∈AB, N∈CM, L(0,5; t; 0),
−−→
N(−0,5p; 0,5p; −p+1). Тогда LN=(0,5(p+1); t−0,5p; p−1).
−−→ −−→ −−→ −−−→
−−→ −−→
−−→ −−−→
Поскольку LN⊥AB, LN⊥CM, имеем LN·AB=0, LN·CM=0.
Отсюда t−0,5p=0, 0,25(1+p)−0,5(t−0,5p)+p−1=0⇒t=0,3,

−−→
−−→
p=0,6. LN=(0,8; 0; −0,4)=0,4(2; 0; −1)⇒|LN|=0,4·
5. Итак,

расстояние между AB и CM равно 0,4· 5, координаты
точек L(0,5; 0,3; 0) и N(−0,3; 0,3; 0,4) полностью опреде-

140

Глава 4. Перпендикулярность прямой и плоскости

ляют положение общего перпендикуляра LN к прямым
AB и CM в пространстве.

О т в е т: 0,4· 5.
11(24.27). Правильная треугольная пирамида рассечена
плоскостью, перпендикулярной основанию и делящей две
стороны основания пополам. Определите площадь сечения
пирамиды этой плоскостью, если известно, что сторона
основания равна a, а высота пирамиды равна H.
Решение. В правильной треугольной пирамиде ABCT
(рис. 4.40) высота TO делит каждую из медиан основания в отношении 2:1. Плоскость сечения проходит
через прямую EH, параллельную высоте, и делит медиa
ану основания на две равные части. Поэтому KP=

2
√a
TO OC
4
и
=
= 3 = . Тогда площадь сечения вычисляется
EH HC √a 4 3
3
1 3
1
3
по формуле · H· a=
aH.
2 4
2
16
T

E
C

K
H

P

O
A

B

Рис. 4.40.

О т в е т:

3
aH.
16

12(24.28). Найдите плоский угол при вершине правильной
четырехугольной пирамиды, если:
а) двугранный угол при основании равен α;
б) двугранный угол при боковом ребре равен α.
Решение.
а) Примем
ния пирамиды
за 1.

длину
Тогда

стороны
(рис. 4.41)

основа△OPM:

§ 24. Двугранный угол. Угол между плоскостями 141
M

E
C

K
P

O
A

B

Рис. 4.41.


1
1 p 2
OM=OP·tg α= ·tg α. △OMB: MB= OM2 +OB2 =
tg α+2.
2

2

1
BP
△MBP: ∠BMP=arcsin
. Тогда ∠BMP=arcsin 1 p 2
=
MB
tg2 α+2
2
1
. Искомый угол
вычисляется по фор=p
tg2 α+2
1
муле ∠BMC=2 arcsin p
. Либо для вычисле2
tg α+2

ния величины этого угла воспользуемся теоремой
косинусов. △BMC: BC2 =2·BM2 −2·BM2 ·cos ∠BMC⇒1=
=2·BM2 (1−cos ∠BMC)⇒

=arccos

tg2

tg2

α

α+2

2

tg2 α+2

=1−cos ∠BMC⇒∠BMC=

.

б) Для вычисления величины угла BMC воспользуемся равенством треугольников BEO и KEO.
α
В треугольнике BEO: ∠BEO= , угол ∠EOB пря-

2
OB
BE
BE=
△BME:
sin ∠BMC=
,
мой,
отсюда
α;
BM
sin
2

2
BP
sin ∠BMC=
; △BMP: BM=
. Учиα
∠BMC
2 sin ·BM
sin
2
2
∠BMC
∠BMC
·cos
, получим
тывая, что sin ∠BMC=2 sin
2
2

∠BMC
2
cos
=
α ;
2
2 sin
2
tg2 α

О т в е т: а) ∠BMC=arccos

tg2 α+2

,

142

Глава 4. Перпендикулярность прямой и плоскости

б) ∠BMC=2 arccos



2

2 sin

α .
2

13(24.29). Через вершину C основания правильной треугольной пирамиды MABC проведена плоскость перпендикулярно боковому ребру MB. Эта плоскость составляет
с плоскостью основания угол, косинус которого равен 2/3.
Определите косинус угла между боковыми гранями пирамиды.
Решение. Требуется определить косинус угла AKC, полученного в сечении плоскости, перпендикулярной ребру
MB, боковыми гранями пирамиды (рис. 4.42).
M
K
B
α
T
A

C

O

Рис. 4.42.

Примем длину ребра AB пирамиды за 1. BO=
1

3 2
1
CO
3
= 2 = .
⇒OK=OB·cos α= · = √ ⇒tg ∠CKO=
1
2 3
OK
2
3

 √ 2
3
3
1−
2
1
∠CKA=2∠CKO⇒cos ∠CKA=  √ 2 = .
7
3
1+
2




3

2

О т в е т: 1/7.
14(24.30). В правильной четырехугольной пирамиде двугранный угол при основании равен α. Найдите двугранный
угол при боковом ребре.
Решение. Примем длину стороны основания пирамиды
ABCKM за 1 (рис. 4.43).
△MOE:

MO= 1 tg α;
2

§ 24. Двугранный угол. Угол между плоскостями 143
M

T
C

K
E

O
A

B

Рис. 4.43.

q
p
p
2
1 = 1 tg2 α+2;
△MOC: MC= MO2 +OC2 = 1
tg
α+
4
2 2
OT⊥MC⇒OT·MC=MO·OC;
p
tg α
OT· 1 tg2 α+2= 1 tg α· √1 ⇒OT= p
;
2

△BKT:

2

2(tg2 α+2)

OK 2
1−
OT
∠BKT=2∠OTK⇒cos BTK=
 ;
OK 2
1+
OT
2

1
√ p
2
OK = 2 · 2(tg2 α+2)⇒cos ∠BTK= −2 ctg α =− cos2 α.
OT tg α
2(1+ctg2 α)

Тогда ∠BKT=arccos(− cos2 α)=π−arccos(cos2 α).
О т в е т: π−arccos(cos2 α).

15(24.31). Основание пирамиды — прямоугольный треугольник с катетами 6 и 8. Все двугранные углы при
основании пирамиды равны 60◦ . Найдите высоту пирамиды.
Решение. Поскольку все двугранные углы при основании
пирамиды равны, ее высота, опущенная из вершины M,
проходит через центр окружности, вписанной в прямоугольный треугольник основания (рис. 4.44).
Найдем радиус OK вписанного в треугольник ABC круга
по формуле OK·(AB+BC+AC)=AB·BC⇒OK(6+8+10)=48⇒
⇒OK=2. Рассмотрим прямоугольный
треугольник OKM:

∠OKM=60◦ ⇒OM=OK·tg 60◦ =2 3.

144

Глава 4. Перпендикулярность прямой и плоскости
M

C

O

A

K

E

B

Рис.4.44.


О т в е т: 2 3.
16(24.32). Через диагональ BK квадрата, лежащего в основании прямого параллелепипеда, и вершину другого
основания параллелепипеда проведена плоскость так, что
в сечении получился треугольник BKC1 с углом при вершине C1 , в два раза большим, чем угол между плоскостью
сечения и основанием параллелепипеда. Найдите угол
BC1 K.
Решение. Обозначим ∠COC1 =α; ∠KC1 O=β и примем длину стороны AB квадрата за 1. Поскольку
OC=OB=OK (рис. 4.45), из треугольника OCC1 получаем
1
1
=√
. Так как BO⊥OC, то по теореме
cos α
2 cos α
OK
1 √
о трех перпендикулярах OC1 ⊥OK⇒tg β=
= √ · 2 cos α=
OC1
2

OC1 =OC·

=cos α. ∠BC1 K=2β, отсюда, в соответствии с условием
задачи, β=α. Найдем величину β из уравнения tg β=cos β.
B1

C1

A1
K1
B
C
O
A

K

Рис. 4.45.

§ 24. Двугранный угол. Угол между плоскостями 145
sin β



5−1
=1⇒sin
β)⇒sin β=
⇒β=arcsin
2
2
cos β

5−1
Отсюда ∠BC1 K=2 arcsin
.
2

5−1
.
О т в е т: 2 arcsin
2

β=(1−sin2



5−1
.
2

17(24.33). Через диагональ куба с ребром 1 проведена
плоскость так, что полученное сечение имеет наименьшую
сумму квадратов сторон. Найдите угол между плоскостью
сечения и основанием куба, а также площадь сечения.
Решение. Любое сечение куба, проходящее через его диагональ, представляет собой параллелограмм, одна из диагоналей которого является диагональю куба. Из планиметрии
известно, что сумма квадратов сторон параллелограмма
равна сумме квадратов его диагоналей. Отметим, что задача
сводится к минимизации длины второй диагонали сечения.
При этом если хотя бы одна точка сечения (отличная от
вершин A и C1 ) лежит на ребрах, сходящихся к A или C1 ,
то сечение представляет
собой прямоугольник, диагонали

которого равны 3. Рассмотрим остальные случаи. Для
этого рассмотрим прямую BB1, введем на ней вектор BB1
(рис. 4.46). Возможны три случая.

E
C1
B1
K

T1
A1

C

B

P
T

A

Рис. 4.46.

146

Глава 4. Перпендикулярность прямой и плоскости

1. Плоскость пересекает прямую BB1 в точке E, расположенной на прямой BB1 выше точки B1 . Сечение
AKC1 P в этом случае представляет собой параллелограмм,
у которого наименьшее значение длины диагонали KP
равно расстоянию между
параллельными прямыми TC

и A1 B1 и составляет 2, т. е. равно AC. При этом AKC1 P —
ромб площадью
S= 1
·AC1 ·KP= 1
2
2

C

B

P

K1

H
T

A

Рис. 4.47.

√ √ √6
3· 2= 2 .

Отметим, что точки K и P являются
серединами ребер TC и A1 B1 . Для
вычисления угла между секущей плоскостью и основанием опустим из точки
K перпендикуляр на ребро AB. Основанием этого перпендикуляра будет
середина ребра AB — точка K1 , длина
его будет равна 1 (рис. 4.47). Проведем
через точку K1 перпендикуляр K1 H
к линии пересечения секущей плоскости с основанием. В прямоугольном
треугольнике APK1

1
AK1 · PK1 = AP · K1H = 1
· 1 ⇒ K1 H = r 2
= √1 .
2
5
1
+1
4

По теореме о трех перпендикулярах,
угол наклона сечения

к основанию куба равен arctg 5.
2. Плоскость сечения пересекает прямую BB1 в точке E,
лежащей на отрезке BB1 (рис. 4.48). Сечение AEC1 P —
параллелограмм, у которого длина диагонали EP принимает наименьшее значение, когда EP — расстояние между
прямыми TT1 и BBB1 . При этом точки E и P — середины
ребер TT1 и BB
, сечение CAE1 P — ромб и его площадь
√ 1
также равна

6
.
2

§ 24. Двугранный угол. Угол между плоскостями 147

C1
B1
T1
A1

E

P
C

B

T

A

Рис. 4.48.

Для определения угла между секущей плоскостью
и основанием отметим, что их линией пересечения будет
прямая l, проходящая через точку A параллельно BT
(рис. 4.49). Опустим из точки
√ B перпендикуляр BH на
2

1

, EB =
и тангенс угла
прямую l, при этом BH =
2
2
между плоскостью сечения и основанием вычисляется как
1

EB
1
2
= √2 = √ , сам угол будет равен arctg
.
BH
2
2
2
2
C

B

H
T

A

Рис. 4.49.

3. Пусть теперь точка E лежит на прямой BB1 ниже
точки B. (рис. 4.50). Как и ранее, диагональ KP параллелограмма AKC1 P будет иметь наименьшую длину, если
KP будет расстоянием между ребрами BC и A1 T1 , точки K
и P будут серединами ребер BC и A1 T1 . Площадь сечения

148

Глава 4. Перпендикулярность прямой и плоскости


6

сохранит при этом прежнее значение
, и расчет угла
2
наклона сечения к основанию аналогичен первому случаю,

причем величина этого угла сохранит значение arctg 5.
C1
B1
T1

A1

P

K

C

T

A

B
E

Рис. 4.50.

О т в е т:





6
2
; arctg 5 или arctg
.
2
2

18(24.35). Правильная четырехугольная пирамида со стороной основания, равной 1, и двугранным углом при
основании, равным 2α, пересекается плоскостью, делящей
пополам угол при основании. Найдите площадь сечения.
Решение. В правильной четырехугольной пирамиде
ABCEM сечение ENLC, проходящее через ребро EC
основания, пересекает противолежащую боковую грань
AMB по прямой NL, параллельной этому ребру (рис. 4.51).
Сечение представляет собой равнобедренную трапецию,
1

площадь которой равна
(EC + NL) · PK. Высота PK
2
трапеции лежит в плоскости, проходящей через высоту
пирамиды MO и середины ребер EC и AB. Угол MPO равен
2α. Апофема MP грани ECM вычисляется из треугольника
1
. По свойству биссектрисы
2 cos 2α
MK
MP
1
=
=
;
внутреннего угла треугольника,
KH
PH
2 cos 2α
MO MH tg 2α MK +KH 1+2 cos 2α
△OMH ∼△TKH ⇒
=
=
=
=
.
KT
KH
2KT
KH
2 cos 2α

MPO по формуле MP =

§ 24. Двугранный угол. Угол между плоскостями 149
M

N

K

A

T

L

H

B

O
P

E

C

Рис. 4.51.

KT =

Отсюда
ника

KPH

ники

AMB

=

MK
=
MK +KH

=

1
2

tg 2α ·2 cos 2α
sin 2α
=
, а из
2(1+2 cos 2α) 1+2 cos 2α
KT
2 cos α
имеем PK =
=
.
sin α 1+2 cos 2α

и

NML

подобны,

поэтому

треугольТреугольNL MK
=
=
AB MH

1
1
1
=
⇒SELNC = (NL+EC)·PK =
KH
1+2 cos 2α
2
1+
MK

1
cos α
+1
. После преобразований полу1+2 cos 2α
1+2 cos 2α

чим: SELNC =
О т в е т:

4 cos2 α

(1+2 cos 2α)2
4 cos2 α

(1 + 2 cos 2α)2

.

.

19(24.36). Дан куб ABCEA1 B1 C1 E1 . Найдите угол между
плоскостью EE1 C1 и плоскостью, проведенной через AC
и середину H ребра A1 E1 .
Решение. Способ 1. Примем длину ребра куба за 1
и построим линию пересечения плоскости ACH и ECC1
(рис. √
4.52): ACH ∩ ECC1 = TC; △ETC ∼ △E1 TM ⇒ ET = 2;
TC = 5.
В треугольнике ETC из вершины E опустим перпендикуляр EP на TC. По теореме о трех перпендикулярах,
AE ⊥ ECC1 ; AP ⊥ TC ⇒ ∠APE — линейный угол двугранного угла ETCA.

150

Глава 4. Перпендикулярность прямой и плоскости
T
z

B1

C1
M

A1

E1

H

P

y
B

C
E

A

x

Рис. 4.52.

В треугольнике TEC имеем EC · ET = EP · TC ⇔ 1 · 2 =

2
= 5 · EP ⇒ EP = √ . Тогда в прямоугольном треугольнике
5




5
5
AEP найдем tg ∠APE =
⇒ ∠APE = arctg
.
2
2

Способ 2. Введем систему координат с началом в точке
A (рис. 4.52). Вектор ~i = (1; 0; 0) является нормальным вектором для плоскости ECC1 . Найдем нор~ для плоскости сечения. Пусть
мальный вектор n



−−→
−→
1
x
~ = (x; y; 1). n
~ ⊥ AH; −
~ · AH
n
AH = ( ; 0; 1) ⇒ n
= 0 = + 1,
2
2
−−→
−−→
−→
~ ⊥ AC;
~ ·−
n
AC = (1; 1; 0) ⇒ n
AC = 0 = x + y ⇒ x = −y. Отсюда
~ = (−2; 2; 1). Поскольку угол между плоскостями равен
n
острому углу между их нормальными векторами,
~ · ~i|
|n
= 2 ⇒ ∠APE = arccos 2 .
3
~
|n| · 1 3

2
5
О т в е т: arccos
или arctg
.
3
2

cos ∠APE =

20(24.37∗ ). Сторона AM равностороннего треугольника
AMK образует с некоторой плоскостью x угол α, а другая
сторона MC — с той же плоскостью угол β. Найдите угол
между плоскостью треугольника и плоскостью x.
Решение. Примем сторону равностороннего треугольника
AMK за 1. Пусть A ∈ x. Опустим из точек M и K

§ 24. Двугранный угол. Угол между плоскостями 151

перпендикуляры MB и KE на x; ∠MAB = α; ∠KAE = β.
Плоскость треугольника AMK пересекает плоскость x
по прямой AC, C = MK ∩ BE (рис. 4.53). Из точки B
проведем перпендикуляр BT к прямой AC. По теореме
о трех перпендикулярах, углом между плоскостью x
и плоскостью AMK является угол MTB. В прямоугольном треугольнике MBT катет MB равен sin α.
Задача сводится к нахождению MT. В треугольнике
MBC из точки K проведем прямую KP параллельно
CB, △MPK ∼ △KEC, KE = sin β ⇒ MP = sin α − sin β;

MK
=
MP

KC
1
KC
sin β

=
⇒ KC =
⇒ MC = KC +
sin β
sin α − sin β
sin β
sin α − sin β
sin α
+1=
. В треугольнике AMC угол M равен 60◦ ;
sin α − sin β

=

по теореме косинусов

AC2 = 1 + MC2 − 2MC cos 60◦ ⇒
⇒ AC2 = 1 +
⇒ AC2 =

sin2 α
sin α


(sin α − sin β)2 sin α − sin β

sin2 α + sin2 β − sin α · sin β
.
(sin α − sin β)2

MC

AC

Так как для треугольника AMC верно, что
=
,
sin A
sin 60◦
то
r
sin2 α + sin2 β − sin α · sin β 2
sin α
=
·√ ;
2
sin A · (sin α − sin β)

(sin α − sin β)

M

P
B

A

K

E
T

C

Рис. 4.53.

3

152

Глава 4. Перпендикулярность прямой и плоскости

sin2 A =

4(sin2

3 sin2 α
;
α + sin2 β − sin α sin β)

p
2 sin2 α + sin2 β − sin α · sin β
MB

sin ∠MTB =
=
.
MT
3
p
2 sin2 α + sin2 β − sin α · sin β

О т в е т: sin ∠MTB =
.
3

21(24.39). Сторона основания правильной четырехугольной пирамиды ABCKM равна 1, а плоский угол при
вершине M равен α. Определите расстояние между скрещивающимися ребрами этой пирамиды.
Решение. В правильной пирамиде ABCKM (рис. 4.54)
угол KMC равен α и сторона основания равна 1. Найдем
расстояние ρ(KC, MB) между скрещивающимися прямыми
KC и MB. Поскольку прямая KC параллельна AB,
ρ(KC, MB) можно вычислить как расстояние от прямой CK
до плоскости AMB. Проведем плоскость OEM через высоту
OM пирамиды и середину отрезка KC — точку E. Плоскость OME перпендикулярна ребру AB пирамиды, значит,
она перпендикулярна грани AMB. Из точки E опустим
перпендикуляр на прямую MP, ET = ρ(KC, MB). В треугольнике PME имеем PE · MO = MP · ET ⇒ MP = ME =
=

EK
1
α =
α ;
tg
2 tg
2
2

M

T
B
P

C
O

E
K

A

Рис. 4.54.

§ 24. Двугранный угол. Угол между плоскостями 153

s

2

q

· MO =
MO = ME − 1
=1
ctg2 α
− 1 ⇒ ET = PEMP
2
2
2
r
α
α

q
ctg2 − 1 · 2 tg
2
2
2 α = cos α .
=
=
1

tg
α
2
2
cos
2

cos α
О т в е т:
α .
cos
2
2

22(24.40). Дан куб ABCEA1 B1 C1 E1 с длиной ребра 1.
Точки M и K — середины ребер CE и CC1 соответственно.
Найдите расстояние между прямыми AK и BM.
Решение.
Способ 1.
Для
нахождения
расстояния
ρ(AK, BM) между прямыми AK и BM (рис. 4.55) проведем
через точку A прямую AH параллельно BM (H = AH ∩ EC).
Так как прямая BM параллельна плоскости AKH,
то расстояние между AK и BM равно расстоянию от
точки M до плоскости AHK. Для упрощения выкладок
обозначим плоскость AKH через α, расстояние от
точки M до плоскости AKH — ρ(M, α). Поскольку M ∈ CH;
2

2

MH = CH, получаем ρ(M, α) = ρ(C, α). Зададимся
3
3
целью найти ρ(C, α). Опустим из точки C перпендикуляр
CT на прямую AH. В прямоугольном треугольнике KCT
расстоянию ρ(C, α) равна высота, опущенная на KT из

B1

C1

A1
E1

K

B

C
M

A

T

E
H

Рис. 4.55.

154

Глава 4. Перпендикулярность прямой и плоскости
C

B

M

точки C. Для нахождения CT испольCT · AH = CH · AE
зуем
равенство
(рис. 4.56):
s
2
CT · 1 + 1 = 3 · 1 ⇒ CT = √3 .
2

2

5

E

A

Из треугольника KCT получаем:

T
H

Рис. 4.56.



ρ(C, α) · KT = CT · KC;

ρ(C, α)· CT 2 +KC2 =CT ·KC⇒
3

⇒ρ(C, α)= √
2

ρ(M, α)= √
Тогда ρ=

KC·SAHM
SAHK

41

·S

;

AHM
41
8
2
= √ =√ .
4 41
41

1
2

= ·AE·HM=

1
.
2

Способ 2. Введем систему координат (рис. 4.57):
z

B1

C1

A1

E1

K

y
B

C
E

A

M
x

Рис. 4.57.







−−−→
1
1
1
B(0, 1, 0), M 1, , 0 , K 1, 1,
; BM= 1; − ; 0 ,
2
2
2


−−→
−−→
1
~ перпендикулярный
AK = 1, 1,
, AB=(0, 1, 0). Вектор n,
2

−−−→
векторам BM,
−−→
~ −
Тогда n·
BM=0;

−−→
~
AK, будем искать в виде n=(x,
y, 1).



1
1
~
n· AK =0⇒x− y=0; x+y+ =0. Отсюда
2

2

§ 24. Двугранный угол. Угол между плоскостями 155


1
1
~
n= − ; − ; 1 ;
6

3

−→ ~
−−→ |−
AB· n|
ρ(BM, AK)=|Прn~ AB|=
=r
~
|n|

2

О т в е т: √

41

1
3
= √2 .
41
1
4 36
+
+
36 36 36

.

22(24.41∗ ). На прямой p в пространстве последовательно
расположены точки A, B, C такие, что AB=27, BC=18.
Найдите расстояние между прямыми p и q, если расстояния от точек A, B, C до прямой q равны 17, 10, 8
соответственно.
Решение. Рассмотрим положение прямых p и q в декартовой системе координат. Не нарушая общности рассуждений, предположим, что прямая q лежит на оси OZ,
а прямая p пересекает ось OY в точке A (рис. 4.58).
z
C
B
y

O
A
B1
x

K

C1

Рис. 4.58.

Спроецируем на плоскость XOY точки B и C:
B1 =ПрXOY B; C1 =ПрXOY C. Так как прямая q параллельна
плоскости ACC1 и прямая p лежит в плоскости ACC1 ,
то расстояние между прямыми p и q равно длине
перпендикуляра OK, опущенного из точки O на прямую
C1 A. Зададимся целью вычислить длину OK. Поскольку
CC1 kq и C1 O⊥q, расстояние ρ(C, q) от точки C до прямой q
равно ρ(C1 , q), т. е. ρ(C1 , q)=C1 O=8. Аналогично рассуждая, получим B1 O=10 и AO=17. В силу параллельности
прямых CC1 и BB1 в прямоугольном треугольнике CC1 A

156

Глава 4. Перпендикулярность прямой и плоскости

имеют

место

равенства

C1 B1 CB 18 2
=
= = .
B1 A BA 27 3

Применяя

теорему косинусов к треугольникам OB1 C1 и OB1 A,
получим: OA2 =OB21 +B1 A2 −2B1A·OB1 ·cos ∠OB1A и OC21 =
Введем
=OB21 +C1 B21 −2·OB1 ·C1 B1 ·cos(180◦ −∠OB1A).
обозначения: C1 B1 =2·k, B1 A=3·k, ∠OB1 A=α, тогда
289=9k2 +100−60·k·cos α, 64=4·k2 +100+40·k·cos α. Из
двух последних равенств найдем C1 A=5·k=15. Это
означает, что ∠OAC1 =90◦ ⇒ρ(O, CC1 A)=8.
О т в е т: 8.
23(24.42). Через диагональ куба с ребром 1 проведена
плоскость. Найдите площадь получившегося сечения, если
известно, что она имеет наименьшее возможное значение.
Решение. Сечение куба, проходящее через диагональ
BK1 , представляет собой параллелограмм BTK1 E, площадь
которого равна удвоенной площади треугольника BTK1.
В свою очередь площадь треугольника BTK1 зависит только
от высоты TP этого треугольника. При этом точка T
принадлежит либо ребру AA1 (рис. 4.59), либо ребру AK
(рис. 4.60) куба.
K1

E

A1
P

B1

K

B

Рис. 4.59.

C1
E

A1

B1

P

C

T
A

K1

C1

T
A

C

K
B

Рис. 4.60.

В обоих случаях отрезок TP имеет наименьшую длину,
когда является расстоянием ρ между скрещивающимися
прямыми: в первом случае A1 A и BK1 , во втором —
1
AC, так
2
1
как AC⊥KBB1; если же T∈AK, то ρ=ρ(A, A1 BC)= AB1 ,
2

AK и BK1 . Если T∈AA1, то ρ=ρ(A, KBB1 )=

§ 24. Двугранный угол. Угол между плоскостями 157

2
так как AB1 ⊥A1 BC. И в том, и в другом случае ρ= .
2

Учитывая,
что в кубе с длиной ребра 1 длина диагонали

BK1 = 3, окончательно получим √для наименьшей площади
SEBTK сечения куба величину
1

О т в е т:



6
.
2

6
.
2

24(24.43∗ ). Дан куб ABCEA1 B1 C1 E1 с длиной ребра 1.
Прямая E1 O проходит через вершину E1 и центр O
грани BCB1C1 . Найдите длину наименьшего отрезка,
середина которого находится на прямой E1 O, а концы —
в плоскостях ABC и BCB1.
Решение. Введем систему координат,
как показано на рис. 4.61. Точка T — середина отрезка KL,
для которого K ∈BCC1, L∈ABC, T∈E1 O. При этом

1 1
L(xL , yL , 0),
K(1, yK , zK ),
O 1, ,
,
E1 (0,1, 1).
2

Запишем

каноническое

x y−1 z−1
=
=
=t,
1 −1
−1
2
2

t∈R.

2

уравнение

Поскольку

прямой

точка

T

OE1 :

лежит

на

прямой OE1 , ее координаты (x, y, z) можно записать
1

1

так: x=t; y=1− t; z=1− t. Учтем равенство векто2
2

−−→ −−→
−−→
t
t
ров LT= TK. При этом TK = 1−t; yK −1+ ; zK −1+ ;
2
2

−−→
t
t
LT= t−xL; 1− −yL ; 1−
.
2

2

z

E1

A1

C1

y

O
T

E

A

K

B1

B

Рис. 4.61.

x

C
L

158

Глава 4. Перпендикулярность прямой и плоскости

Получим систему уравнений:
8
8
> 1−t=t−xL,
>
>
>
< 2t=1+xL,
< y −1+ t =1− t −y ,
t=2−yK −yL , ⇒
K
L

2
2
>
>
>
>
: t=2−z
>
: z −1+ t =1− t
K
K
2
2
8
>
q
>
< xL =2t−1,
−−→
⇒ > t=2−yK −yL , ⇒|LK|= (1−xL )2 +(yK −yL )2 +z2K =
>
: z =2−t
q K
= (2−2t)2 +(yK −yL )2 +(t−2)2.
Выражение
под знаком корня преобразуем к виду
s
2
6
4
5 t−
+ + (yK − yL )2 . Ясно, что наименьшее значение
5
5
r
−−→
−−→
4
2
или |LK| = √ .
величины |LK| =
5

О т в е т:

5

2
√ .
5

§ 25. ОРТОГОНАЛЬНОЕ ПРОЕКТИРОВАНИЕ

Главным инструментом в решении задач по данной теме
выступает теорема об ортогональной проекции плоского
многоугольника.
Она используется для нахождения угла между плоскостями, нахождения площади сечения многогранника,
исследования формы и размеров проекции плоских и объемных фигур. В начале изучения этого материала полезно
предложить учащимся в устной форме ответить на следующие вопросы 1) .
1. Может ли площадь проектируемой фигуры быть
меньше площади проекции этой фигуры?
2. Верно ли, что ортогональной проекцией прямоугольного треугольника является прямоугольный треугольник?
3. В каком случае площадь фигуры равна площади
ортогональной проекции этой фигуры?
1) В качестве вопросов, способствующих эффективному изучению
теории ортогонального проектирования, полезно использовать многие
из устных задач (№ 1–15) к § 25 из учебника.

§ 25. Ортогональное проектирование 159

Особое внимание рекомендуется уделить задачам, связанным с прямоугольным тетраэдром, т. е. треугольной
пирамидой, у которой все плоские углы при вершине
прямые.
При решении задач на прямоугольный тетраэдр часто
используется координатный метод, так как ребра такого
тетраэдра образуют прямоугольную систему координат
с центром в вершине тетраэдра. Вообще говоря, в задачах
на ортогональное проектирование во многих случаях
координатный метод играет важную роль. Он позволяет,
не проводя сложных построений, оформить решение в виде
простых и понятных выкладок. Другим важным моментом
изучения темы является проектирование на грани двугранного угла фигур, лежащих в биссекторной плоскости этого
угла. При решении ряда таких задач важно отметить, что
любая прямая в пространстве, которая образует одинаковые углы с парой пересекающихся плоскостей, обязательно
будет параллельна одной из биссекторных плоскостей
этой пары. Это теоретическое утверждение стоит отдельно
доказать на занятии. Использование данного факта дает
возможность уйти от громоздких геометрических построений и ограничиться лишь чертежами ортогональных
проекций, как это показано в решении задачи 7.
Интересен случай, когда двугранный угол является
прямым. Здесь сочетание метода координат с успешным
применением тригонометрии в планиметрических «подзадачах» дает быстрый и красивый результат (задачи 10, 11).
Важно также остановиться на вопросе проектирования
объемной фигуры на прямую в пространстве (R3 → R1 ).
Каждой точке фигуры при этом ставится в соответствие ее ортогональная проекция на заданную прямую
(задача 8).
Упражнения и задачи для закрепления
1(25.7). Отрезок длины l находится на прямой, образующей
с некоторой плоскостью угол φ. Найдите длину ортогональной проекции данного отрезка на эту плоскость.
О т в е т: l cos φ.

160

Глава 4. Перпендикулярность прямой и плоскости

2(25.8). Найдите угол между гранями правильного тетраэдра.
Решение. Проекцией грани ABC на плоскость ADC
является треугольник AOC (рис. 4.62), где точка O —
центр
правильного
треугольника
ADC.
Так
как
S
1
1
S
= SПр△ABC и △AOC = cos φ ⇒ cos φ = , то
3 △ADC
S△ABC
3
1
φ = arccos .
3

S△AOC =

B

C
E
A

O
D

Рис. 4.62.

О т в е т: arccos

1
.
3

3(25.10). Пусть имеется некоторый прямоугольный тетраэдр ABCE, в котором плоские углы при вершине E —
прямые. Известно, что P — площадь его боковой грани
CAE, а Q — площадь основания ABC. Найдите площадь
ортогональной проекции грани CAE на основание.
Решение. Если в качестве основания прямоугольного
тетраэдра рассмотреть грань AEC (рис. 4.63), то косинус
P

угла φ между гранями ABC и CAE равен
, так как
Q
треугольник CAE является проекцией треугольника ABC.
Но это означает, что площадь проекции грани CAE на
грань ABC можно найти по формуле P · cos φ =
О т в е т:

P2
.
Q

P2
.
Q

§ 25. Ортогональное проектирование 161
B

E
Q
P
A

C

Рис. 4.63.

4(25.12). Диагонали ромба равны 10 и 4. Плоскость ромба
составляет с плоскостью проекции угол 60◦ . Найдите
площадь проекции ромба.
Решение.

Как

ABCE

ромба
1

известно

из

вычисляется

планиметрии,
по

формуле

площадь
S=

1
AC ×
2

× BE = 10 · 4 = 20. Тогда площадь проекции ромба равна
2
S = 20 cos 60◦ = 10.
О т в е т: 10.
5(25.13). Найдите ортогональную проекцию ромба с диагоналями 5 и 6, меньшая диагональ которого перпендикулярна плоскости проекции.
Решение. Так как диагонали ромба взаимно перпендикулярны, то вторая диагональ ромба параллельна плоскости проекций. Поэтому ортогональной проекцией ромба
является отрезок, длина которого равна длине второй
диагонали.
О т в е т: 6.
6. В правильной четырехугольной пирамиде TABCD через
ребро AB и середину ребра TC проведено сечение, площадь
которого в 9/8 раза больше площади основания. Найдите
величину угла между плоскостью сечения и плоскостью
основания, а также угол наклона бокового ребра данной
пирамиды.
Решение. Не нарушая общности рассуждений, примем
длину стороны основания пирамиды за 1. Сечение пред-

162

Глава 4. Перпендикулярность прямой и плоскости

ставляет собой равнобедренную трапецию ABKE, у которой верхнее основание KE в 2 раза меньше AB (рис. 4.65).
T

C

D
N

E

K
E

K

L
D

O

C

M
O
B

A

A

Рис. 4.64.

B

Рис. 4.65.



1
1 3
Вычислим площадь проекции сечения Sпр =
1+
=
2
2 4
9
= . Угол φ между плоскостью сечения и плос16
Sпр

костью

основания

найдем

9

по

9

формуле

8

1

Sсеч

=cos φ.

Поскольку Sсеч = , cos φ= · = ⇒φ=60◦ . В пря8
16 9 2
моугольном треугольнике √
TOL (рис. 4.64): NM ⊥OL,
1
1
3
TO, NM = tg 60◦ =
. Тогда
2
4
4

3
TO=2 ·tg 60◦ ·2=3 3. Отсюда угол
4
TO
=arctg
ребра пирамиды равен arctg
OB

NM =


О т в е т: 60◦ , arctg 3 6.

высота пирамиды

наклона бокового


3 3
=arctg 3 6.
√1
2

7. ABCA1B1 C1 — правильная треугольная призма, все
ребра которой равны между собой; K — точка на ребре AB,
отличная от A и B; M — точка на прямой B1 C1 ; L — точка
в плоскости грани ACC1 A1 . Прямая KL образует равные
углы с плоскостями ABC и ABB1A1 ; прямая LM образует
равные углы с плоскостями BCC1B1 и ACC1 A1 , прямая
KM также образует равные углы с плоскостями BCC1B1

§ 25. Ортогональное проектирование 163

и ACC1A1 . Известно также, что KL = KM = 1. Найти ребро
призмы.
Решение. Для доказательства используем утверждение
о том, что прямая, образующая с пересекающимися
плоскостями α и β одинаковые углы, параллельна одной
из двух биссекторных плоскостей двугранного угла (α; β).
Из условия задачи вытекает, что прямые MK и ML
параллельны биссекторным плоскостям двугранных углов,
с гранями ACC1 и BCC1 . При этом возможны два
случая: либо эти прямые параллельны одной биссекторной
плоскости, либо разным.
Рассмотрим первый случай.
Изобразим на чертеже ортогональную проекцию призмы на
основание ABC (рис. 4.66). Точки
M и L проектируются соответственно в точки M1 ; L1 .
Обозначим
KB
через
x.

Тогда KL1 = (a − x) 3; KL2 = KL21 +
+ LL21 = 3(a − x)2 + LL21 = 1.
Поскольку KL лежит в биссекторной плоскости двугран- A
ного угла с гранями ABC

и ABB1 , LL1 = KL1 = (a − x) 3.

L1

C

M1

K

B

1

Рис. 4.66.
Отсюда (a − x)2 = . В треуголь6
нике KM1 M, по теореме Пифа2
2 + a2 = 1.
гора, KM12 + MM12 =
8KM , следовательно, 3x


>
< (a − x)2 = 1
6 + 14
Решением системы >
.
6 , является a =
8
:3x2 + a2 = 1

Рассмотрим второй случай. Опять построим ортогональную проекцию призмы на основание
ABC

(рис. 4.67): KB = x; AL1 = 2x; LL1 = L1 L2 = 3x; (KL1 )2 =
= (KL2 )2 + (L1 L2 )2 = (a − 2x)2 + 3x2 ; (KL)2 = (LL1 )2 + (KL1 )2 =
= (a − 2x)2 + 6x2 . Учитывая, что KL = 1, получим 10x2 −
− 4ax + a2 = 1. Поскольку KM12 + MM12 = KM2 ; 3x2 + a2 = 1,

164

Глава 4. Перпендикулярность прямой и плоскости

C

A

L1

M1

L2

K

B

Рис. 4.67.

придем к системе
7

ется a = √
О т в е т:


10x2 − 4ax + a2 = 1,
Решением ее явля3x2 + a2 = 1.

.

97


7
6 + 14
√ ;
.
8
97

В задаче 8(25.11) используется одно важное свойство
прямоугольного тетраэдра: высота, опущенная из вершины, при которой все плоские углы прямые, попадает
в ортоцентр (точку пересечения высот) основания. Решение
задачи без использования этого свойства будет громоздким,
поскольку фактически будет включать в себя доказательство последнего.
8(25.11). В тетраэдре ABCE все плоские углы при вершине
E — прямые. EH — высота тетраэдра, причем, известно
отношение площадей

SAHB
S
= k. Найдите AEB .
SBHC
SBEC

Решение. Так как ∠BEC = ∠CEA = ∠BEA = 90◦, то
EABC — прямоугольный тетраэдр с высотой EH, следовательно, точка H является точкой пересечения высот
треугольника ABC. Значит, BT ⊥ AC, (T = BH ∩ AC).
Отсюда

k=

отношение

SABH
BH · AT
AT
=
=
SBCH
BH · CT
CT.

Необходимо

найти

SAEB
. В силу того что треугольники AEB
SBEC

и BEC являются прямоугольными, это отношение можно

§ 25. Ортогональное проектирование 165
BE · AE

AE

AE

представить в виде:
=
. Обозначим
= p.
BE · CE
CE
CE
Задача сводится к планиметрической. В прямоугольном
треугольнике из вершины прямого угла E проведена высота
AE

AT

ET. Надо найти отношение
, если
= k. Эта задача
CE
CT
легко решается рассмотрением подобных треугольников
AET, ETC, AEC.
p=k

AE
AT
ET
=
=
= p, AT = p · ET = k · CT,
EC
ET
CT


k
CT
= ⇒ p2 = k ⇒ p = k.
ET
p

8(25.26∗ ). Докажите, что площадь ортогональной проекции
куба с ребром 1 на любую плоскость равна длине проекции
куба на прямую, перпендикулярную этой плоскости.
Решение. Обозначим плоскость проекции через α,
а прямую, перпендикулярную этой плоскости, — через l
(рис. 4.68).
C1
C

D1

l

D
B1
A1
α

B
A

Рис. 4.68.

Проекция куба на прямую совпадает с проекцией на эту
прямую одной из диагоналей куба (в случае, показанном
на рис. 4.68, такой диагональю является AC1 ), а площадь
проекции куба на плоскость α равна сумме площадей
проекций трех взаимно перпендикулярных граней: ABCD,
ABB1 A1 и ADD1 A1 (так как проекция куба на плоскость α
совпадает с проекцией на ту же плоскость указанных трех
граней).

166

Глава 4. Перпендикулярность прямой и плоскости

−−−−→ −−−−→ −−−−→ −−−−→
Пусть A1 B1 , A1 D1 , A1 A11 , A1 C11 — векторы, для которых
точки A1 , B1 , D1 , A11 , C11 являются соответственно
проекциями точек A, B, D, A1 , C1 на прямую l. Поскольку
−−−−→ −−−−→ −−−−→ −−−−→
−−−→ −−→ −−→ −−−→
AC1 = AB + AD + AA1, имеем A1 C11 = A1B1 + A1 D1 + A1 A11 и,
−−−−→ −−−−→ −−−−→
в силу сонаправленности векторов A1 B1 , A1 D1 , A1 A11 ,
−−−−→
−−−−→
−−−−→
−−−−→
получим, что A1 C11 = A1 B1 + A1 D1 + A1 A11 , т. е. длина
проекции куба на прямую l равна сумме длин проекций на
ту же прямую трех его попарно перпендикулярных ребер
AB, AD и AA1 .
Обозначим углы между плоскостями граней ABCD,
ABB1 A1 , ADD1 A1 и плоскостью α через φ1 , φ2 , φ3
соответственно. Тогда площадь проекции куба равна
cos φ1 + cos φ2 + cos φ3 . Учитывая, что углы между ребрами AA1 , AD, AB и прямой l также равны
−−−−→
соответственно φ1 , φ2 , φ3 , получим, что |A1 C11 | =
−−−−→ −−−−→ −−−−→
= |A1 B1 | + |A1D1 | + |A1A11 | = cos φ1 + cos φ2 + cos φ3 , т. е. длина
проекции куба на прямую l равна площади проекции куба
на плоскость α.
9(25.27∗ ). Прямая, проходящая через одну из вершин
куба, образует с ребрами, исходящими из той же вершины, углы α, β,√ γ соответственно. Докажите, что
cos α + cos β + cos γ 6 3.
Решение. Будем считать, что ребро куба равно 1. Тогда
из решения предыдущей задачи следует, что сумма
cos α + cos β + cos γ равна площади ортогональной проекции
куба на плоскость, перпендикулярную прямой l, и равна,
в свою очередь, длине проекции куба на прямую l.
Так как проекция куба на прямую l совпадает с проекцией на эту√прямую одной из диагоналей куба, длина которой равна 3, а длина проекции диагонали не превышает

длины самой этой диагонали, то cos α + cos β + cos γ 6 3.
10(25.30∗ ). Прямоугольные проекции плоского четырехугольника ABCD на две взаимно перпендикулярные
плоскости являются квадратами со сторонами 1. Найдите

§ 25. Ортогональное проектирование 167

периметр четырехугольника,
если известно, что одна из

его сторон имеет длину 1,5.
Решение. Поскольку площади проекций четырехугольника
на две взаимно перпендикулярные плоскости равны, в соотSпр

ветствии с формулой
= cos φ плоскость четырехугольS
ника составляет равные углы с плоскостями проекций.
Отсюда заключаем, что четырехугольник лежит в биссекторной плоскости прямого двугранного угла. Введем
систему координат, как показано на рис. 4.69, и, не нарушая общности рассуждений, поместим в начало вершину A
четырехугольника. Обозначим квадраты, являющиеся его
проекциями на плоскости xAy и yAz соответственно, через
AB1 C1 D1 , AB2 C2 D2 .
z

C

y

B
E
D
x

A

Рис. 4.69.

Отметим, что BB1 k CC1 k DD1, BB2 k CC2 k DD2, и точка E
пересечения диагоналей четырехугольника ABCD проектируется в точки пересечения диагоналей соответствующих квадратов. Из сказанного вытекает, что диагонали четырехугольника ABCD делятся в точке E
пополам, следовательно, четырехугольник ABCD является параллелограммом. Кроме этого, поскольку четырехугольник лежит в биссекторной плоскости, для его
вершин имеют место равенства yB = zB , yC = zC , yD = zD .
r
Пусть AD =
ADD1

3
,
2

тогда в прямоугольном треугольнике
r
r
p
3
1
2
2
имеем zD = DD1 = AD − AD1 =
−1=
. Рас2

2

168

Глава 4. Перпендикулярность прямой и плоскости

B1

y

D1
F

x

A

Рис. 4.70.

смотрим △B1 AD
r 1 на плоскости xAy (рис. 4.70). Поскольку
zD = yD = AF =

1
, где F — точка пересечения диагоналей
2

относиквадрата AB1 C1 D1 , точки B1 , D1 симметричны
r
r
тельно оси ординат, т. е. yB = zB = BB1 =

1
⇒ AB =
2

3
. Это
2

означает, что параллелограмм
ABCD является
ромбом со
r
r
3

и периметр его равен 4
стороной AB =
2

О т в е т: 4 1,5.

3
.
2

11(25.31∗ ). Прямоугольные проекции треугольника ABC
на две взаимно перпендикулярные плоскости являются
правильными треугольниками со сторонами,
√ равными 1.
Медиана AD треугольника ABC равна
1,125. Найдите BC.
Решение. Введем прямоугольную систему координат
с началом в точке A (рис. 4.71). По аналогии с решением предыдущей задачи (25.30) можно утверждать, что
треугольник ABC лежит в биссекторной плоскости прямого
угла, образованного плоскостями проекций xAy, yAz
(рис. 4.72).
△AB1C1 = ПрxAy △ABC,
△AB2 C2 =
Обозначим:
= ПрyAz △ABC Отметим, что yB = zB ; yE = zE ; yC = zC .
В треугольнике AB1 C1 проведем медиану AE1 , которая
является проекцией на xAy медианы AE. Поскольку

3

треугольник AB1 C1 является равносторонним, AE1 =
.
2
Из треугольника AEE1 , rпо теореме
r Пифагора, следует,
p
9
3
3
что EE1 = AE2 − AE21 =
− =
. Но EE1 = zE = yE
8

4

8

§ 25. Ортогональное проектирование 169
y

z

B1

B

E1

yE

E
y

C1

C
x

A

Рис. 4.71.

p
и xE = AE21 − y2E =

x

A

Рис. 4.72.

r

3
3
− =
4
8

r

3
. Получили, что xE = yE ,
8

значит, AE1 — биссектриса первого координатного угла,
при этом угол наклона AC1 к оси абсцисс равен 15◦ .
Поэтому C1 (cos 15◦ ; sin 15◦ ), B1 (cos 75◦ ; sin 75◦ ), значит, C(cos 15◦ ; sin 15◦ ; sin 15◦ ), B(cos 75◦ ; sin 75◦ ; sin 75◦ ).
Используя тригонометрические формулы, координаты
точки B можно привести к виду B(sin 15◦ ; cos 15◦ ; cos 15◦ ).
Тогда
−−→
BC = (cos 15◦ − sin 15◦ ; sin 15◦ − cos 15◦ ; sin 15◦ − cos 15◦ )
и

q
p
p
BC = 3(cos 15◦ − sin 15◦ )2 = 3(1 − sin 30◦ ) = 3 .
2

О т в е т:



1,5.

12(25.33). Основание AC и вершина B
равнобедренного треугольника находятся на различных гранях прямого
двугранного угла с ребром l. Точки
A и B равноудалены от l на расстояние a, а проекция точки C на ребро l
равноудалена от проекций A и B на l.
Найдите расстояние от точки C до l,
если AB образует с l угол, равный 60◦ .

z
y

B
y
O

A
C
x

x

Рис. 4.73.

Решение. Введем систему координат (рис. 4.73) и, не нарушая общности рассуждений, положим, что точка B лежит
на оси аппликат, а ось ординат совпадает с прямой l. В соответствии с условием координаты вершин треугольника

170

Глава 4. Перпендикулярность прямой и плоскости

ABC имеют вид: B(0, 0, a), A(a, y, 0), C(x, y/2,
 0). Рассмот−−→
−−→
y
рим векторы BA = (a; y; −a), BC = x; ; −a , ~j = (0,1, 0).
2
r
2
p
−−→ −−→
y
|BA| = |BC| ⇒ a2 + y2 + a2 = x2 +
+ a2 ⇒ 4x2 = 4a2 + 3y2 .
4

−−→

−−→
1
|y|
|~j · BA|
|cos(~j, BA)| = ⇒ −−→ = p
⇒ 3y2 = 2a2 . Отсюда 4x2 =
2 + y2
2
2a
|
BA|
r
= 6a2 ⇒ x =
О т в е т:



3
a.
2

1,5a.

13(25.34∗ ). Двугранный угол между плоскостями p и q
равен α. В плоскости p лежит правильный треугольник со
стороной 1. Докажите, что сумма квадратов длин проекций
сторон этого треугольника на плоскость q не зависит от
его расположения в плоскости p.
Решение. Не нарушая общности рассуждений, будем
считать, что вершина C треугольника ABC лежит на
линии l пересечения плоскостей p и q (рис. 4.74), а треугольник A1 B1 C является проекцией треугольника ABC
на плоскость q. Опустим из точек A и B соответственно
перпендикуляры AF и BE на прямую l. По теореме о трех
перпендикулярах, ∠AFA1 = ∠BEB1. Тогда A1 C2 = AC2 −
− AA21 = 1 − (AF sin α)2 , B1 C2 = BC2 − BB21 = 1 − (BE sin α)2 ,
A1 B21 = AB2 − (AA1 − BB1)2 = 1 − (AF sin α)2 − (BE sin α)2 +
p
B

A

C

F

l

E
B1
q

A1

Рис. 4.74.

§ 25. Ортогональное проектирование 171

+2 · AF · BE sin2 α. Преобразуем выражение A1 C2 + CB21 +
+ A1 B21 к виду 3 − 2 · sin2 α(AF 2 + BE2 − AF · BE). Для сокращения записи обозначим ∠BCE = β.
Тогда BE = sin β;
AF = sin(180◦ − 60◦ − β) = sin(120◦ − β) =


= cos(30◦ − β) = 23 cos β + 1
sin β ⇒ AF 2 + BE2 − AF · BE =
2
2


2 β − 3 cos β sin β − 1 sin2 β =
= 23 cos β + 1
sin
β
+
sin
2
2
2


=3
cos2 β + 23 sin β cos β + 1
sin2 β + sin2 β −
4
4


sin2 β.
− 23 cos β sin β − 1
2
3
(cos2 β +
4
3
A1 C2 + CB21 + A1 B21 = 3 − 2 · sin2 β.
4

После приведения подобных членов получим
3

+ sin2 β) = . Отсюда
4
Таким образом, сумма квадратов проекций сторон треугольника ABC на плоскость q не зависит от положения
треугольника в плоскости p.

14(25.36∗ ). Двугранный угол между плоскостями p и q
равен β. В плоскости p лежит квадрат со стороной 1.
Докажите, что периметр проекции этого квадрата на
плоскость q наибольший, когда диагональ квадрата параллельна плоскости q.
Решение. Проекцией квадрата ABCE, лежащего в плоскости p, на плоскость q является параллелограмм, поэтому
для решения задачи достаточно рассмотреть проекцию
прямоугольного треугольника ABC на плоскость q и выяснить, при каких условиях сумма длин проекций его
катетов максимальна. Не нарушая общности рассуждений,
расположим треугольник ABC на плоскости p так, как
это показано на рис. 4.75. Пусть p ∩ q = l, BE ⊥ l, CT ⊥ l,
BB1 ⊥ q; CC1 ⊥ q. Тогда
BB21 = (1 − EA2) sin2 β,
CC21 = (1 − TA2) sin2 β,

172

Глава 4. Перпендикулярность прямой и плоскости

q
B

C

B
A
T
l E

C1

C

p

B1
E

A

Рис. 4.75.

T

Рис. 4.76.

B1 A2 = 1 − BB21 = 1 − sin2 β + EA2 sin2 β,
C1 A2 = 1 − CC21 = 1 − sin2 β + TA2 sin2 β,
B1 A2 = cos2 β + EA2 sin2 β,
C1 A2 = cos2 β + TA2 sin2 β.
Рассмотрим прямоугольную трапецию EBCT в плоскости p (рис. 4.76). Обозначим через α угол ∠CAT. Тогда,
поскольку AC = AB = 1, выполняются равенства:
TA = cos α,

EA = sin α ⇒ EA2 + TA2 = 1 ⇒

⇒ B1 A2 + C1 A2 = 2 cos2 β + sin2 βB1 A + C1A =
q
= B1 A2 + C1 A2 + 2B1A · C1A =
q
= 2 cos2 β + sin2 β + 2B1A · C1A.

Учитывая, что сумма B1 A2 + C1A2 является постоянной величиной, не зависящей от α, поставим
целью найти максимальное значение произведения
B1 A · C1 A. Для упрощения выкладок обозначим B1 A = x,
C1 A = y, B1 A2 + C1A2 = C, C = const. Используем известное
C2

C

тождество: x2 + y2 > 2xy ⇒ max(xy) = ⇒ max(x2 y2 ) =

2
4
2
2
2
2
2
2
2
⇒ 4x (C − x ) = C ⇒ (2x − C) = 0 ⇒ 2x = C;
2 cos β +
1

+ 2 sin2 β · sin2 α = 2 cos2 β + sin2 β ⇒ sin α = √ ⇒ α = 45◦ .
2

Отсюда следует, что максимальный периметр проекции
квадрата ABCE соответствует случаю, когда диагональ

§ 25. Ортогональное проектирование 173

квадрата BC параллельна
и требовалось доказать.

плоскости

проекций,

что

15(25.37∗ ). Докажите, что проекция правильного тетраэдра
на плоскость будет иметь наибольшую площадь, когда
эта плоскость параллельна двум скрещивающимся ребрам
тетраэдра, и найдите это значение площади для тетраэдра
с ребром a.
Решение. Проекция тетраэдра может быть треугольником
или четырехугольником. В первом случае она является
проекцией√ одной из граней, поэтому ее площадь не пре3

a2 , a — длина ребра правильного тетраэдра.
восходит
4
Во втором случае диагонали четырехугольника являются
проекциями ребер тетраэдра, поэтому максимальное зна1

чение площади проекции S = a2 . В этом случае пара
2
противоположных
√ ребер параллельна плоскости проекций.

3 2 1 2
a < a , получим, что наибольшая
4
2
1 2
площадь проекции правильного тетраэдра равна
a .
2
1 2
О т в е т:
a .
2

Учитывая, что

16(25.38∗ ). Сторона основания правильной треугольной
пирамиды равна 1, а боковые ребра наклонены к плоскости
основания под углом α. Площадь проекции пирамиды
на некоторую плоскость β имеет наибольшее значение.
Найдите величину этой площади.
Решение. Проекция правильной пирамиды на плоскость β
может быть представлена тремя случаями.
1. Проекция основания.
2. Проекция боковой грани.
3. Четырехугольник, диагоналями которого являются
проекции бокового ребра и ребра основания пирамиды.
Рассмотрим все три случая.
1. Значение площади
проекции √пирамиды выражается

формулой Sпр =

3
3
cos β ⇒ Sпр max =
.
4
4

174

Глава 4. Перпендикулярность прямой и плоскости

2. Для вычисления площади проекции вычислим пло1

щадь боковой грани (рис. 4.77) BEC: OE=OC tg α= √ tg α,
q

p

EK = OE2 +OK 2 =

3

1 tg2 α+ 1 ⇒
3
12

⇒SBEC =SПр max = 1 EK ·BC= 1
2
2

q

1 tg2 α+ 1 =
3
12

q
q
1
1 − 3,

tg2 α+ 1
=
4 2 3
cos2 α 4
2 3
q
1
1 − 3 cos β.
SПрBEC = √
cos2 α 4
2 3

1
= √

E

A

C
O

K

B

Рис. 4.77.

3. Максимальная площадь проекции выражается формулой Sпр

max

=

1
cos β
BC · AE = √
. Заметим, что вели2
2 3 cos α

чина площади проекции в этом случае всегда больше, чем
в случае 2.
Сравним
максимальные
площади проекций в случаях 1



3
1
3 1
1
2
− √
=

. Если cos α > , т. е.
4
2
2
3
cos
α
3
3 3 cos α
2
α 6 arccos , то ответом в задаче будет формула случая 2.
3
2
Если cos α < — ответ будет записан по формуле случая 3.
3


3
2
3
О т в е т:
при
0 < α 6 arccos ,
при
4
3
6 cos α
2
π
arccos < α < .
3
2

и 3:

§ 26. Векторное произведение векторов 175

§ 26. ВЕКТОРНОЕ ПРОИЗВЕДЕНИЕ ВЕКТОРОВ

При изучении данного параграфа важно выделить различие между скалярным и векторным произведением
векторов. Законы скалярного произведения устанавливают
соответствие между множеством векторов и множеством
чисел (каждой паре векторов ставится в соответствие
число). Векторное произведение отображает множество
векторов на себя, т. е. каждой паре векторов ставится
в соответствие вектор. При этом надо подчеркнуть, что,
по законам векторного произведения, любая пара линейно
независимых векторов порождает тройку линейно независимых векторов. Иными словами, векторное произведение отображает двухмерное пространство векторов
(плоскость) в трехмерное геометрическое пространство.
Наглядной демонстрацией тому является задача 5. Решение
ее определяет принцип перехода от декартовой системы
координат на плоскости к декартовой системе координат
в пространстве. Механизм векторного произведения играет
важную роль в физике (вопросы к § 26, задачи 1 и 4). Кроме
того, следует сразу доказать алгебраические свойства
векторного произведения, вытекающие из его определения:
~ ×b
~
~ = −b
~ × a,
1) a
~
~
~
~ ),
2) (λa × b) = λ(a × b
~
~
~
~
~ × ~c.
3) (a + b) × c = a × ~c + b
Упражнения и задачи для закрепления
~
1(26.1). Докажите, что если точку приложения силы F
переместить вдоль прямой действия силы, то момент силы
не изменится.
~ в точках A
Решение. Рассмотрим приложение вектора F
и B на прямой l (рис. 4.78).
l

E

~e1
O

A

~
F
~e2

Рис. 4.78.

B

~
F

176

Глава 4. Перпендикулярность прямой и плоскости

Пусть OE ⊥ l. Тогда
−−→ ~ ~
~ · |e~ | sin ∠OAE = |F|
~ · OE,
M1 = F × e1 = |F|
1
−−→ ~ ~
~ · |e~ | sin ∠OBE = |F|
~ · OE.
M2 = F × e2 = |F|
2
−−→ −−→
~ не зависит от точки
Поскольку M1 = M2, момент силы F
ее приложения на заданной прямой, что и требовалось
доказать.
2(26.2). Докажите, что точки A, B, C лежат на одной
−−→ −−→
прямой тогда и только тогда, когда AB × AC = ~0.

−−→ −−→ ~ −−→ −−→ −−→ −−→ −−→ ~
Решение.
× AC = 0 ⇔ AB × (AB + BC) = AB × BC = 0 ⇔
(−−→ −−→ AB
(
(
,
AB k AC, ,
,
sin ∠BAC = 0, ,
∠BAC = π,
,
,
,
−−→ ~
,
,
,
,
,
A

B,
∠BAC = 0, что и требова⇔,


AB
=
0,
,
,
,
,
*
* C ∈ AB,
−→ ~
A≡C
* −
AC = 0
лось доказать.
3(26.3). Докажите, что для любых векторов в пространстве
модуль их векторного произведения меньше или равен
произведению модулей этих векторов, причем равенство
~ ⊥ b.
~
достигается тогда и только тогда, когда a

~ b
~ — произвольные векторы, угол между
Решение. Пусть a,
~ × b|
~ · |b|
~ · |b|.
~ = |a|
~ sin α 6 |a|
~ Равенкоторыми равен α. Тогда |a
ство
при условии, что
( достигается
(
,
sin
α
=
1,
,
~
~
,
a ⊥ b,
,
,
,
,
~
,
~ = ~0. Так как нулевой вектор перпенa = 0, ⇔ ,aa
,
,
,
,
*b
~ = ~0.
* b
~ =0
~ × b|
~ · |b|
~ 6 |a|
~
дикулярен любому вектору, то утверждения |a
~
~
~
~
~
~
и |a × b| = |a| · |b| ⇔ a ⊥ b доказаны.
4(26.4). При каких условиях момент силы относительно
точки равен нулю?
~
~ равен 0,
О т в е т: а) вектор ~e плеча силы F
~
~
б) вектор силы F равен 0,
~ параллелен вектору плеча ~e.
в) вектор силы F

§ 26. Векторное произведение векторов 177

5(26.5). Используя непосредственно определение векторного произведения, найдите векторы
~
а) ~i × j;
~
г) 2j~× 3k;

б) j~× ~i;
д) j~× (j~+ ~k);

~
в) ~i × k;
~ − 3j)
~ × 6~i.
е) (2k

~ б) −k,
~ в) −j,
~ г) 6 ~i, д) ~i, е) 12j~+ 18k.
~
О т в е т: а) k,
6(26.6). Выясните, является ли тройка векторов левой или
правой, если:
~ = ~i + j~, b
~=k
~ , ~c = j~;
а) a
~ = ~j + k,
~ b
~ = ~i − j,
~ ~c = k;
~
б) a
~
~
~
~
~
~
в) a = k, b = 3j, c = −2i.
О т в е т: а) левая, б) левая, в) правая.
7(26.7). Докажите, что:
а) при перестановке двух соседних векторов из упоря~ b,
~ ~c ориентация всей тройки
доченной тройки векторов a,
меняется на противоположную;
б) при круговой перестановке трех векторов ориентация
не меняется.
Решение. Утверждение а) доказывается посредством чертежа (рис. 4.79). Для доказательства утверждения б) достаточно представить круговую перестановку (рис. 4.80) двумя
последовательными перестановками. Например, переста~ b,
~ можно выполнить, перейдя сначала
~ ~cк b,
~ ~c, a
новку от a,
~
~
~
~
~ При этом ориентация
~ c к b,
~ a, c, затем к b,
~ ~c, a.
от a, b,
тройки векторов дважды поменяется на противоположную,
т. е. вернется к исходной.
~c
~c
~b
~a
~a

~b

Рис. 4.79.

Рис. 4.80.

178

Глава 4. Перпендикулярность прямой и плоскости

~ b,
~
8(26.8). Докажите, что для любых трех векторов a,
~c, удовлетворяющих условию a
~ +b
~ + ~c = ~0, справедливы
~ ×b
~=b
~ × ~c = ~c × ~a.
равенства a
Решение. Учитывая свойства векторного произведения,
запишем равенства:
8
>
~ ×b
~ × (−a
~ − ~c) = ~0 − a
~ × ~c = ~c × ~a,
~=a

:a
~ =b
~ ×b
~ = (−~c − b)
~ ×b
~ = −~c × b
~−0
~ × ~c,
что и требовалось доказать.
9(26.9). Докажите, что в единичном кубе ABCDA1B1 C1 D1
−−−→ −−−→
вектор AB1 × AD1 равен вектору одной из диагоналей куба.

−−→ ~
Решение. Для краткости изложения обозначим AB = a,
−−→ ~ −−−→ ~
−−−→ −−−→ ~ ~
~ + ~c) =
AD = b, AA1 = c (рис. 4.81). AB1 × AD1 = (a + c) × (b




−→
~ ×b
~ × ~c + ~c × b
~ = −(~c + a)
~ +b
~+a
~ = −~c + b
~−a
~ = B A+−
=a
AD =
1
−−−→
= B1 D, что и требовалось доказать.
B1

C1

A1
D1

~c
~a
A

C

B

~b

D

Рис. 4.81.

~ b,
~ ~c компланарны
10(26.10). Докажите, что векторы a,
~ ×b
~ × ~c (или ~c × b)
~ иa
~
тогда и только тогда, когда векторы a
коллинеарны.
~ b,
~ ~c параллельны плоскости
Решение. Пусть векторы a,
~
~
~
α. Тогда векторы a × b и a × ~c коллинеарны, так как

§ 27. Векторное произведение векторов и его свойства 179

перпендикулярны плоскости α. Обратно, пусть векторы
~ ×b
~ × ~c коллинеарны, тогда каждый из векторов a,
~ b,
~ и a
~
a
~c параллелен плоскости, перпендикулярной векторам a
~ ×b
~
~ × ~c. Таким образом, векторы a
~ , c~ компланарны.
и a
~, b
§ 27. ВЕКТОРНОЕ ПРОИЗВЕДЕНИЕ ВЕКТОРОВ
И ЕГО СВОЙСТВА

Векторное произведение двух неколлинеарных векторов
широко используется в решении многих геометрических
задач, является важным инструментом аналитической
геометрии. С его помощью возможно:
1) найти нормальный вектор плоскости по двум векторам, параллельным этой плоскости,
2) найти общий перпендикуляр к двум скрещивающимся прямым,
3) найти вектор, параллельный двум пересекающимся
плоскостям,
4) найти площади треугольника и плоского четырехугольника, заданных своими вершинами в пространственной прямоугольной системе координат.
Так как применение векторного произведения в стереометрии в большинстве случаев сопряжено с координатным
методом решения задач, удобно вычислять векторное
произведение по канонам аналитической геометрии, т. е.
через определитель. Определители второго порядка используются в школьной программе для записи формул Крамера
решения линейных систем уравнений. Поэтому знакомство
с определителем третьего порядка послужит еще одной
«ступенькой» в понимании роли этой замечательной
конструкции в математике. Достаточно показать разло~
~i
j~ k
жение определителя ∆ = ax ay az по элементам перbx by bz
~
вой строки: ∆ = (ay bz − by az )~i − (ax bz − bx az )j~+ (ax by − bx ay )k.
Стоит обратить внимание на тот факт, что выражения,
стоящие в скобках, представляют собой определитель
второго порядка, который получается после вычеркивания

180

Глава 4. Перпендикулярность прямой и плоскости

соответствующего элемента из первой строки. Например,
коэффициент (ay bz − by az ) перед вектором ~i получается как
результат вычисления определителя после вычеркивания
элементов строки и столбца, содержащих ~i. Аналогичным
~ k.
~ При
образом определяются коэффициенты перед j,
~
векторе j выбран знак минус перед скобкой (ax bz − bx az ).
В соответствии со свойствами векторного произведения
общий множитель любой строки можно выносить за знак
~
~
~i
~i
j~
k
j~ k
определителя, т. е. ∆ = nax nay naz = n · m · ax ay az .
mbx mby mbz
bx by bz
Задачи для закрепления
~ и b,
~ ×b
~
~ что a
~=b
~ × a?
1. Существуют ли такие векторы a

~ j~× k,
~ k
~ × j,
~ ~i × ~i, j~× ~i, ~i × k,
~ k
~ × j.
~
2. Вычислите ~i × j,
~ × (b
~ ×b
~ + ~c) = a
~ + ~a × ~c.
3. Докажите, что a

~ × µb)
~ × b).
~ = λ · µ(a
~
4. Докажите, что (λa

Использование векторного произведения дает возможность получить «рабочие» формулы для решения ряда
стереометрических задач векторным методом. Рассмотрим
их.
1. В пространстве две скрещивающиеся прямые l1 , l2
заданы соответственно точками M, N и направляющими
~ b
~ . Расстояние ρ(l , l ) между прямыми
векторами a,
1
2
−−−→
вычисляется по формуле ρ(l1 , l2 ) = |Прa×
~ b
~ MN|.
~ к плоскости α может быть найден
2. Нормальный вектор n
~ × b.
~ b
~ Здесь a,
~ — любые неколлинеарные
по формуле a
векторы, параллельные плоскости α.
3. Расстояние ρ(l, α) между прямой l и параллельной ей
−−−→
плоскостью α вычисляется по формуле ρ(l, α) = |Прn~ MN|.
~ — любой нормальный вектор плоскости α, M —
Здесь n
произвольная точка прямой l, N — произвольная точка
плоскости α.

§ 27. Векторное произведение векторов и его свойства 181

4. Площадь треугольника ABC вычисляется по формуле
1 −−→ −−→
S = |AB × BC|, причем векторное произведение здесь
2
может быть выражено любыми векторами, составляющими
−−→ −−→
треугольник, например BC × CA.

5. Площадь плоского четырехугольника ABCD вычисляется через векторное произведение его диагоналей
1 −−→ −−→
SABCD = |AB × CD|.
2
Следует отметить, что при решении задач на вычисление
площадей сечений с помощью векторного произведения
для расчета координат точек в пространстве целесообразно
повторить с учащимися формулы деления отрезка в данном
отношении, теоремы Менелая и Фалеса.
Упражнения
1.(27.7). Найдите: а) площадь параллелограмма, построен~ = (2; 2; −2), b
~ = (3; 2; 6);
ного на векторах a
б) площадь треугольника с вершинами A(7; 3; 4),
B(1; 0; 6), C(4; 5; −2).
Решение.

~
~i j~ k
~
~
а) S = |a × b| = 2 1 −2 =
3 2 6
~ =
= (1 · 6 − 2 · (−2))~i − (2 · 6 − 3 · (−2))j~+ (2 · 2 − 3 · 1)k



~ = 100 + 324 + 1 = 425 = 5 17;
= |10~i − 18j~− k|
~
~
j~
k
1 −−→ −−→
1 i
1
б) S = |AB × AC| =
−6 −3 2 = |(14; −42; −21)| =
2
2
2
−3 2 −6
7
7 √
4 + 36 + 9 = 24,5.
= |(2; −6; −3)| =
2
2

О т в е т: 5 17, 24,5.
2(27.8). Докажите, что площадь параллелограмма, построенного на диагоналях данного параллелограмма, равна
удвоенной площади этого параллелограмма.
Решение. Пусть дан параллелограмм ABCD с диагоналями AC и BD. Его площадь можно вычислить по формуле

182

Глава 4. Перпендикулярность прямой и плоскости

1 −−→ −−→
площади плоского четырехугольника SABCD = |AC × BD|.
2
Но площадь S параллелограмма, построенного на векторах
−−→ −−→
−−→ −−→
AC, BD, вычисляется как |AC × BD|. Отсюда S = 2SABCD .

3(27.9). Боковые стороны трапеции задаются векторами ~c,
~ большее основание — вектором b.
~ Докажите, что плоd,
~ − 1 |~c × d|.
~
щадь трапеции выражается формулой S = |~c × b|
2

Решение. Достроим трапецию ABCD до паралле−−→
−−→ ~
−−→ ~
лограмма ABCE (рис. 4.82). AB = ~c, AC = b,
CD = d.
SABCD = SABCE − S△DCE . Отсюда
−−→ −−→
−−→ −−→
~ − 1 |~c × d|.
~
SABCD = |AB × AC| − 1 |CE · CD| = |~c × b|
2

2

Можно заметить, что если в условии задачи
~
меньшее основание трапеции обозначить вектором b,
−−→ −−→ 1 −−→ −−→ ~ ~ 1 ~ ~
то SABCD = |AB × BD| + |DE × CD| = |c × b| + |c × d|. Здесь
2
2
−−→ −−→
E ∈ AC, AE = BD.
B

D

A

E

C

Рис. 4.82.

4(27.10). Каждой грани произвольного тетраэдра поставлен
в соответствие вектор, модуль которого равен площади этой
грани, а сам вектор перпендикулярен ей и направлен вне
тетраэдра. Докажите, что сумма всех четырех векторов
равна нулевому вектору.
~ b,
~ ~c соответственно векторы
Решение. Обозначим через a,
−−→ −−→ −−→
AB, AC, AD тетраэдра ABCD (рис. 4.83). Тогда внешние
нормальные векторы к граням тетраэдра ABD, ACD, ABC,
~
~ ~,
CBD можно записать соответственно в виде: n
ABD = c × a






~
~
~
~ × ~c, n
~ n
n
=b
= ~a × b,
= BD × BC.
ACD

ABC

CBD

§ 27. Векторное произведение векторов и его свойства 183
D

B
A
C

Рис. 4.83.

~
~
Отметим при этом, что |n
ABD | = 2SABD , |nACD | = 2SACD ,



−−→ ~ ~
~
~
~
|n
|n
BD = ~c − a,
BC = b − a.
ABC | = 2SABC ,
CBD | = 2SCBD,
−−→ −−→ ~ ~ ~ ~ ~ ~
Вычислим BD × BC = c × b − c × a − a × b и просуммируем
−−→ −−→
~
~
~
~ +b

~ × ~c + a
n
Получим ~c × a
ABD + nACD + nABC + BD × BC.
~
~
~
~
~
~
~
~
× b + c × b − c × a − a × b = 0.
5.(27.12). Площади двух смежных граней параллелепипеда
равны S1 и S2 , а площадь диагонального сечения параллелепипеда, проходящего через общее ребро этих граней, —
S. Докажите, что S2 = S21 + S22 + 2S1 S2 cos φ, где φ — угол
между гранями с площадями S1 , S2 .
Решение. Угол между плоскостями BAA1 и DAA1
(рис. 4.84)
равен
острому
углу
между
норма−−−→ ~
−−→ ~
лями к этим плоскостям. Пусть AA1 = c, AB = b,
−−→ ~
~ + b)
~ × ~c + b
~ × ~c|2 +
~ × ~c|2 = |a
~ × ~c|2 , S2 = |a
AD = a. Тогда S2 = |(a
2
2
2
~
~
~
~
~
~
+ 2|a × c| · |b × c| cos φ + |b × c| = S1 + 2S1 S2 cos φ + S2 . Здесь
cos φ > 0, если ∠BAD 6 90◦ и cos φ < 0, если ∠BAD > 90◦.
B1

C1

A1
D1
C

B
A

D

Рис. 4.84.

184

Глава 4. Перпендикулярность прямой и плоскости

6(27.13). Докажите, что если в параллелепипеде площади
двух смежных граней равны площади проходящего через
общее ребро этих граней диагонального сечения параллелепипеда, то угол между этими гранями равен 120◦ .
Решение. Из решения предыдущей задачи следует, что
S2 = S21 + S22 + 2S1 S2 cos φ, при этом в настоящей задаче
cos φ < 0 и S1 = S2 = S. Тогда после необходимых упрощений
получим S = −2S cos φ ⇒ φ = 120◦.
7. Основанием прямой призмы ABCA1 B1 C1 служит треугольник ABC с углом B, равным 90◦ , и углом C,
равным 30◦ . Найдите площадь сечения призмы плоскостью, проходящей через середину бокового ребра CC1
и вершину B и параллельной диагонали AC1 боковой
грани AA1 C1 C, если расстояние между AC1 и секущей
плоскостью равно 1/3, а гипотенуза основания призмы
равна 4.
Решение.
В соответствии с условием задачи AB = 2,

BC = 2 3. Построим сечение (рис. 4.85): пл. ACC1:
KL k AC1 ,

KL ∩ AA1 = Q,

AQ =

1
AN = AB;
3

QB1 ∩ AB = N,
Введем систему координат:
z

1
A Q,
3 1

KLNB1 — искомое

A1

B1
C1

K
y
N

пл. ABB1 :

A
L

B
C
Q

Рис. 4.85.

x

сечение.

§ 27. Векторное произведение векторов и его свойства 185


B(0; 0; 0), A(0; 2; 0), C(2 3; 0; 0), B1 (0; 0; h), A1 (0; 2; h),





1
4
C1 (2 3; 0; h), L( 3; 1; 0), K 2 3; 0; h , N 0; ; 0 .
2

3

Найдем площадь сечения:
1 −−−→ −−→
1
SKLNB =
B1 L × NK =
1
2
2

~
j~
√i
3
1

4
2 3 −
3

~
k
−h =
h
2

~
~
j~
k

√i
1
(−5h; −15h; −20 3) =
3
1 −h =

12
12 3 −8 3h


5
5 √
=
(h; 3 3h; 4 3) =
28h2 + 48.
12
12
Чтобы
найти
h,
используем
условие
задачи:
−−→
1
ρ(AC1 , α) = . Кроме того, ρ(AC1 , α) = |Прn~ AN|, здесь
√ 3 √
~ = (h; 3 3h; 4 3). Точки A и N (A ∈ AC , N ∈ α)
n
1
выбраны как наиболее удобные по координатной записи.


−−→
~ · AN|
−−→
2
|n
2 3h
1
AN = 0; − ; 0 ,
ρ(AC1 , α) =
=p
= ⇒
3
~
3
2
|n|
28h + 48
r

1 1
= ·
2 6

⇒ h2 =

3
5
⇒S=
5
12

О т в е т:

28 · 3
3 5
+ 48 =
.
5
2


3 5
.
2

8. Найдите площадь сечений правильной треугольной
пирамиды TABC плоскостью, которая проводит через
середину бокового ребра TA, пересекает сторону основания AB в точке N, так что BN = 2AN, и параллельна
медиане основания AD, если сторона основания пирамиды
равна 12, а расстояние от вершины пирамиды T до секущей
плоскости равно 8/5.
Решение. Пусть P — середина ребра AT, точка N делит
ребро AB в отношении 2 : 1. Построим сечение пирамиды
через точки P, N параллельно AD.
Пл. ABC: NK k AD, пл. ATD : PE k AD, пл. CTB :
KE ∩ TC = L, PNKL — искомое сечение.

186

Глава 4. Перпендикулярность прямой и плоскости
z

T
L
P

T

E

B

L

N
x

E

K
D

O

A

y

C

C

Рис. 4.86.

D

K

Рис. 4.87.

Введем систему координат с началом в точке D
(рис. 4.86). При расчете координат вершин сечения PNKL
удобно использовать:
1) теорему

Фалеса:

AP
DE
=
= 1;
PT
ET

BN
BK
=
= 2,
NA
DK

△ABD:

2) теорему Менелая (рис. 4.87): △CTD :



CL
1
CL 4
·1· =1⇒
= ;
LT
4
LT 1

△ATD:

CL TE DK
·
·
=1⇒
LT ED KC

3) формулы деления отрезка в данном отношении:

xC +4·xT 8 3
=
;
1+4
5
z +4·zT 4
zL = C
= h.
1+4
5

xL =

A(6 3; 0; 0),
K(0; −2; 0), L

B(0; −6; 0),

8 3 6 4
; ; h .
5
5 5
~i

12 √
1
~ × KP
~ =1 −
3
NL
S=
5
2
2


yL =

yC +4·yT 6
= ;
1+4
5

C(0; 6; 0),

j~
16
5


T(2 3; 0; h),

~
k
4
h =
5
h
2

4 3
2
~
~
~i
j k



1 4 1
1
= · ·
(0; 11 3h; −44 3) ,
−3√ 3 4 h =
2 5 2
5
8 3 4 h

§ 27. Векторное произведение векторов и его свойства 187


11 3
11 3 √ 2
S=
|(0; h; −4)| =
h + 16.
5
5

Для

h

нахождения

8
ρ(T, α) = .
5

Приведем

используем

его

к

виду

условие

задачи
−−→
ρ(T, α) = |Прn~ KT|.

~ = (0; h; −4), K — выбрана как наиболее удобЗдесь n
ная по координатной записи точка на плоскости

−−→
2h
8
256
= ⇒ h2 =
,
сечения; KT = (2 3; 2; h), ρ(T, α) = p
5
9
2
h + 16
√ r
S=

11 3
5

256
44
+ 16 = √ .
9
3

44

О т в е т: √ .
3

9. В правильной треугольной
пирамиде SABC со стороной

a и боковым ребром a 3 плоскость α проходит через высоту
основания. Вычислите площадь наименьшего сечения
пирамиды этой плоскостью.
Решение. Сечение пирамиды плоскостью α представляет
собой треугольник DCK, вершина K которого лежит на
ребре AS. Наименьшее значение площади этого треугольника соответствует случаю, когда его высота, опущенная из
точки K, является расстоянием между скрещивающимися
прямыми DC и AS, т. е. равно ρ(AS, DC). Введем систему
координат с началом в точке D:
r 





a
a 3
a 3
; 0; 0 , C 0;
; 0 , S 0;
; 2a
2
2
6

−−→
1
1 a 3
Smin = DC·ρ(AS, DC)= ·
|Пр~ AD|.
n
2
2
2

A −

a
; 0; 0 , B
2

z

S

K
y
C
A
D
B

x

Рис. 4.88.

2
.
3

188

Глава 4. Перпендикулярность прямой и плоскости


−−→
a
Здесь AD=
; 0; 0 ;
2

~
~i
j~
k



−−→
~ (j×
~ AS)=
0
1
0
nk

√ =(4a 6; 0; −3a)k(4 2; 0; − 3).
3a a 3 4a 6
Здесь значок «k » использован для обозначения коллинеар−−→
−→
~ (j×
~ AS)⇔
~
~ −
n=k·
(j×
AS), k∈R. Можно
ности векторов,
√ т. е. nk

~
выбрать n=(4
2; 0; − 3).
−−→
~ AD


Smin =
О т в е т:




2 6
a 3 2a
a√
2

=
·
=
.
4
35
2 35

~
n


a2 6
√ .
2 35

10. Найдите площадь сечения правильной четырехугольной пирамиды TABCD плоскостью, параллельной апофеме
TL боковой грани TBC и медиане AM боковой грани
TAB и проходящей через середину бокового ребра TC,
если сторона основания пирамиды равна 8, а расстояние
от вершины пирамиды до секущей плоскости равно 40/21.
Решение. Построим сечение пирамиды плоскостью α
(рис. 4.89):
T

z

P

M

K
Q

B

y

N

L

C
A

E

D

x

Рис. 4.89.

пл. AMK: KE k AM, KE = AM ⇒ AE =
пл. BTC: KN k TL, KN =

1
TL;
2

1
AD;
2

§ 27. Векторное произведение векторов и его свойства 189

пл. ABC: EN ∩ DC = Q ∈ пл.DTC ⇒ α ∩ DTC = QP,
EPKN — искомое сечение.
Запишем координаты точек: A(0; 0; 0), T(4; 4; h),
C(8; 8; 0), E(4; 0; 0), D(8; 0; 0), N(6; 8; 0), K 6; 6;

h
. Для
2

расчета координат точки P применим теорему Менелая для
DP TK

CQ

DP

1

DP

треугольника DTC:
·
·
=1⇒
·1· =1⇒
=2.
PT KC QD
PT
2
PT
Используя формулы деления отрезка в данном отноx +2x

y +2y

16

8

T
T
xP = D
= ,
yP = D
= ,
1+2
3
3
3

2
16 8 2h
zP = h⇒P
; ;
. Запишем формулу для площади

шении,
3

получим:
3

3

3

~i
сечения: Sсеч. =

1 −−→ −−→
1
EK × PN =
2
2

j~

2

6

2
3

16
3

~
~i j~
k
1
× 4 12 h = |(−20h; 5h; 20)|.
6
1 8 −h
5
5 √
Sсеч. = |(−4h; h; 4)|=
17h2 +16.
6

6

используем условие ρ(T, α)=

~
k
h
1 1 2
= · · ×
2
2 2 3
2h

3

Для

нахождения

h

40
. В то же время ρ(T, α)
21

−−→ ~
можно представить в виде ρ(T, α)=|Пр~ TE|, n=(−4h;
h; 4),
−−→

~ TE|
−−→
40 |n·
TE=(0; 4; h)⇒ =
⇒p

n

h

5
.
21
5 √
35
25·17+16= .
Отсюда h2 =25⇒Sсеч. =
6
2
21

~
|n|

17h2 +16

=

О т в е т: Sсеч. = 17,5.
11. В кубе ABCBA1B1 C1 D1 со стороной a точка K является
серединой стороны верхнего основания B1 C1 , точка L делит
другую сторону C1 D1 этого основания в отношении 2 : 1,
считая от вершины C1 , точка N является серединой бокового ребра AA1 . Найдите площадь сечения, проходящего
через точки K, L, N.
Решение. Построим сечение куба через точки K, L,
N (рис. 4.90), пл. A1 B1 C1 : KL ∩ A1 D1 = Q, пл. AA1 D1 :

190

Глава 4. Перпендикулярность прямой и плоскости
B1

z

K

C1
L

G

D1

A1

Q
T
y
N
C

B
x

D

A

Рис. 4.90.

NQ ∩ DD1 = T, пл. BB1 C : KG k TN, NTLKG — искомое
сечение. Площадь сечения вычислим, используя формулу Sпр = S · cos φ, где φ — угол между нормальными
векторами плоскости основания куба и плоскости сечения. Площадь проекции сечения куба на плоскость
ABC можно вычислить как разность SABCD − S△KLC =
1

1
1 1 2
5
= a − KC1 · C1 L = a2 − · · a2 = a2 .
2
2 2 3
6
2

В декартовой
системе координат с центром в вершине куба A коор
a

a

вершин имеют вид K
; a; a , L a; ; a ,
2
3



−−→
−−→
a
a
a
a a
N 0; 0;
. Отсюда NK =
; a;
, NL = a; ;
. Нор-

динаты

2

2

2

3

2

~ сечения можно принять пропормальный вектор n
циональным (коллинеарным) векторному произведению
~
~i j~ k
~
~i j~ k
1
1
−−→ −−→
−−→ −−→ 1
~ k NL
NL × NK: n
× NK k
= (−4; −3; 10).
k
6
2
3
3 2
1
2
1
1
1
1
2
2
~ = (0; 0; 1).
Нормальный вектор плоскости основания — k
Тогда cos φ =

О т в е т:



~

~k
~ k
~
n

5 2
a .
3

10

= √

5 5

и S=

Sпр
cos φ

=


5 2 10
5 2
a · √ =
a .
6
3
5 5

Глава 5

МНОГОГРАННЫЕ УГЛЫ

§ 28. ТРЕХГРАННЫЕ УГЛЫ

При изучении темы необходимо вернуться к понятию
многогранника и его элементов, повторить, как определяются ребра, грани, вершины многогранника, его плоские
и двугранные углы. Затем дать определение многогранного
угла. Следует отметить, что многие свойства трехгранного
угла, а именно вторая теорема косинусов для трехгранного
угла, теорема синусов для трехгранного угла, теорема
о трех синусах, рассмотренные в § 28 учебника, достаточно
специфичны, используются для ограниченного круга задач.
Поэтому изучение этих свойств лучше проводить на примерах отдельных теоретических задач на доказательство.
Упражнения и задачи для закрепления
1(28.1). Могут ли плоские углы трехгранного угла быть
равны: а) 120◦ , 75◦ , 40◦ ; б) 90◦ , 60◦ , 40◦ ; в) 130◦ , 110◦ ,
145◦ ?
О т в е т: а) нет; б) да; в) нет.
2(28.2). Докажите, что если сумма плоских углов трехгранного угла равна 180◦ , то все эти углы — острые.
Решение. Если сумма плоских углов трехгранного угла
равна 180◦ , то все эти углы острые, так как в этом случае
тупой угол был бы больше суммы двух остальных плоских
углов, что невозможно.
3(28.3). Можно ли в сечении правильного тетраэдра плоскостью получить:
а) трапецию; б) прямоугольник; в) квадрат; г) параллелограмм, отличный от прямоугольника?
О т в е т: а) да; б) да; в) да; г) нет.

192

Глава 5. Многогранные углы

4(28.4). Докажите, что не существует плоскости, перпендикулярной всем трем граням трехгранного угла.
Решение. Предположим противное. Пусть
перпендикулярна всем трем граням данного
угла. Тогда ребра этого угла параллельны
дикуляры к плоскости α, что противоречит
трехгранного угла.

плоскость α
трехгранного
как перпенопределению

5(28.6). Могут ли двугранные углы трехгранного угла быть
равны:
а) 70◦ , 60◦ , 80◦ ; б) 30◦ , 45◦ , 100◦ ; в) 120◦ , 150◦ , 60◦ ?
О т в е т: а) да; б) нет; в) да.
6(28.7). Докажите, что если двугранные углы трехгранного
угла равны, то каждый из них больше 60◦ .
Решение. Пусть каждый двугранный угол трехгранного
угла равен α. Тогда 3α > 180◦ и α > 60◦ .
7(28.8). Докажите, что если плоские углы трехгранного
угла равны, то равны и его двугранные углы.
Решение. Отложим от вершины M трехгранного угла равные отрезки MA, MB и MC (рис. 5.1). Тогда треугольники
AMB, BMC и AMC равны по двум
сторонам и углу между ними, откуда
AB = BC = CA.
Пусть
AD1 ⊥ BM,
BD

CM,
CD

AM.
Тогда
углы
2
3
D
AD1 C,
BD2 A,
CD3 B
являются
A
C линейными углами соответствующих
двугранных углов, но так как △AD1 C =
B
= △BD2 A = △CD3B (по трем сторонам),
Рис. 5.1.
то ∠AD1 C = ∠BD2A = ∠CD3B.
8(28.9). Плоские углы трехгранного угла равны 60◦ , 60◦
и 90◦ . На общем ребре двух равных плоских углов отложен
отрезок MA длины a. Найдите:
а) длину проекции MA на плоскость третьей грани,
б) угол, который образует ребро MA с плоскостью
третьей грани.
a
О т в е т: а) √ ; б) 45◦ .
M

2

§ 28. Трехгранные углы 193

9(28.10). Трехгранный угол называется прямым, если все
его плоские углы прямые. Докажите, что все двугранные
углы прямого трехгранного угла также являются прямыми.
Решение. Плоские углы прямого трехгранного угла являются линейными углами двугранных углов этого угла.
10(28.11). На ребрах прямого трехгранного угла с вершиной M взяты точки A, B и C. Докажите, что проекция
точки M на плоскость ABC совпадает с точкой пересечения
высот треугольника ABC.
Решение. Обозначим через H проекцию вершины M на
плоскость ABC. Поскольку ребро MC перпендикулярно
плоскости ABM, MC ⊥ AB. Так как MH — высота тетраэдра MABC, то MH ⊥ AB. Отсюда ребро AB перпендикулярно плоскости CMH, а, значит, AB ⊥ CH и BH ⊥ AC.
11(28.12). Найдите геометрическое место точек, равноудаленных от плоскостей всех трех граней трехгранного угла
и лежащих внутри этого угла.
Решение. Геометрическим местом точек, равноудаленных
от плоскостей граней двугранного угла, является биссекторная плоскость, которая делит пополам линейный угол
этого двугранного угла.
Если MABC — данный трехгранный угол, то биссекторные плоскости каких-либо двух двугранных углов этого
угла пересекаются по прямой l, каждая точка которой
равноудалена от плоскостей всех трех граней данного трехгранного угла, поэтому биссекторная плоскость третьего
двугранного угла также проходит через прямую l, которая
и является искомым геометрическим местом точек.
12. Докажите, что сумма величин двугранных углов трехгранного угла больше 180◦ .
Решение. Пусть двугранные углы трехгранного угла
MABC равны A, B и C. Возьмем внутри угла MABC
точку M1 и опустим перпендикуляры MA1 , MB1, MC1 на
грани BMC, AMC, AMB соответственно (рис. 5.2). Обозначим ∠B1 M1 C1 = α1 , ∠A1 M1 C1 = β1, ∠A1 M1 B1 = γ1. Тогда

194

Глава 5. Многогранные углы
M

A1
B1
A

C1

M1
B

C

α1 = 180◦ — A, β1 = 180◦ − B, γ1 = 180◦ − C.
α1 ,
β1 ,
γ1 — плоские
Поскольку
углы трехгранного угла, выполняются
неравенства α1 + β1 + γ1 < 360◦
и A + B + C = 540◦ − (α1 + β1 + γ1) > 540◦ −
− 360◦ = 180◦ .

13(28.15). Докажите, что если двугранные углы трехгранного угла равны, то
равны и его плоские углы.
Рис. 5.2.

Решение. Пусть точка S1 лежит внутри трехгранного
угла SABC, A1 , B1 , C1 — основания перпендикуляров,
опущенных из нее на грани BSC, ASC, ASB соответственно
(рис. 5.3), ∠BCS = α; ∠CSA = β; ∠ASB = γ, ∠B1 S1 C1 = α1 ,
∠C1 S1 A1 = β1, ∠A1 S1 B1 = γ1 , A, B, C — величины двугранных углов трехгранного угла SABC с ребрами SA, SB, SC
соответственно, P — точка пересечения плоскости B1 S1 C1
с ребром SA.
Поскольку прямые S1 B1 и S1 C1
S
перпендикулярны граням ASC и ASB,
ребро SA перпендикулярно плоскости B1 S1 C1 и, таким образом,
B1 A
1
P
угол
B1 PC1
является
линейным
углом
двугранного
угла
при
ребре
S1
C
C1
A
SA,
т. е.
∠B1 PC1 = A.
Тогда
из
B
четырехугольника
B1 S1 C1 P
находим:
∠B1 S1 C1 +B1 PC1 =α+A=360◦ −
Рис. 5.3.
−(∠PB1 S1 +∠PC1 S1 )=360◦ −(90◦ +90◦ )=
=180◦ . Аналогично, β1 +B=γ1 +C=180◦.
Итак, из равенства углов A, B, C вытекает равенство
плоских углов α1 , β1 , γ1 трехгранного угла S1 A1 B1 C1 .
Но в этом случае равны и двугранные углы A1 , B1 ,
C1 трехгранного угла SA1 B1 C1 . Учитывая равенства
A1 +α=B1 +β=C1 +γ=180◦, получаем, что α=β=γ, т. е. плоские углы трехгранного угла SABC равны.
14. Плоские углы трехгранного угла равны α, β, γ,
а противоположные им двугранные углы — A, B и C.
Докажите, что cos α=

cos A+cos B·cos C
.
sin B·sin C

§ 28. Трехгранные углы 195

Решение. Возьмем внутри угла SABC точку S1 и опустим
перпендикуляры S1 A1 , S1 B1 , S1 C1 соответственно на
грани BSC, ASC, ASB (рис. 5.3 к задаче 13). Обозначим
плоские углы трехгранного угла S1 A1 B1 C1 через α1 ,
β1 , γ1 , а двугранные углы этого угла — через A1 , B1 ,
C1 . Тогда α1 =π−A, β1 =π−B, γ1 =π−C; A1 =π−α, B1 =π−β,
C1 =π−γ. Применяя теорему косинусов к трехгранному
cos α1 −cos β1 ·cos γ1
или
sin β1 ·sin γ1
− cos A−cos B·cos C
cos A+cos B·cos C
− cos α=
, т. е. cos α=
.
sin B·sin C
sin B·sin C

углу S1 A1 B1 C1 , получим: cos A1 =

15(28.17). Докажите, что если три двугранных угла одного
трехгранного угла соответственно равны трем двугранным
углам другого трехгранного угла, то такие трехгранные
углы равны.
Решение. Исходя из определения трехгранного угла,
достаточно доказать равенство соответствующих плоских углов для заданных углов трехгранных. Обратим внимание на результат задачи 14. Применим его:
cos A+cos B·cos C

. Косинусы плоских углов выраcos α=
sin B·sin C
жаются через тригонометрические функции двугранных
углов, которые, по условию задачи, одинаковы для углов
трехгранных. Отсюда вытекает необходимое для решения
задачи равенство этих плоских углов.
16 (теорема синусов для трехгранного угла). Пусть плоские
углы трехгранного угла равны α, β, γ, а противоположные им двугранные углы — A, B, C. Докажите, что
sin α sin β sin γ
=
=
для случая, когда α, β, γ — острые углы.
sin A sin B sin C

Решение. Обозначим ∠ASB=β, ∠ASC=γ. Проведем перпендикуляр AH из точки A на ребре SA к плоскости SBC,
а также перпендикуляры AP и AQ к прямым SB и SC
соответственно. Поскольку α, β, γ — острые углы, высоты
AP, AQ окажутся на гранях трехгранного угла. По теореме
о трех перпендикулярах, HP⊥SB, HQ⊥SC⇒∠B=∠APH,
∠C=∠AQH (рис. 5.4). Тогда из прямоугольных треугольников AHP, AHQ получим: AH=AP sin B=AQ sin C. Прямоугольные треугольники APS, AQS, в свою очередь, дают

196

Глава 5. Многогранные углы
A

B
P

S

H
Q
C

Рис. 5.4.

равенства AP=AS sin β, AQ=AS sin γ. Отсюда следует,
sin β

sin γ

=
. Аналогично доказываются равенства для
что
sin B sin C
остальных пар углов.
17(28.18). Постройте плоскость, пересекающую данный
трехгранный угол и образующую с его ребрами равные
углы.
Решение. Отложим от вершины трехгранного угла
ABCE отрезки EA1 , EB1 , EC1 одинаковой длины.
Докажем, что плоскость A1 B1 C1
является
искомой. Пусть точка O является центром окружности, описанной около треугольника A1 B1 C1 . Рассмотрим треугольники EA1 O, EB1 O, EC1 O. Они равны
по трем сторонам (поэтому ∠EA1 O=∠EB1O=∠EC1 O)
и
являются
прямоугольными.
8−−→ −−−→ −−Действительно,
−→

)=0,
1
EO· OA1 = EO· OB1 = EO· OC1 ⇒ > −−→ −−−→ −−−→1
Обо: EO·(OA − OC )=0.
1
1
−−−→ −−−→
−−−→ −−−→ ~
значим OA1 − OB1 = a
~ , OA1 − OC1 = b.
Это означает, что
прямая EO перпендикулярна двум непараллельным
прямым плоскости ABC, т. е. EO⊥ABC. Отсюда следует,
что A1 O, B1 O, C1 O — проекции отрезков EA, EB, EC
на плоскость ABC и каждый из равных углов ∠EA1 O,
∠EB1 O, ∠EC1 O является углом наклона соответствующего
ребра к плоскости ABC.
17(28.20). Сумма плоских углов трехгранного угла равна
180◦ . Докажите, что сумма косинусов его двугранных углов
равна 1.

§ 29. Многогранные углы 197

Решение.

По

теореме

косинусов

для

трехгранного

cos A+cos B cos C
cos B+cos A cos C
угла, cos α=
, cos β=
, cos γ=
sin B sin C
sin A sin C
cos C+cos A cos B
. Из тео=cos(180◦ −(α+β))=− cos(α+β)=
sin B sin A

ремы синусов (задача 15) следует, что

sin α = sin β = sin γ ⇒sin γ=sin(α+β)=sin α cos β+
sin A sin B sin C

+cos α sin β=t sin A cos β+t cos α sin B=t sin C.
Здесь t — некоторое число. Тогда sin A cos β + cos α sin B =
= sin C. В то же время
sin C = cos B + cos A cos C + cos A + cos B cos C ⇒
sin C

sin C

⇒ sin2 C = cos B + cos A cos C + cos A + cos B cos C ⇒
⇒ 1 − cos2 C = (cos A + cos B)(1 + cos C) ⇒
⇒ cos A + cos B + cos C = 1.

§ 29. МНОГОГРАННЫЕ УГЛЫ

Теоретический материал этого параграфа является обобщением сведений по тематике двугранного и трехгранного
углов на многомерный случай. Решение задач § 29 служит
основой для последующего изучения свойств выпуклых
многогранников 11 классе.
Упражнения и задачи
1(29.1). Могут ли плоские углы многогранного угла быть
равны: а) 50◦ , 40◦ , 130◦ , 150◦ ; б) 35◦ , 75◦ , 80◦ , 45◦ , 125◦ ;
в) 70◦ , 40◦ , 80◦ ?
О т в е т: а) нет; б) нет; в) да.
2(29.2). Докажите, что если сумма плоских углов многогранного угла равна 180◦ , то все эти плоские углы острые.
Решение. Пусть, например, один из плоских углов многогранного угла не является острым, т. е. какой-либо угол
β > 90◦ . Тогда сумма величин остальных плоских углов не
должна превышать 90◦ , тем самым нарушается свойство 1
многогранных углов.

198

Глава 5. Многогранные углы

3(29.3). Докажите, что каждый плоский угол многогранного угла меньше суммы остальных плоских углов, опираясь на аналогичное утверждение для трехгранного угла.
Решение. Пусть α1 — один из плоских углов многогранного угла с ребрами AA1 , AA2 , . . ., AAn , α1 = ∠A1AA2 .
Обозначим
остальные
плоские
углы
α2 = ∠A2 AA3 ,
α3 = ∠A3AA4 ,. . . , αn = ∠An AA1 . Рассмотрим трехгранный угол с ребрами AA1 , AA2 , AA3 . По свойству
трехгранного угла, α1 < α2 + ∠A1AA3 . Далее по тому
же свойству ∠A1 AA3 < α3 + ∠A1 AA4 . Продолжая цепочку
неравенств, в итоге придем к ∠A1 AAn−1 < αn−1 + ∠A1AAn .
Но ∠A1 AAn = αn , поэтому α1 < α2 + α3 + . . . + αn .
4(29.4). Докажите, что сумма двугранных углов n-гранного
угла больше (n − 2) 180◦ .
Решение. Пусть имеется n-гранный угол с ребрами
AA1 , AA2 , . . ., AAn .
Возьмем
произвольную
точку M внутри этого угла и построим n-гранный
угол MA1 A2 . . .An , полярный данному. По свойству
полярных углов, A1 + α1 = A2 + α2 = . . . = An + αn = 180◦ .
A1 , A2 , . . ., An — двугранные
углы
исходного
Здесь
угла AA1 A2 . . .An , а α1 , α2 , . . ., αn — плоские углы при
вершине M угла MA1 A2 . . .An . Отметим, что, по
свойству плоских углов, α1 +α2 +. . .+αn